You are on page 1of 149

lOMoAR cPSD| 41174528

1) Paeds Final MCQs Corrected 230307 141005

Pediatrics (М е д и ц и н с к и
у н и в е р с и т е т в Пл о в д и в )

Scan to open on Studocu


Studocu is not sponsored or endorsed by any college or university
Downloaded by Kato Caleb (katocalebnatamba@gmail.com)
lOMoAR cPSD| 41174528

Mon+Tue+Wed+Thursday (Page 28-54)

Wednesday

1. At what age you would expect a child to smile in response to his mother’s smile?
(1 correct answer)
(1 Point)
15 weeks
6weeks
18 weeksия 1
1 weeks

2. The most common cause for obesity in children is:


(1 correct answer)
(1 Point)
Overnutrition (common reason => diet & lack of physical activity)
type 2 diabetes mellitus
hypothalamic disorders
chromosome disorders

3. Which of the following conditions is an absolute contraindication for breastfeeding?


(1 correct answer)
(1 Point)
prematurity
Galactosemia (baby is not able to metabolize galactose in the milk properly, hence must be
stopped)
breast milk jaundice (continue breast-feeding - prevented by freq feedings)
Hypoglycemia

4. What are the most common causes for infant mortality worldwide?
(1 correct answer)
(1 Point)
HIV, malaria
congenitalabnormalities,prematurity
malnutrition
injuries, drownings

5. The triad of congenital heart defect, blindness and deafness is typical for which
congenital infection:
(1 correct answer)
(1 Point)
congenitalrubella (3 C’s => Cardiac defect; cochlear defect & cataract)
congenital cytomegalovirus infection
congenital toxoplasmosis

Downloaded by Kato Caleb (katocalebnatamba@gmail.com)


lOMoAR cPSD| 41174528

congenital varicella

6. Which is the most severe consequence of hypoxic-ischemic encephalopathy?


(1 correct answer)
(1 Point)
Brain calcifications
Bilateral damage to the basal ganglia - PVL + lesions in thalamus + basal ganglia
Intraventricular hemorrhage grade I
Multicystic encephalomalacia
L varyingsizedcysticlesions inthebrainencountered indevelopingretire
or infants
7. The most common cause of acute gastroenteritis in childhood is:
(1 correct answer)
(1 Point)
Viruses (Most frequent cause => rotavirus infection. Accounts for 60% of the cases in children <
2yrs)
fungi
toxins
bacteria - Shigella, Salmonella, Campylobacter

8. Autoimmune hepatitis may occur in association with:


(1 correct answer)
(1 Point)
Wilson’s disease
Cystic fibrosis
Inflammatoryboweldisease (Autoimmune hepatitis is associated with other autoimmune
diseases such as IBS; Hashimoto thyroiditis; SLE; Grave’s disease etc)
Thalassemia major

9. In persistent ductus arteriosus there is:


(1 correct answer)
(1 Point)
reduced pulmonary vascular markings on chest X-ray
continuousmurmurbelowtheleftclavicle
weak peripheral pulses
Cyanosis

10. Which of the following is NOT true for coarctation of the aorta in infants:
(1 correct answer)
(1 Point)
presents with cardiogenic shock - if very severe = >PDA closure -- heart unable to pump against
such narrowing
is one of the duct-dependent congenital heart defects in the neonatal period - no cyanosis until
DA closes

Downloaded by Kato Caleb (katocalebnatamba@gmail.com)


lOMoAR cPSD| 41174528

presentsalwaysimmediatelyafterbirth (There is post-ductal or adult form which is common in


adults)
chest X-ray shows cardiomegaly -

11. In case of a foreign body in the larynx there is:


(1 correct answer)
(1 Point)
Expiratory dyspnea
Saliva leakage
Inspiratorydyspnea - inspiratory stridor
Valve mechanism and emphysema

12. Arterial hypertension with paroxysmal palpitations, flushing and sweating can be found
in:
(1 correct answer)
(1 Point)
Supraventricular tachycardia
Congenital adrenal hyperplasia
Coarctation of the aorta
Pheochromocytoma (characterized by tachycardia; flushing & sweating)

13. Which of the following conditions is common in patients with cerebral palsy and other
severe brain injuries?
(1 correct answer)
(1 Point)
Obesity
Premature puberty
Gastroesophagealreflux - common esp in CP - probs w/feeding, swallowing,poor posture etc
Macrocephaly (slow head growth)

14. In congenital adrenal hyperplasia there is:


(1 correct answer)
(1 Point)
Normal pubertal development
False premature iso- or heterosexual puberty (peripheral precocious puberty => due to excess
androgen secretion) - normal external genetalia in males but pseudohermaphroditism in
females
Delayed bone age
True precocious puberty

15. Healthy infants double their birth weight around what age?
(1 correct answer)
(1 Point)
7 -8 months

Downloaded by Kato Caleb (katocalebnatamba@gmail.com)


lOMoAR cPSD| 41174528

4-5months
9 -10months
11-12 months - triple

16. Which of the following conditions predisposes to hypocalcemia and rickets.


(1 Point)
Cerebral palsy
Congenital heart defect
Recurrent urinary tract infections
Coeliacdisease ( Vitamin D malabsorption. This further reduces absorption of calcium => Rickets
& Hypocalcemia)

17. Bilirubin can be found in the urine of newborns in all EXCEPT:


(1 correct answer)
(1 Point)
neonatal hepatitis (Increased amount of conjugated bilirubin=> passes in urine)
atresia of bile ducts (Increased amount of C. bilirubin => passes in urine)
haemolytic disease of the newborn (Increase amount of unconjugated bilirubin)
congenital infection (Increased amount of C. bilirubin => passes in urine)

18. The innocent / accidental / heart murmurs are characterized by the following features:
(1 correct answer)
(1 Point)
Theyareusuallysystolic. (5’s => asymptomatic; soft; systolic murmurs only; heard on left sternal
edge & normal S1 & S2 sounds heard)
They are combined with cardiomegaly
They often propagate to the axilla.
They are usually loud - 3-4th degree.

19. The main characteristics of nephrotic syndrome are:


(1 correct answer)
(1 Point)
Proteinuria, hematuria, hyperlipidemia, edema
Proteinuria, leukocyturia, edema
Proteinuria,hypoalbuminaemia,hyperlipidemia,edema
Proteinuria, hematuria, leukocyturia, edema

20. Which are the main prophylactic/preventer medications in asthma in children:


(1 correct answer)
(1 Point)
Shortacting beta-2 agonists (Relievers)
Antihistamines
Inhaledsteroids (Inhaled glucocorticoids such as fluticasone- 1st line)
Theophylline

Downloaded by Kato Caleb (katocalebnatamba@gmail.com)


lOMoAR cPSD| 41174528

21. Which newborns are at risk of necrotizing enterocolitis:


(1 correct answer)
(1 Point)
Neonates with congenital infection
Neonates born to mother with poorly controlled diabetes mellitus
Severelyprematureneonates (Bowel of preterm babies is vulnerable to ischemic injuries &
bacterial invasion)
Children with congenital abnormalities of the gastrointestinal tract.

22. In which condition is the “sunsetting eyes” sign observed?


(1 correct answer)
(1 Point)
Microcephaly
Hydrocephalus (In hydrocephalus => Bulging anterior fontanelle + eyes deviate downwards =>
Sun-setting eyes) - Sx of ͫICP, disproportionately large head
Spinal muscular atrophy
Down syndrome

23. Which of the following diseases is characterized by short stature and absent puberty:
(1 correct answer)
(1 Point)
Turnersyndrome
Down syndrome
Graves disease
Kleinfelter syndrome

24. Haemorrhagic disease of the newborn is due to:


(1 correct answer)
(1 Point)
hypofibrinogenemia
deficiencyofvit.Kdependentfactorsofcoagulation - 2,7,9,10 - ͫPT
factor VII deficiency
factor IX deficiency

25. Expiratory dyspnea is a sign in:


(1 correct answer)
(1 Point)
viral laryngotracheobronchitis (croup)
bronchialasthma - prolonged expiration + air trapping
retropharyngeal abscess
Epiglottitis

26. The most common cause of acute bronchiolitis is:

Downloaded by Kato Caleb (katocalebnatamba@gmail.com)


lOMoAR cPSD| 41174528

(1 correct answer)
(1 Point)
respiratorysyncytialvirus/RSV/ (80% of the cases)
influenza virus
adenovirus
Rhinovirus

27. What is the most common type of diabetes in children?


(1 correct answer)
(1 Point)
MODI
Type 2 diabetes
LADA
Type1diabetes

28. Fluctuating during the day muscle weakness and bulbar manifestations аre typical for:
(1 correct answer)
(1 Point)
Cerebral palsy
Spinal muscular atrophy
Progressive muscular dystrophy
Myastheniagravis (Fluctuating weakness involving specific muscle groups. Ocular muscle groups
are primarily affected)

29. What is the mode of inheritance of glucose 6-phosphate- dehydrogenase deficiency


anemia?
(1 correct answer)
(1 Point)
Autosomal recessive
None of the above
Autosomal dominant
X-linked

30. Iron deficiency anemia is characterized by:


(1 correct answer)
(1 Point)
Low iron; low ferritin; high MCV
Lowiron;lowferritin;lowMCV (with increased TIBC)
High iron; high ferritin; high MCV
Low iron; high ferritin; low MCV - anemia of chronic inflammation

31. What are the possible complications of juvenile idiopathic arthritis:


(1 correct answer)
(1 Point)

Downloaded by Kato Caleb (katocalebnatamba@gmail.com)


lOMoAR cPSD| 41174528

chronic anterior uveitis-- leads to blindness


growth failure
joint contractures - also knock knees, limping, swan/boutonniere deformity of hands
alloftheabove

32. In which condition is the “test with the three cups” positive for blood in the last cup?
(1 correct answer)
(1 Point)
Urethral trauma - 1st glass
Pyelonephritis - all cups - upper UT, bladder
Acute post streptococcal glomerulonephritis - all cups
Hemorrhagiccystitis - last cup (+) - bladder neck + triangular alrea, posterior urethra

33. Which of the following statements is incorrect?


(1 correct answer)
(1 Point)
Acute kidney injury in childhood has a good prognosis
Acute kidney injury is usually a reversible decrease in kidney function
Acute kidney injury is usually manifested with anemia due to erythropoietin deficiency and
impaired bone mineralization (This is true for CKD) - needs time for lack of EPO + VitD to causes
anemia + rickets
Oliguria is usually present in acute kidney injury

34. What is NOT true for chronic non-specific diarrhea:


(1 correct answer)
(1 Point)
the children have normal growth
there are undigested vegetable particles in the stools
usually in children who drink a lot of fruit juice (or in children with undiagnosed celiac disease)
thechildrenoftenhaveanemia - normal physical + growth exams

35. What is NOT part of the extramedullary blast infiltration syndrome in acute lymphoblastic
leukemia:
(1 correct answer)
(1 Point)
petechialrash - {seen as a result of thrombocytopenia due to bone marrow infiltration =
intramedullary }
Lymphadenomegaly - extramedullary
hepatomegaly- extramedullary
renal involvement (also affects the bone marrow; reticulo-endothelial system- liver,
spleen,lymph nodes; CNS & testes) . - - extramedullary

36. 4 months old boy was admitted to the hospital for investigations. He was born from normal
pregnancy with weight 2900 g. Shortly after the delivery a systolic heart murmur on the left

Downloaded by Kato Caleb (katocalebnatamba@gmail.com)


lOMoAR cPSD| 41174528

sternal edge was heard on the routine examination of the newborn. Lately the mother has
noticed that the child is more irritable, and his lips and tongue turn blue when crying. X-ray of
the child is shown below.
What is the most likely diagnosis?
(1 correct answer)
(2 Points)
Transposition of the great arteries
TetrologyofFallot - tet spells common in infants esp 2-6 mo
Myocardiopathy
Coarctation of the aorta

37. A 4 months old boy is transferred from another hospital with the diagnosis of Di George's
syndrome. Which of the following findings proves this diagnosis?
(1 correct answer)
(2 Points)
B-cell immune deficiency (Mainly T cell immune deficiency)
Disorders of phagocytosis
Lack of neutrophils
Lackofathymusgland (Mainly characterized by unusual facial appearance such as
underdeveloped chin, eyes with heavy eyelids, ears that are rotated back; heart defects &
thymus gland abnormalities. In complete Di George’s => lack thymus gland)

38. An 8-year- old child is brought to the Emergency department with complaints of repeated
vomiting, profuse diarrhea and abdominal pains that started 2 hours after consumption of wild
mushrooms. Choose the right statement:
(1 correct answer)
(2 Points)
This is Amanita phalloides poisoning, start immediate treatment with Silibinin . -typical onset >6
hrs
This is probably irritative mushroom poisoning, start treatment with rehydration and activated
charcoal
Plasmapheresis must be performed within the first 24 hours
We should expect development of severe liver damage
alwayslookat lymphedema in he surrounding
39. A 16-year-old girl from norm patl pregnancy of a 40-year- old mother wtaisssbueosrn with mild
6 tefdsy
intrauterine retardation / -1 SD / and swelling of the wrists and feet. During childhood she has
had stunted growth and learning difficulties at school.The reason for consulting a pediatric
endocrinologist is her short height -140 cm and absence of menstruation.
What is the most likely diagnosis?
(1 correct answer)
(2 Points)
Constitutional delay in growth and puberty (although delayed, they should have showed signs
of puberty + have normal growth during childhood)
Down syndrome (they undergo puberty)

Downloaded by Kato Caleb (katocalebnatamba@gmail.com)


lOMoAR cPSD| 41174528

Kleinfelter syndrome (tall stature)


Turnersyndrome - short stature, amenorrhea, infertility,...

40. A 15-year old girl is finishing her treatment for tuberculosis (TB) of the tracheo-bronchial
lymph nodes. For the last 10 days she has been complaining of chest pain which is worse on
lying down and during deep inspiration and is relieved by sitting up. On examination her HR is
110/min at rest, there are no murmurs but her heart sounds are muffled. Chest X-ray shows
cardiomegaly. ECG is with ST elevation. What is the most likely diagnosis:
(1 correct answer) please on allleads
(2 Points)
dilated cardiomyopathy
TB myocarditis
TB endocarditis
TBpericarditis (history of previous infection, ST elevation, cardiomegaly => Pericarditis) -
improvement w/sitting up ..

41. A 5-year-old girl was hospitalized due to macroscopic hematuria for several days and a
palpable mass in the right abdominal half, not crossing the midline. On physical examination she
looks well. What is the most likely diagnosis?
(1 correct answer)
(2 Points)
Nephroblastoma (doesn’t cross midline + hematuria)
Pheochromocytoma
Congenital hydronephrosis
Neuroblastoma (crosses the midline)

42. The pubertal development of a 14 years old boy started at 11 years of age with testicular
enlargement followed by an increase of the penis, pubic and axillary hair growth, mutation of
the voice and growth spurt of about 10 cm during the last year. His bone age corresponds to his
chronological age.
What is the most likely diagnosis?
(1 correct answer)
(2 Points)
Gonadotropin-dependent / centralprematurepuberty Gonadotropin-independent /
peripheralprematurepuberty Constitutionaldelayingrowthandpuberty (no puberty onset/ no
testicular enlargement > 14 yrs). Physiologicalpuberty

43. A 5-month-old infant has a dry cough almost every night. The infant has normal growth and
neurodevelopment. Mother reports frequent regurgitations and some vomiting of breast milk.
Examination of his respiratory system is normal. What is the most likely diagnosis?
(1 correct answer)
(2 Points)
bronchiolitis

Downloaded by Kato Caleb (katocalebnatamba@gmail.com)


lOMoAR cPSD| 41174528

asthma
gastro-esophagealreflux
cystic fibrosis

44. An 18-month-old boy is admitted at the Pediatric Department for abdominal bloating,
irritability, loss of appetite and intermittent diarrhoea. On examination he looks unwell and is
irritable, with reduced subcutaneous fatty tissue, distended abdomen. Laboratory results show
significant increase of anti-tissue transglutaminase antibodies (anti-tTG Ab) and endomysial
antibodies (EMA).
What is your diagnosis:
(1 correct answer)
(2 Points)
Milk protein intolerance
Cystic fibrosis
Coeliacdisease (anti-TTG antibodies. antiEMA)
Crohn’s disease

45. A 13-year-old girl presents with a runny nose, fever and sore throat. Her family doctor
diagnoses her with acute tonsillitis, but also notices that she has a goitre. Family history –
mother has thyroid disease.
Laboratory studies - TSH- 4.2 (0.34 - 5.6) mIU / L; fT4-12 (7-14.4) pmol / L; anti-thyroid
peroxidase (TPO) antibodies 584 (0-9) IU / mL; TSH-receptor antibodies-0.4 (0-1.8) IU / L.
What is the most likely diagnosis:
(1 correct answer)
(2 Points)
Graves disease
Sporadic goiter
Acute thyroiditis
Hashimoto'sthyroiditis (due to the presence of TPO antibodies).

46. A 1year 7 months old boy from normal pregnancy and delivery is still only babbling and says
no distinct words. He is able to walk, scribbles with pen and feed himself. He points to the
objects he wants and looks at the eyes of his mum when frightened. What is the most
appropriate first action?
(1 correct answer)
(2 Points)
Perform a full developmental assessment (neurodevelopment is normal)
Assessment for autism spectrum disorder
Hearingtest
Reassurance of the parents

47. An adult cardiologist examines the ECG of a 10 months old infant and finds lots of
peculiarities. Which of the following is normal:
(1 correct answer)

Downloaded by Kato Caleb (katocalebnatamba@gmail.com)


lOMoAR cPSD| 41174528

(2 Points)
left axis deviations
physiological A-V block first degree
low-voltage of the QRS complexes
right axis deviations - + T wave inversions = normal

48. A 5 years old boy suffers from severe Hemophilia A with hemarthrosis every week. What is
best treatment option?
(1 correct answer)
(2 Points)
Immobilization
Prophylacticinjectionofcoagulationfactorconcentrate - F8
Restricted physical activity
On demand injection of coagulation factor concentrate (when bleeds occur)

49. A 3 years old boy was consulted by a child rheumatologist because of high fever and
suggested Kawasaki's disease. The consultant excluded this diagnosis because he observed one
of the findings below. Which of these findings EXCLUDES Kawasaki's disease?
(1 correct answer)
(2 Points)
polymorphous rash
fever lasting >5 days
suppurativebilateralconjunctivitis (bilateral non-purulent conjunctivitis is in the diagnosis).
mucous membranes changes - dry, fissured lips, strawberry tongue

50. A 14-year-old girl became acutely unwell with fever, sore throat and malaise. Because
symptomatic treatment had no effect the family doctor prescribed her Ampicillin. The next day
she develops a maculo-papular rash on the body. On examination she is febrile- 38°C, has mild
edema of the eyelids, severe nasal obstruction, neck lymphadenopathy, enlarged tonsils with
grayish exudate on them. Lab results show: leucocytes18х10^9, with 70% lymphocytes, one
third of them being similar to monocytes, Hgb and platelets within reference range. Mild
elevation of ASAT and ALAT.CRP- 5 mg/ml.
Which is the most likely diagnosis:
(1 correct answer)
(2 Points)
streptococcal tonsillitis (there would be no maculopapular rash after prescription of ampicillin).
Hodgkin’s lymphoma
herpangina
infectiousmononucleosis (because the maculo-papular rash developed in the body after the
prescription of ampicillin).

51. An 18-month-old, previously healthy, child develops diffuse petechial rash all over the body
and bleeding from the gums 10 days after vaccination (measles, mumps, rubella). The rest of
lOMoAR cPSD| 41174528

T
Downloaded byKato Caleb (katocalebnatamba@gmail.com)
lOMoAR cPSD| 41174528

it

the physical examination is normal. Blood tests shows values of hemoglobin, RBC and WBC, but
low level of PLT - 12 x10 / 9 / l. What is the right management in this case?
(From 1 to 5 correct answers out of 5 possible answers; 1 point for each correct answer, minus 1
point for each wrong answers)
Immediatecorticosteroidtreatment
Urgent platelet transfusion (only in severe case as they raise the platelet count only for few hrs).
Rituximab therapy
Treatment with corticosteroid, but after a bone marrow biopsy
Treatment with high-dose immunoglobulin

Mucocut bleeding e.g gum bleeding - Start Tx in ITP -- 1st CS ---then if it fails IVIGs ---Rituximab
---splenectomy

52. A 3-year old girl is brought to the emergency department because of generalised itchy rash
on the face and body and dry cough after eating peanut butter. On examination she is
hemodynamically stable, with generalized urticarial rash, vesicular breathing with prolonged
expiration and wheeze bilaterally. The rest of her examination is normal.
What tests would you perform to reach a diagnosis?
(From 1 to 5 correct answers out of 5 possible answers; 1 point for each correct answer, minus 1
point for each wrong answers)
lung function tests
PECKER o specific IgE
skin-prick test for nut allergies
bronchoscopy
total IgE
O
53. This 2-year-old boy lives in Africa on a maize-growing farm. Which of the following is true:
(From 1 to 5 correct answers out of 5 possible answers; 1 point for each correct answer, minus 1
point for each wrong answers)
his long-term intellectual development is not at risk - physical and intellectual disabilities in
severe cases
he has severe protein deficiency but near normal calorie intake
he is prone to protracted and severe infections (due to working in a farm & protein deficiency)
the major macronutrient lacking in his diet is fat
he is likely to have increased appetite - kwashiorkor = low appetite

54. A 10-month-old infant, born at 38 gestational weeks with weight 2400 g and length 48 cm.
For about 2 months he has had frequent vomiting, no diarrhea and is mainly breastfed. With a
weight gain of 300 gr for the last 2 months. On examination the child has reduced subcutaneous
fat tissue, weight 7000 g, height 71 cm and head circumference 44 cm. Normal lung and heart
status, no hepatosplenomegaly. Laboratory tests - Hb -90g / l; RBC - 3.8x 10/12; WBC - 13x 10/9;
PLT - 210x 10/9; ESR- 40mm. Urine: Protein +; sediment-7-8 erythrocytes, 30 leukocytes, 5-6
epithelial cells/ per field. What are your recommendations?

Downloaded by Kato Caleb (katocalebnatamba@gmail.com)


lOMoAR cPSD| 41174528

(From 1 to 5 correct answers out of 5 possible answers; 1 point for each correct answer, minus 1
point for each wrong answers)
Urine culture (due to proteinuria; hematuria& pyuria)
Ultrasound of the abdomen (to rule out urinary malformations etc)
Consultation with an endocrinologist
Examination of trypsin in faeces
Conducting a sweat test

55. What are the main clinical presentations of T-cell non-Hodgkin lymphomas in children?
(From x 5 correct answers out of 5 possible answers; 1 point for each correct answer, minus 1
point for each wrong answers)
Cervical lymphadenomegaly (typical for B cell non-Hodgkin’s lymphoma)
Abdominal mass (typical for B cell non-Hodgkin’s lymphoma)
Bonemarrowinfiltration
Bone marrow aplasia
Mediastinaltumormass

56. A 6 months old child from normal pregnancy and delivery, with birth weight 3400g. was
admitted to hospital with complaints of persistent cough and recurrent pulmonary infections.
On examination he was found in unsatisfactory general condition, with body weight 5300 g,
pale skin and mucouses. RR - 50 / min, emphysematous chest, moderate dyspnea with
wheezing of the lungs and crepitations bilaterally. HR-140/min, abdomen - soft, liver- 3 cm. The
mother reported that the child had good appetite, but frequent voluminous, greasy stools.
What investigations would you order to confirm the diagnosis?
(From 1 to 5 correct answers out of 5 possible answers; 1 point for each correct answer, minus 1
point for each wrong answers)
Echocardiography
Anti-tTGantibodies
Trypsininstools (because of greasy stools => to check for pancreatic function)
Faecal calprotectin
Sweattest ( to rule out CF as symptoms are suggestive of CF).

57. A 10-year-old girl is consulted because she became very distracted at school. Her mother
also noticed that she has episodes when she stops her activity for a few seconds, stares and
then resumes the activity as if she had never stopped. What you would recommend?
(From 1 to 5 correct answers out of 5 possible answers; 1 point for each correct answer, minus 1
point for each wrong answers)
Send her to a child psychiatrist
PerformanEEG
Perform a brain CT
Start behavioural therapy for ADHD
Performahyperventilationprovocativetest

Downloaded by Kato Caleb (katocalebnatamba@gmail.com)


lOMoAR cPSD| 41174528

58. A previously well 1-year-old infant has had a runny nose and has been sneezing and
coughing for 2 days. Two other members of the family had similar symptoms. Four hours ago,
his cough became much worse. On physical examination, he is in moderate respiratory distress
with nasal flaring, hyperexpansion of the chest, and easily audible wheezing without rales.
Which of the following is the appropriate next course of action?
(From 1 to 5 correct answers out of 5 possible answers; 1 point for each correct answer, minus 1
point for each wrong answers)
Continuous positive airway pressure (CPAP)
IV Antibiotics
Short-acting bronchodilators
Monitoring oxygenation and fluid status
Bronchoscopy

59. A 4-month-old infant is referred to the Gastroenterology Department due to chronic


constipation since birth. He was born at home and has not been seen by a doctor so far. He is
breastfed. According to the mother his birth weight was around 3000 gr.

On examination his current weight is 4.2 kg, there is significant abdominal distension, severe
developmental delay. Abdominal distension is mostly due to a massively air-filled bowel,
although there is some palpable mass of stools. What additional laboratory tests and
procedures will you consider to establish the diagnosis:
(From 1 to 5 correct answers out of 5 possible answers; 1 point for each correct answer, minus 1
point for each wrong answers)
coeliac disease screening
suctionrectalbiopsy - hirschsprung (but does not account for developmental delay + most air
filled)
electrolytes in blood
thyroidfunctiontests (due to developmental delay?) - congenital hypothyroidism - chronic
constipation, distended abdomen, developmental delay, poor growth
stool culture

60. After a difficult vaginal delivery of a 4200 gr boy you notice that the child does not move his
right upper extremity. There is swelling around the shoulder and the passive movements in the
shoulder are painful. On palpation you find the crepitus of the clavicle. Moro reflex is absent on
the right, grasping reflex is symmetrical, biceps reflex is absent on the right. The rest of the
physical examination showed no abnormalities.

A.What is the most likely diagnosis?


(1 correct answer)
(2 Points)
Pseudoparalisis of Parrot
Rightupperbrachialplexuspalsy(Erb’spalsy)withclaviclefracture (because the Moro reflex &
biceps reflex is absent )
Distortion of the right shoulder

Downloaded by Kato Caleb (katocalebnatamba@gmail.com)


lOMoAR cPSD| 41174528

Osteomyelitis of the right humerus

61B. Which laboratory and imaging studies you would recommend?


(1 correct answer)
(2 Points)
Congenital infections screening
EMG–nerveconductionvelocitytest
CT of the right shoulder
CT of the brain

62C. Which of the following you would recommend?


(1 correct answer)
(2 Points)
Surgical draining of osteomyelitis
Electrophoresis with Nivalin (Glantamine) after 2 weeks of age
Surgicalfixationoftheclaviclefacture
Antibiotics

63 A 16-year-old girl with a known small ventricular septal defect (VSD) had a tattoo done 10
days ago. For the last 6 days she has been having fever up to 39ºC with chills, malaise and
painful and swollen knees, ankles and wrist joints. On examination she has fever 38.1ºC, RR
18/min, HR 110/min. The fore-mentioned joints are oedematous, with increased local
temperature and reduced range of movement. Her initial lab results are: Hgb 88 g/l, MCV 85 fl,
WBC 24x109/L, Plt 290x109/L,ESR 60mm/h, urine – protein 1+, RBC 40/hpf. Echocardiography
confirms small VSD.

A.Which of the following investigations will be most useful in guiding her management:
(1 correct answer)
(2 Points)
severalbloodcultures- children with congenital heart disease - ͫrisk for infective endocarditis
esp w/Hx of tattoo + Sx of inflammation + IE
urine culture
serology for hepatitis B
rheumatoid factor

64After admission her condition deteriorates with RR 30/min, crepitations at the lung bases, HR
130/min, and a new soft murmur is heard at the apex with radiation to the axilla. Liver is
palpted at 4 cm below the costal margin, urine output is reduced. Thin red-brown lines appear
under her nails.

B. What is the most likely diagnosis:


(1 correct answer)
(2 Points)
hepatitis B (no jaundice or no info on LFT).

Downloaded by Kato Caleb (katocalebnatamba@gmail.com)


lOMoAR cPSD| 41174528

heart failure due to VSD


bacterialendocarditis (due to fever, chills => signs of infection)
polyarticular juvenile idiopathic arthritis

65C. Her treatment should be:


(1 correct answer)
(2 Points)
high dose aspirin
r
4-8 weeks of antibiotics, diuretics and ACE-inhibitor
high dose corticosteroids
high dose immunoglobulins

66A 4-year-old girl is admitted to hospital due to vomiting and watery stools for 2 days. On
examination she is drowsy, has sunken eyes, reduced turgor and elasticity of the skin, acetone
breath, HR 140/min, RR 30/min, BP 80/40 mmHg. Abdomen is soft, normal bowel sounds.

A. Which urgent tests will you request:


(1 correct answer)
(2 Points)
lumbar puncture to rule out meningitis
stool culture
bloodforcompletebloodcount,electrolytes,urea,cretinine,uricacid,bloodglucose,CRP
abdominal ultrasound to rule out intussusception

67B. What emergency therapy would you start while waiting for the test results?
(1 correct answer)
(2 Points)
I.v metoclopramide, followed by oral rehydration
intravenousrehydration
I.v. Dexametasone and 10% Mannitol to reduce cerebral edema
insulin infusion

68. One hour later the child is still in the same condition, her urine output only about 10 ml of
urine. Urine results - specific gravity 1038, ketones 2+, glucose- neg, protein 1+, sediment -
normal.

C. What is the most probable diagnosis?


(1 correct answer)Immersive Reader
(2 Points)
Fasting hypoglycaemia
Diabeticketoacidosis
Prerenal acute kidney injury in a child with gastroenteritis
Neuroinfection

Downloaded by Kato Caleb (katocalebnatamba@gmail.com)


lOMoAR cPSD| 41174528

Thursday

1. Which age period in childhood is termed “pre-school”?


(1 correct answer)
(1 Point)
From2to5years (3-6 years)
From 1 to 2 years
From birth to 3 years
From birth to 5 years

2. The dramatic reduction in child mortality in the developed world in the last decades is
due to all EXCEPT?
(1 correct answer)
(1 Point)
reductioninbirthrate (only one that makes sense) antibiotics and
treatment of severe bacterial infections immunizations and
reductions of deaths due to infectious causes
Mainly because of improved living conditions and health knowledge of the population

3. If a breastfed infant is weaned after 6 months of age there is risk of:


(1 correct answer)
(1 Point)
Aplastic anemia
Salt overload
Irondeficiencyanemia- after 6 mo = risk of malnutrition
Folate deficiency anemia - folic acid present in breastmilk

4. The most common cause for weight faltering is:


(1 correct answer)
(1 Point)
infections
endocrine disorders
inadequatefoodintake (inadequate calorie intake => 90% of the cases)
congenital malformations

5. Hemorrhagic disease of the newborn is:


(1 correct answer)
(1 Point)
Coagulopathy (-ͭVit K dependent Coagulation factors - 2,7,9,10 - Hemophilia; von Willebrand
disease; Liver disease; Vitamin K deficiency; Calcium deficiency & use of anticoagulants).
Vasculopathy (HSP; Vitamin C deficiency & Cushing’s disease)
DIC (disseminated intravascular coagulation) syndrome
Thrombocytopenia (together with impaired platelet function=> Thrombopathy)

Downloaded by Kato Caleb (katocalebnatamba@gmail.com)


lOMoAR cPSD| 41174528

6. A 2-day-old newborn with respiratory rate 24/min and heart rate 140/min has:
(1 correct answer)
(1 Point)
Normalheartrateandbradypnea (Normal respiratory rate for newborn => 30-50; pulse=>
110-160).
Normal respiratory and heart rate
Bradycardia and bradypnea
Tachycardia and bradypnea

7. Which of the following statements regarding hemolytic disease of the newborn is true:
(1 correct answer)
(1 Point)
jaundice appears after the 7th day (Occurs within 24 hrs)
it is due to antibodies against fetal erythrocytes because of incompatibility between maternal
and fetal erythrocytic antigens
kernicterus is due to the deposition of direct bilirubin in brain nuclei (Indirect bilirubin)
there is iron deficiency anemia - normocytic

8. Which of the following does NOT cause neonatal hepatitis:


(1 correct answer)
(1 Point)
Congenitallisteriosis - mainly respiratory distress + skin lesions --can cause meningitis too
Herpes simplex virus
Cytomegalovirus
Chlamidia trachomatis

9. During the first year of life the height (length) increases with:
(1 correct answer)
(1 Point)
around 50 cm
around 12 cm
around 6 cm
around25cm (height at 1 year would be 74cm. Height/length at birth is usually 50cm. (74-50=
24cm. Therefore, ans should be this).

10. Which of the following antidotes must be applied in paracetamol (acetaminophen)


poisoning?
(1 correct answer)
(1 Point)
Atropine (for organophosphorus pesticides poisoning)
Ethanol - methanol + ethylene glycol poisoning
Sodium bicarbonate (treatment for metabolic acidosis)
N-acetylcysteine

Downloaded by Kato Caleb (katocalebnatamba@gmail.com)


lOMoAR cPSD| 41174528

11. Bile-stained vomitting is NOT characteristic for:


(1 correct answer)
(1 Point)
intestinal malrotation with volvulus
pyloricstenosis (non-bilious vomiting)
duodenal atresia
meconium ileus

12. Wilson disease may present with:


(1 correct answer)
(1 Point)
seizures
heart failure
Vit. B12 deficiency anemia
hepaticpresentationandneuropsychiatricfeatures - affects mainly kidneys, liver, eyes, brain...

13. The most common cause for arterial hypertension in YOUNG children is:
(1 correct answer)
(1 Point)
brain tumors
hypertension secondary to chronic kidney diseases
metabolic syndrome
essentialhypertension (a.k.a primary hypertension/ idiopathic hypertension => no identifiable
secondary cause).- most common in obese teens.

14. Тhe first sign of puberty in boys is:


(1 correct answer)
(1 Point)
Pubarche
Growth spurt
Voice mutation
Enlargementofthetestes - > 4 ml

15. The characteristic physical findings in lobar pneumonia are:


(1 correct answer)
(1 Point)
Dull percussion tone, vesicular breath sounds with localized crackles, positive bronchophony
Dullpercussiontone,bronchialbreathing,positivebronchophony
Hyperresonant percussion tone, reduced breath sounds with prolonged expiration , positive
bronchophony
Hyperresonant percussion tone, bronchial breathing, positive bronchophony

16. What is true for cyanosis due to respiratory causes:

Downloaded by Kato Caleb (katocalebnatamba@gmail.com)


lOMoAR cPSD| 41174528

(1 correct answer)
(1 Point)
improveswithoxygentherapy
is worse when the child is anemic
improves if the child squats down
it worsens during crying

17. The most common type of arrhythmia in children is:


(1 correct answer)
(1 Point)
supraventriculartachycardia
1st degree A-V block
ventricular tachycardia
2nd degree A-V block

18. What is NOT true for cephalhematoma in neonates?


(1 correct answer)
(1 Point)
Itextendsbeyondthemarginsoftheskullbone (does not cross the suture lines) - caput succ =
cross suture lines
It is confines within the margins of the skull sutures
It usually involves the parietal bone
It is a bleeding below the periosteum -- periosteum is firmly attached to sutures but loosely to
the actual bone -- hematoma = confined

19. What is the first-line therapy in asthma exacerbation:


(1 correct answer)
(1 Point)
Oxygen therapy
I.v Novphyllin
InhaledSalbutamol Reliever
I.v corticosteroids

20. The main characteristics of nephrotic syndrome are:


(1 correct answer)
(1 Point)
Proteinuria, hematuria, leucocyturia, edema
Proteinuria,hypoproteinemia,hyperlipidemia,edema
Proteinuria, hematuria, hyperlipidemia, edema
Proteinuria, leucocyturia, edema

21. In chronic granulomatous disease there is:


(1 correct answer)
(1 Point)

Downloaded by Kato Caleb (katocalebnatamba@gmail.com)


lOMoAR cPSD| 41174528

T-cell immune deficiency


Abnormalphagocytosis (neutrophils + maf fail to produce superoxide after phagocytosis of
microorganisms).
B-cell immune deficiency
Absolute neutropenia

22. Which of the following is NOT a congenital heart defect with left to right shunt:
(1 correct answer)
(1 Point)
Atrial septal defect
TetralogyofFallot (right to left shunt)
Persistent ductus arteriosus
Ventricular septal defect

23. When unrecognized, which of the following congenital heart defects causes cardiogenic
shock in the early neonatal period:
(1 correct answer)
(1 Point)
Tricuspid insufficiency
Tetralogy of Fallot
Atrial septal defect
Infantiletypecoarctationoftheaorta (Poor feeding; respiratory distress & shock may develop
before 2 weeks of age).

24. Which is not typical for spinal muscular atrophy type 1? (1 correct answer)
(1 Point)
Tendonhyperreflexiaandpathologicalreflexes (deep tendon reflex is absent) - SMA = LMN --
hyporeflexia + no pathological reflex
normal intellectual development
fascicultions of the tongue (present) - LMN lesion
Muscle weakness (weakness of bulbar muscles)

25. Which is the most severe consequence of hypoxic-ischemic encephalopathy?


(1 correct answer)
(1 Point)
Brain calcifications
Porencephalic cyst
Multicysticencephalomalacia
Intraventricular hemorrhage grade I

26. Which is NOT a cause of constipation in the neonatal period:


(1 correct answer)
(1 Point)
Hirschpung’s disease

Downloaded by Kato Caleb (katocalebnatamba@gmail.com)


lOMoAR cPSD| 41174528

Abdominal groans
Congenital hypothyroidism
Lactoseintolerance
Hypocalcemia (Hypercalcemia causes constipation)1
27. Motor dysfunction in cerebral palsy is due to:
(1 correct answer)
(1 Point)
Progressive damage to the brain
Non-progressivedamagetothebrain (cerebral palsy occurs as a result of PERMANENT but
non-progressive brain lesion).
Damage to the limb muscles
Damage to the spinal cord

28. Which of the following disorders does NOT present with coagulopathy?
(1 correct answer)
(1 Point)
Fanconi’sanemia (type of aplastic anemia)
Liver cirrhosis
Hemophilia
Hemorrhagic disease of the newborn

29. What is characteristic for Henoch-Schonlein purpura:


(1 correct answer)
(1 Point)
Prolonged bleeding time (coagulation profile is normal in HSP)
Normalclottingandbleedingtime
Thromocytopenia (platelet is normal or elevated)
Prolonged clotting time

30. Iron deficiency anemia is:


(1 correct answer)
(1 Point)
Hypochromic macrocytic
Hypochromicmicrocytic
Microspherocytic
Normochromic normocytic

31. What is NOT present in hemolytic anemia:


(1 correct answer)
(1 Point)
increased LDH (lactate dehydrogenase)
increased serum iron
directhyperbilirubinemia (correct ans is indirect hyperbilirubinemia)
Reticulocytosis

Downloaded by Kato Caleb (katocalebnatamba@gmail.com)


lOMoAR cPSD| 41174528

32. Newborn with congenital hypothyroidism has:


(1 correct answer)
(1 Point)
Tachycardia (hypothyroidism => bradycardia)
Ossified bones in the left wrist
Diarrhea (constipation)
Prolongedjaundice - ͫUCB

33. What is characteristic for diabetes mellitus type II:


(1 correct answer)
(1 Point)
Tendency for diabetic ketoacidosis (true for type I diabetes)
Monogenic inheritance (polygenic)
Iscommonlyassociatedwithobesity
Starts in early childhood (type I diabetes)

34. Which newborns are at risk of necrotizing enterocolitis:


(1 correct answer)
(1 Point)
With congenital infection
Severelyprematureneonates (common in children with low birth-weight)
Neonates born to mother with poorly controlled diabetes mellitus
Children with congenital abnormalities of the GIT

35. A-4-month-old infant from normal pregnancy and delivery, with normal development so far.
During a routine check-up the family doctor established tachypnea at 50/min. Mother confirms
that it has been breathing like that since discharge from the neonatal unit. On examination
there is also hyperresonant percussion tone with reduced breath sounds in the left suclavicular
area. The infant is afebrile, without any cough. The heart sounds are loudest slightly to the right
of where they are normally heard best. Which of the following diagnosis is most likely?
(1 correct answer)
(2 Points)
right upper lobe aplasia
right-sided pneumothorax (the breath sound will be reduced on the right side)
congenitallobaremphysemaoftheleftupperlobe
Congenital pneumonia in the left upper lobe (no cough or signs of infection)

36. A three-month old infant is admitted because of pneumonia. The patient was treated for
two weeks at home without improvement. On the 3rd day after the birth of the infant a heart
murmur was detected and parents were advised to consult a pediatric cardiologist after
discharge from the neonatal unit, but so far they have not done it. On examination the child is
pink but with tachydyspnea and bilateral crepitations. There is also a loud P2; soft pansytolic
murmur, loudest at the lower left sternal border. Liver is palpable 3 cm below the costal margin.

Downloaded by Kato Caleb (katocalebnatamba@gmail.com)


lOMoAR cPSD| 41174528

What are the expected findings from the following tests?


(1 correct answer)
(2 Points)
chestX-ray–cardiomegalywithincreasedvascularmarkings (thinking of VSD here because the
sound is heard in lower left sternal border) + pansystolic murmur, w/crepitations + resp
infection
ECG- partial right bundle branch block and right ventricular hypertrophy
chest X-ray - boot-shaped heart with reduced vascular markings
echocardiography – pulmonary hypertension with R-L shunt

37. A 1-month-old infant is brought to the family doctor because of persistent jaundice. The
pregnancy and delivery were uneventful. The infant is breastfed and has gained 1 kg since birth.
On examination it looks well, has good muscle tone, mild jaundice, pink mucous membranes.
No hepatosplenomegaly, normal voice. Mother confirms that the urine is transparent and the
stools are bright yellow. What is the most appropriate management?
(1 correct answer)
(2 Points)
Hereditary spherocytosis should be ruled out - pink mucus memb + mild jaundice etc
reassuremotherasthisismostlikelybreastmilkjaundice - mild jaundice w/o Sx of anemia,
normal stool + urine
The infant has the mildest form of haemolytic disease of the newborn- haemolytic anemia
X-ray of left knee for bone age

38. A previously healthy 3-year-old child wakes up at night with high fever of 39.5, hypo-to
aphonia, drooling due to inability to swallow the saliva, inspiratory stridor The child looks
severely ill and is sitting up and leaning forwards. What is your diagnosis?
(1 correct answer)
(2 Points)
Foreign body aspiration
Acuteepiglottitis (child is unable to swallow=> drools down the chin & sits forward)
Asthma exacerbation
Pseudo-croup (viral laryngotracheobronchitis) - stridor but w/o dysphagia, high fever, drooling

39. A 10-year-old girl has been complaining of severe abdominal pain for the last couple of days.
Since this morning the pain has increased and the child started vomiting. She was examined by
a pediatric gastroenterologist, who performed an abdominal ultrasound and found infiltration
of the terminal ileum. Lab results- mild normocytic anemia and thrombocytopenia. LDH – 2000
E/L; chest X-rays reveals a group of enlarged lymph nodes next to the trachea. What is the most
likely diagnosis?
(1 correct answer)
(2 Points)
Crohn’s disease
Non-Hodgkinlymphoma - GI infiltration, anemia, thrombocytopenia, mediastinal LAD ...
Phlegmonous appendicitis

Downloaded by Kato Caleb (katocalebnatamba@gmail.com)


lOMoAR cPSD| 41174528

Fecaloma

40. An 18-month-old child had tests because of significant pallor. The results show Hgb - 87 g/l,
RBC – 3.5 х10/12/l , MCV – 100 fl , MCH – 34 pg, WBC– 4.8 х10/9/l , platelets – 162 х10/9/l ;
LDH – 584 E/l. At 6 months of age the child was diagnosed with cow’s milk allergy, so for the last
10 year it was fed mainly goat’s milk. Which is the most probable diagnosis?
(1 correct answer)
slightly (2 Points)
high Iron deficiency anemia (MCV & MCH would be low)
Fanconi’s anemia (there would be thrombocytopenia but here we have thrombocytosis)
Folatedeficiencyanemia (MCV & MCH is slightly increased)
Leukemia

41. A 14-month-old child was born by emergency cesarean section at 36 gestational weeks
because of abruption of the placenta with birth weight 2400 gr, length – 48сm, head
circumference – 33см, with mild asphyxia. He started smiling, cooing and following object on
time. He was turning from back to front at 6 months, and sitting at 9 months of age. He was
standing independently and cruising around furniture at 1 year. He is still not walking
independently and walks on tiptoes when held by two hands with some crossing over of both
legs (scissoring). The way he plays with bricks is adequate for his age. He waves “bye-bye” and
blows kisses. Says 3 words. On examination there is increased muscle tone of the thigh
adductors and difficult dorsiflexion of feet. Tendon hyperreflexia for lower limbs with clonuses
of both feet and positive Babinski sign bilaterally. The rest of the examination is normal. What is
the most likely diagnosis?
(1 correct answer)
(2 Points)
Cerebralpalsy–spasticdiplegicform (scissoring & increased muscle tone => cerebral palsy. Only
the lower limbs are affected so diplegic type).
Global neurodevelopmental delay
Cerebral palsy- spastic quadriplegic form
Congenital muscular dystrophy

42. A previously healthy 7-year-old boy with normal intellectual development. Two hours after
falling asleep he sits up in bed, unable to talk, with facial asymetry and tremor of the right oral
angle for 3 min, followed by saliva drooling. After that he goes back to sleep. In the morning he
remembers what happened, says he could hear people talking during the episode but could not
answer back and had a choking feeling. He has normal somatic and neurological examination.
EEG shows- centro-temporal spikes on the left. Which is the most likely diagnosis?
(1 correct answer)
(2 Points)
Pseudo-croup
Laryngeal spasm due to hypoparathyroidism
Benignrolandicepilepsy
Hysteria
I common childhoodepilepsy

Downloaded by Kato Caleb (katocalebnatamba@gmail.com)


lOMoAR cPSD| 41174528

43. A 3-year-old girl from the first normal pregnancy and term delivery. She has been having
bulky, foul smelling stool once to twice /day since she turned 1 year of age. She is quite
apathetic and a fussy eater. On examination she has reduced subcutaneous fat, wasted
buttocks, distended abdomen, no hepatosplenomegaly. Normal respiratory and cardio-vascular
examination. Her results are: Hb- 95 g / l, MCV 60 fl (75-87), serum iron 1.7 μmol / l (10.5 - 23
μmol / l)AST -35 IU/L(0-60); ALT 40 IU/L(0-45); Creatinine 60 μmol/ (35-75), urea-5.3 mmol/ll
(2.6-7.2);Urine- protein-negative; sediment-3-4 leukocytes;
Tissue transglutaminase antibodies - 240 IU/l(0-15)
What is the most likely diagnosis?
(1 correct answer)
(2 Points)
Crohn's disease
Chronic anemia
Chronic kidney disease
Celiacdisease (due to anti-TTG antibodies)

44. A 7-year old boy with Duchenne Muscular Dystrophy, who has not been properly followed
up, develops dyspnea, orthopnea, tachycardia and hepatomegaly. What is the most likely cause:
(1 correct answer)
(2 Points)
dilatedcardiomyopathy (common cause of death in DMD) -dyspnea, orthopnea, tachycardia,
hepatomegaly (HF)
myocarditis bronchial
asthma bacterial
endocarditis

45. A 14-year-old girl had high fever, sore throat and enlarged submandibular lymph nodes for 3
days. On examination there was evidence of purulent tonsillitis, so she was prescribed
Augmentin (Amoxicillin + clavulonic acid). Next day her fever persisted and she developed
maculo-papular rash on the body. This time examination revealed hepatosplenomegaly, mild
edema of upper eyelids and the rash. Lab results – Hgb 125g/l; increased WBC 15х 10/9 with
lymphocytosis – 70% -1/3 of them similar to monocytes, neutrophils15%, Plt– 180х 10/9; ESR –
25mm, АSАТ – 150 U/l, АLАТ – 200 U/l, LDH – 600U/l. What is the most likely diagnosis?
(1 correct answer)
(2 Points)
Acute agranulocytosis
Acute streptococcal tonsillitis with allergic rash
Infectiousmononucleosis (if rash development after amoxicillin/ ampicillin treatment =>
infectious mononucleosis) + purulent tonsillitis, LAD, HSM, edema
Acute lymphoblastic leukemia

46. A 10-year old boy started treatment with Trimethoprim-sulfamethoxazol (Biseptol) because
of acute sinusitis. On the second day he had abdominal pain, acute palor and jaundice, fatigue,

Downloaded by Kato Caleb (katocalebnatamba@gmail.com)


lOMoAR cPSD| 41174528

and his urine became darker. No other symptoms. On examination he is pale and jaundiced,
has vesicular breath sounds, tachycardia to 110/min, rhythmic heart sounds, spleen -0.5 cm
below the costal margin. Succusio renalis – negative. Lab results - Hgb 70g/l; RBC- 3.2 x10/12/l,
WBC – 8.8 x10/9/l, PLT- 348 x10/9/l, АSAT – 50U; АLAT- 40U; bilirubin – 87μmol/l; direct
bilirubin – 10 μmol/l, LDH – 2000U. Coombs ˋtest – negative. Family history – uncle with a few
episode of acute anemia from early childhood. What is the most likely diagnosis?
(1 correct answer)
(2 Points)
Acute viral hepatitis (Normal ASAT & ALAT so maybe not hepatitis)
Hemolyticanemiaduetoglucose-6-phosphatedehydrogenasedeficiency(G-6-PD) {G6PD is
usually triggered by infection; certain drugs; fava beans} - triggered here by the sulfonamide
Acute aplastic anemia (platelet is increased, if it was aplastic anemia, platelet would be
decreased too)
Auto-immune hemolytic anemia (Coomb’s test is (-ve))

47. Kallmann's syndrome is associated with?


(1 correct answer)
(2 Points)
Ambiguous genitalia - CAH
Hypogonadotropichypogonadism - gonadotropin prob - isolated FSH/LH def -- ͭ
testosterone/estradiol -- delayed puberty (central type)
Isosexual precocious puberty
Hypergonadotropic hypogonadism klien felter, turner - peripheral type of delayed puberty

48. A 5-month -old infant from first normal pregnancy and delivery at term with birth weight
3300 gr and length 51 cm . Breast fed. Weaned at 4 months with cereal and fruit puree. Has 4
previous hospital admissions because of obstructive bronchitis and was treated with inhaled
Salbutamol and parenteral corticosteroids. Afterwards at home usually takes oral steroid for a
few days after each hospitalization. Examined by the family doctor because of significant
weight gain. Currently weighs 8 kg- above the 97th centile, length 58 cm-below the 3rd centile.
Blood pressure 110/70- above 95th centile. Normal respiratory examination. No
hepatosplenomegaly. Hypertrichosis. Dilated blood vessels on the cheeks. Normal
neurodevelopment. hirsutism
What is the cause of obesity and short stature?
(1 correct answer)
(2 Points)

Metabolic syndrome
Obesity because of overnutrition
Cushingsyndrome (due to persistent use of corticosteroids) - hypertrichosis, overweight...
Probably familial

Downloaded by Kato Caleb (katocalebnatamba@gmail.com)


lOMoAR cPSD| 41174528

49. An 8-year- old child is brought to the Emergency department with complaints of repeated
vomiting, profuse diarrhea and abdominal pain that started 1 hour after consumption of wild
mushrooms. Choose the right statement:
(1 correct answer)
(2 Points)
Plasmapheresis must be performed within the first 24 hours
We should expect development of severe liver damage
This is Amanita phalloides poisoning, start immediate treatment with Silibinin .
This is probably irritative mushroom poisoning, start treatment with rehydration and activated
charcoal

50. A 6-year-old boy had purulent tonsillitis 10 days ago. Now he presents with macroscopic
hematuria and periorbital edema. What is the right management?
(From 1 to 5 correct answers out of 5 possible answers; 1 point for each correct answer, minus 1
point for each wrong answers)
Start treatment with corticosteroids
Test for urea, creatinine, uric acid, C3 and C4 complement fractions, electrolytes and
antistreptolysin antibodies
Kidney biopsy
Monitoringofbloodpressureandurineoutput
Start treatment with albumin infusions
PI Gr
51. А 2-year-old girl is admitted to hospital because of pneumonia. She was treated by her
family doctor with an antibiotic at home but with no effect. She was born on time with birth
weight 3100 gr and length 51 cm. Uneventful perinatal period. She has been suffering from
frequent chest infections. She has good appetite , but frequent bulky, fatty stools. On
examination she looks unwell, has non-productive cough, tachypnoea at 50/min,
emphysematous chest, wheeze and crepitations bilaterally. Cardio-vascular system- no
abnormalities detected. Abdomen- soft, slightly distended, no palpable organomegaly. Current
weight 10.5 kg, height 80 cm ( both are < 3rd centile). What tests would be most useful in
making the diagnosis?
(From 1 to 5 correct answers out of 5 possible answers; 1 point for each correct answer, minus 1
point for each wrong answers)
fecal calprotectin
X-ray of sinuses
anti-tTG antibodies
chest X-ray
Sweat test

52. A 1-year-old child is consulted because of malaise, episodes of restlessness, crying,


sweating and tachycardia to 200/min and blood pressure 150/90. Abdominal ultrasound
showed a tumor, originating from the left adrenal gland. There is anemia. Urine is normal.
Which of the following tumor markers are characteristic for the suspected malignancy?

Downloaded by Kato Caleb (katocalebnatamba@gmail.com)


lOMoAR cPSD| 41174528

(From 1 to 5 correct answers out of 5 possible answers; 1 point for each correct answer, minus 1
point for each wrong answers)
Neuron-specificenolase (tumor marker for neuroblastoma & SCLC)
Carcinoembryonic antigen (tumor marker for colon carcinoma)
Vanillylmandelicacidinurine (tumor marker for neuroblastoma)
Alpha fetoprotein (tumor marker for liver; ovary & testicular cancer)
Chorion gonadotropin (tumor marker for testicular carcinoma)

53. An 8-year-old girl with normal growth and development. For the last 3 years during spring
time, has complained of frequent sneezing and itchy nose and eyes. Parents always gave her
antihistamines with good effect. During the last year she has had these symptoms when the
weather is wet and foggy, especially in the morning. Sometimes she also has cough and wheeze.
What tests would you recommend to make the diagnosis?
(From 1 to 5 correct answers out of 5 possible answers; 1 point for each correct answer, minus 1
point for each wrong answers)
SerumIgE
Echocardiography
Lung function tests with bronchodilator test (to rule out Asthma which is triggered by cold
weather).
Nose swab for culture
Sweat test

54. A previously healthy 9- year-old boy has been having low grade fever, fatigue, reduced
appetite and dry cough for the last 2 months. Recently the cough became productive with blood
stained sputum occasionally. The grandfather, who lives with the family, has a chronic cough. On
examination the child has normal respiratory status. Chest X-ray shows opacification of the right
middle lobe. FBC and DCC are within reference range. ESR 42 mm.
Which further tests would you recommend?
(From 1 to 5 correct answers out of 5 possible answers; 1 point for each correct answer, minus 1
point for each wrong answers)
skin-prick tests for allergies
interferon-gammareleaseassay(IGRA)fortuberculosis
bronchodilator test
bronchoscopy to rule out foreign body in the airways
microscopyofsputumtolookforacid-fastbacilli

55. A 2-day-old neonate, born to mother with diabetes mellitus with birth weight 4000 gr and
length 53 cm, has generalized tonic seizures. Which of the following medications would you use
to treat the seizures?
(From 1 to 5 correct answers out of 5 possible answers; 1 point for each correct answer, minus 1
point for each wrong answers)
Calciumgluconate10%
Sodium bicarbonate 10%
Vitamin C

Downloaded by Kato Caleb (katocalebnatamba@gmail.com)


lOMoAR cPSD| 41174528

Glucose25%
Potassium chloride 15%

56. A 4-year-old boy, with normal development and no past medical history of note, is admitted
to hospital because of profuse petechial rash, mainly on the face, neck and torso that occured
the previous day. Physical examination does not reveal any other abnormalities. Blood tests on
admission – normal levels of hemoglobin, WBC, RBC; PLT - 10x10/9 / l, LDH – normal value.
What is the right management?
(From 1 to 5 correct answers out of 5 possible answers; 1 point for each correct answer, minus 1
point for each wrong answers)
Monitoringonly,asthepatientisnotcurrentlybleedingprofusely
Splenectomy
Immediate platelet transfusion
I.v antibiotics
Treatment with corticosteroids or high-dose immunoglobulin after confirmation of the diagnosis
by bone marrow biopsy

57. A 6-month-old infant falls ill for the first time with fever and refusal to feed. There is no
t vomiting and diarrhea. Normal physical examination. Laboratory results show leucocytosis with
neutrophilia, raised ESR. Which of the following test would you request next to clarify the
diagnosis?
(From 1 to 5 correct answers out of 5 possible answers; 1 point for each correct answer, minus 1
point for each wrong answers)
Tests for immune deficiency
Urine test - biochemistry, microscopy and culture
Abdominal ultrasound - check the urinary system as its a 1st UT?
Stool culture
Pharyngeal aspirate for virology studies

58. The girl in the photo is 7 years old from the first normal pregnancy, born at term with weight
2500 gr - on the 3rd percentile and lenght 48 cm - on the 3rd percentile. She was operated for
coarctation of the aorta in the neonatal period. Her heart condition has been monitored by a
pediatric cardiologist – she has normal heart function. She has had recurrent upper respiratory
tract infections and acute otitis. She has some learning difficulties at school. She is referred
because she is the shortest in her class.
What tests will be most helpful for the diagnosis?
(From 1 to 5 correct answers out of 5 possible answers; 1 point for each correct answer, minus 1
point for each wrong answers)
Echocardiography
Growth hormone
Cytogenetic test - Coarctation of aorta is associated w/Turners (DS = AVSD)
Thyroid hormones
Magnetic resonance imaging of the central nervous system

Downloaded by Kato Caleb (katocalebnatamba@gmail.com)


lOMoAR cPSD| 41174528

59A 4-year-old child with normal growth. Fully immunized. Does not suffer from frequent
infections. On the day of the admission he was playing unsupervised when his parents heard
him coughing violently. He also had a hoarse voice. A few minutes later the cough subsided, but
his breathing was slightly faster than normal. On admission the child is in serious condition with
perioral cyanosis. Dyspnea with tachypnea up to 55/min, reduced vocal fremitus and dull
prcussion tone in the right lung. Tachycardia to 140/min, clear heart sounds, no murmurs.
Abdomen- no hepatosplenomegaly.

А. Which of the following tests would you request to make the diagnosis and plan for
emergency treatment?
(1 correct answer)
(2 Points)
ChestX-ray - likely to be foreign body
Troponin
Echocardiography
Blood gas analysis
Virology

60B. Chest X-ray shows opacification of the right lung, reduced intercostal spaces on the right
and mediastinal shift to the right. The left lung is intact. FBC- within reference range. Blood gas-
hypoxia, рСО2 – 55mmHg. Echocardiography shows normal myocardial contractility. What is
your diagnosis?
(1 correct answer)
(2 Points)
Empyema on the right
Foreignbodyaspirationwithatelectasisoftherightlung (mediastinal shift to the affected side
=> atelectasis)
Hypoplasia of the right lung
Severe pneumococcal pneumonia of the right lung

61C. What is management?


(1 correct answer)
(2 Points)
Antibioics
Oxygen therapy and monitoring
Emergencybronchoscopyforextractionofforeignbody
Surgical drainage of the pleural effusion

62. A-10-year-old boy has been complaining of fever and sore throat for the last 2 days. On
examination there is pharyngitis. The rest of his examination is unremarkable.

A. His rapid streptococcal antigen test is positive, so you prescribe oral antibiotic for:
(1 correct answer)
(2 Points)

Downloaded by Kato Caleb (katocalebnatamba@gmail.com)


lOMoAR cPSD| 41174528

20 days
10days (treatment for 10 days with penicillin V or erythromycin)
2 days
5 days

63B. 4 weeks later he returns because of palpitations and fatigue, especially after physical
exercise. His knees are swollen, red and very painful. It becomes clear that his mother gave him
the prescribed antibiotic for only 2 days as his fever and sore throat subsided quickly. On
examination he has RR 35/min; crepitations at the lung bases, HR 130/min; systolic murmur at
the apex, radiating to the left axilla. His liver is palpable 3 cm below the costal margin. No
splenomegaly. He also has a wide spread rash.
What is the likely diagnosis: F s
(1 correct answer) R a
(2Points)
I ƒ
acute rheumatic fever (both the joints and heart is affected)
infective endocarditis
post-streptococcal arthritis
infectious mononucleosis

64C. Which of the following medications would NOT be used :


(1 correct answer)
(2 Points)
long-term antibiotic prophylaxis - to prevent chronic rheumatic heart disease
diuretics and angiotensin-converting enzyme inhibitor - in cases of HF
anticoagulants
aspirin +/- corticosteroids - anti-inflammatory

65A. 4-year-old boy is admitted to hospital because his urine is red. For the last few days he has
been micturating less than usual. A week ago he had transient diarrhea after a day in the
mountains. On examination he looks unwell. He is pale, with subicterus, some petechia on the
legs and buttocks. Normal respiratory examination. Tachycardia to 140/min, blood pressure
130/85 mmHg. Abdomen- soft, difusely painful. Hgb- 85 g/L, Plt 33×10 9/L, WBC 15×109/L,
urea 11.1 mmol/L, creatinine 160 μmol/L, CRP- 35.2 mg/L., bilirubin – 75 μmol/L. Normal
coagulation – АРТТ, INR. Urine – 1(+) protein, lots of erythrocytes.
А. What is the most likely diagnosis?
(1 correct answer)
(2 Points)
Hemolytic-uremicsyndromeХУС (due to decreased platelets; anemia & increased urea).
Transient diarrhea (Ecoli) --- HUS
Sepsis
Pre-renal acute kidney injury
Henoch-Schonlein purpura

66B. What additional tests would you request to confirm the diagnosis?

Downloaded by Kato Caleb (katocalebnatamba@gmail.com)


lOMoAR cPSD| 41174528

(1 correct answer)
(2 Points)
Bone marrow biopsy
Blood culture
Freehemoglobin,redbloodcellsmorphology
Toxicology screen of vomit

67C. In view of the probable diagnosis what do you think is the abnormality on this blood
smear?
(1 correct answer)Immersive Reader
(2 Points)
Normal morphology of erythrocytes, leukocytes and platelets
Blast cells
Schizocytes (fragmented part of RBC & is seen in intravascular hemolysis) - sheared RBC
Hypersegmented neutrophils

The most common cause of death in the neonatal period in the developed world is:
(1 correct answer)
(1 Point)
Congenitalanomalies
Haemorrhagic disease of the newborn
Acute intoxication
Acute gastroenteritis

2Which of the following signs of richets appears earliest?


(1 correct answer)
(1 Point)
Craniotabes
Rachitic rosary
Bowing of the legs
Frontal bossing
Earliest sign = in infants -- softening of skill bones (craniotabes) --- then -- delayed closure of
fontanelles + frontal bossing

3Which of the following signs is NOT found in failure to thrive or kwashiorkor:


(1 correct answer)
(1 Point)
splenomegaly
hair fragility or depigmantation
hepatomegaly
generalized oedema
Kwashiorkor -- lack of protein = fatty liver infiltrates (Hepatomegaly), hair fragility, edema

Downloaded by Kato Caleb (katocalebnatamba@gmail.com)


lOMoAR cPSD| 41174528

4Which group of neonates is at high risk of the late form of haemorrhagic disease of the
newborn:
(1 correct answer)
(1 Point)
with disorders of the gastro-intestinal tract leading to fat malabsorbtion - fat sol vit = ADEK
breast fed babies - breast milk = low in Vit K
alloftheabove
treated with broad spectrum antibiotics - dysbacteriosis of gut flora -- no Vit K prod

5Which of the following is true for physiological jaundice of the newborn:


(1 correct answer)
(1 Point)
it is due to inadequate sunlight exposure
it is due to enzyme deficiency in erythrocytes
itisduetoincreasedhaemolysisoferythrocytesandloweractivityofliverglucuroniltransferase
newborns = >RBCs w/shorter lifespan i.e >hemolysis + lower amount of UGT i.e slower
conjugation of bilirubin
it is due to underdevelopment of the bile ducts in the liver

6What is true for caput succedaneum in a neonate?


(1 correct answer)
(1 Point)
It is a subperiostial bleeding - cephalhematoma
It can cause seizures - benign
Itextendsbeyondthemarginsoftheskullbones
It is confines within the margins of the skull sutures - cephalhematoma

7Which of the following is true for cyanosis due to respiratory causes:


(1 correct answer)
(1 Point)
isusuallyassociatedwithtachypnoea is
worse if the child is anemic increases
during crying
goes always with wheezing

8Physical findings during asthma exacerbation include:


(1 correct answer)
(1 Point)
Hyperresonant percussion tone, broncho-vesicular breathing on auscultation
Normal percussion tone, vesicular breathing with crepitations
Hyperresonantpercussiontone,vesicularbreathingwithprolongedexpirationandwheeze
Dull percussion tone, c

9Common causes for recurrent wheeze in children are all EXCEPT:

Downloaded by Kato Caleb (katocalebnatamba@gmail.com)


lOMoAR cPSD| 41174528

(1 correct answer)
(1 Point)
bronchopulmonary dysplasia - chronic lung disease in preemies due to delay in maturation --
chronic inflammation --- exudate accumulation -- wheezing, dyspnea etc
Asthma
gastro-esophageal reflux disease - aspiration of gastric contents -- GERD induced asthma
commoncold - >URT + acute

10Which serum laboratory test is most sensitive and most specific for pancreatitis?
(1 correct answer)
(1 Point)
Aspartate-amino-transferase (ASAT) - heart+liver
Lipase
Lactate-dehydrogenase (LDH) - general marker of tissue damage
Alanin-amino-transferase (ALAT) - liver + kidneys

11Pain in the lumbar area, hematuria and fever are most typical for:
(1 correct answer)
(1 Point)
Acute hemorrhagic cystitis
Acute post streptococcal glomerulonephritis
Pyelonephritisduetonephrolithiasis
Chronic glomerulonephritis

12Pancreatic insufficiency may lead to malabsorption of:


(1 correct answer)
(1 Point)
allnutrients - gen malabsorption etiology - gut disorders, pancreatic or biliary probs
proteins
carbohydrates
calcium

13Wilson’s disease may present with:


(1 correct answer)
(1 Point)
liverdamageandneuropsychiatricfeatures- excess copper --- accumulates in liver, brain,
kidneys + eyes
urinary tract infections
Vit. B12 deficiency anemia
Pneumonia

14In the last year, a 7-year old boy with Duchenne muscular dystrophy, that has not been
properly followed-up, develops dyspnea, orthopnea, tachycardia and hepatomegaly.
What is the most likely cause:

Downloaded by Kato Caleb (katocalebnatamba@gmail.com)


lOMoAR cPSD| 41174528

(1 correct answer)
(1 Point)
bronchial asthma
dilatedcardiomyopathy - complications DMD - resp m weakness, dilated CM
bacterial endocarditis
acute myocarditis

15Which of the following is NOT a communication that provides fetal circulation?


(1 correct answer)
(1 Point)
ductus arteriosus Botali - pulmonary a. to aorta
foramen ovale - RA to LA
ductusomphaloentericus - vitelline duct connecting the yolk to fetus
ductus venosus Arantii - umbilical vein to IVC

16In which disease breathing 100% oxygen does not raise oxygen concentration in blood?
(1 correct answer)
(1 Point)
myocarditis
ventricular septal defect
endocarditis
transpositionofgreatarteries - isolated circuits

17In the first decade of life, inverted (negative) T wave in V1-V3 of ECG is:
(1 correct answer)
(1 Point)
abnormal finding
normalfinding

18What is the recommended first line treatment for Kawasaki's disease?


(1 correct answer)
(1 Point)
antibiotics
highdoseintravenousimmunoglobulinandaspirin - reduce inflammation in vessels
NSAIDs
Corticosteroids

19Which of the following syndromes includes vomiting?


(1 correct answer)
(1 Point) Hypoproductive
anemia Persistent fetal
circulation Bronchial
obstruction
Increasedintracranialpressure - headache, NV, blurred vision, etc

Downloaded by Kato Caleb (katocalebnatamba@gmail.com)


lOMoAR cPSD| 41174528

20The most common cause of nephrotic syndrome in childhood is:


(1 correct answer)
(1 Point)
Poststreptococcal glopmerulonephritis - nephritic syndrome
Idiopathicnephroticsyndrome = Minimal change disease - most common eps in 2-5 yrs
Ig A nephritis - nephritic syndrome
Chronic glomerulonephritis - nephritic syndrome ?

21For which of the following conditions MACROSCOPIC hematuria is common?


(1 correct answer)
(1 Point)
Nephrolithiasis - gross hematuria, flank pain, dysuria, freq etc
chronic glomerulonephritis
interstitial nephritis
vesico-ureteral reflux

22In latent iron deficiency there is:


(1 correct answer) to
(1 Point) HEBEIlot g
Low hemoglobin, low serum iron and low ferritin - IDA
Low hemoglobin, low serum iron and high ferritin
Normal hemoglobin, low serum iron and low ferritin
g
Normal hemoglobin, normal serum iron and low ferritin - Pre-latent stage

23What is true for microspherocytosis?


(1 correct answer)
(1 Point)
Target cells are typical - thalassemia, IDA, sickle cell anemia etc
It is due to iron deficiency - microcytosis, hypochromia, poikilocytosis e.g pencil, target
Itisanautosomaldominantdisease - congenital defect in RBC membrane defect
It is due to glucos-6-phosphate dehydrogenase deficiency - heinz bodies, bite cells

24What аre the characteristic hemorrhagic skin lesions in immune thrombocytopenia?


(1 correct answer)
(1 Point)
Transient erythematous macules mainly on the body, proximal limbs and face
Hemorrhagic to hemorrhagic-necrotic, star-shaped, mainly in the hypostatic areas
Erythematous macular or urticarial lesions, progressing to papules and later to palpable purpura
Petechialskinrashmainlyonthelegsandface

25Neuroblastoma originates most commonly from:


(1 correct answer)
(1 Point)

Downloaded by Kato Caleb (katocalebnatamba@gmail.com)


lOMoAR cPSD| 41174528

bones
suprarenalgland - SNS inn of medulla
thymus
lymph nodes

26Honeymoon period in diabetes mellitus type 1 is:


(1 correct answer)
(1 Point)
any period of good glycemic control
the last 3 months when HbA1c is below 7%
the first several meetings between the patient and the endocrinologist preceding
communication of unfavorable prognosis
periodoflowexogenousinsulinneedsduetosomeendogenouspancreaticproduction - patient
looks like they are getting better but later - >insulin needed

27The main fetal factors for intaruterine growth are:


(1 correct answer)
(1 Point)
Thyroid hormones
Sex hormones
Glucoseandinsulin
Growth hormone
Fetus - insulin, early childhood (<3 yrs) - thyroid hormones, childhood - Thyroid + GH, puberty -
GH + sex hormones

28Klinefelter's syndrome is characterized by:


(1 correct answer)
(1 Point)
Malephenotypewithdystrophictestes
High levels of testosterone and female phenotype due to androgen receptor insensitivity --
primary gonadal failure (insensitivity) -- low levels of testosterone
Chromosome set 47 XYY - 47XXY
Ambiguous genitalia with presence of ovotestis - penis, small testes, female type pubic hair,
gynaecomastia

29Which of the following is NOT a typical finding in simple febrile seizure?


(1 correct answer)
(1 Point)
Average age is between 6 months and 6 years
An episode lasts less than 15 minutes.
Only one episode in a 24 hour period
Seizurestartsasafocalseizurethenbecomesgeneralized - typical \
s = gen only

Downloaded by Kato Caleb (katocalebnatamba@gmail.com)


lOMoAR cPSD| 41174528

30Which of the clinical signs is NOT specific for poisoning with organophosphates:
(1 correct answer)
(1 Point)
Sweating
Mydriasis
Bronchospasm
Bradicardia
PSNS - SLUDGE -- salivation, lacrimation, urine incontinence, diarrhea, GI cramps, emesis/vom

31Which of the following statements about cerebral palsy (CP) is NOT true:
(1 correct answer)
(1 Point)
the most common type of CP is spastic
the most common causes for it are antenatal
itisaprogressivedisorder - permanent but non-progesive disorder of movement + posture it
is the commonest motor disorder in childhood

32Which of the following diseases is characterized by generalized muscular hypotonia, severe


muscle weakness, tendin areflexia, preserved non-motor neuropsychological development?
(1 correct answer)
(1 Point)
Hydrocephalus
West syndrome - intellectual + developmental disabilities in Sx or cryptogenic epilepsy
syndromes
Spinalmuscularatrophy
Cerebral palsy - hypotonic form - Sx of UMN lesion - hyperreflexia, hypertonia

33Which of the following is true about patients with autism spectrum disorder?
(1 correct answer)
(1 Point)
They have good peer relationships with others
They don’t spontaneously seek to share their enjoyment
Theypointtodifferentobjectsfrequently
They have good eye-to-eye contact

34Which condition does predisposes to recurrent pneumonias in infants?


(1 correct answer)
(1 Point)
Atrial septal defect. --minor defect = low risk for recurrent infections
Tetralogy of Fallot -- pul stenosis -- pul hypovol
Brutonagammaglobulinemia - absence of mature B cells -- severe Ig def -- recurrent pyogenic
infections eg pneumonia
Congenital myasthenia

Downloaded by Kato Caleb (katocalebnatamba@gmail.com)


lOMoAR cPSD| 41174528

35An anxious mother is seeking advice for her 2 months old girl. The baby is fed exclusively with
breast milk by sucking from the nipples and does not fall asleep at the end of the feeding, but is
crying. The weight gain is about 500g/month and is probably declining. No water is given
between meals. What should be your advice:
(1 correct answer)
(2 Points)
starteachfeedingwithformulaandfinishwithsomebreastmilk
first try to increase breast milk production by sucking from both breasts during each feeding
replace instantly breast milk feeding with formula feeding
start instantly adding formula after each breast feeding

36A 2-year old boy comes to your office. He is just being potty trained. He enjoys pulling off his
clothes but cannot dress them up. He plays with joy and pretends to prepare tea for his mother.
He is using two-word sentences, asks "What is this? and builds a 6 blocks tower. He enjoys
running and climbing on the sofa.
What is true about the development of this child?
(1 correct answer)
(2 Points)
A consultation with speech therapist is recommended
He needs full assessment for autism spectrum disorder
A consultation with behavioural therapist is recommended
Hehasnormallanguagedevelopment

37This 2-year-old boy has been brought to hospital by social services. He lives with his mum,
who has learning difficulties, in a remote village. How would you describe his condition:
(1 correct answer)
(2 Points)
2nd degree hypotrophy
stunting
kwashiorkor
marasmus or hypotrophy 3rd grade poteeding
temperate
tammy
38An 8 days old neonate is admitted to hospital because of weak sucking and sleepiness. The Energy

from
clinical examination shows mottled skin, breathing rate 70/min, few diffuse small sized moist

140g/l, WBC 28.109/l, Neutrophils 65%, CRP 12mg/l. What is the most probable diagnosis:
rales, heart rate 170/min, liver at 3cm below the costal margin , spleen - at 2cm. CBC shows Hb
(1 correct answer)

lethargy
(2Points)
Neonatal sepsis
IN 1.5 20mgIL
Tetralogy of Fallot
main Hyaline membrane disease usually first 48 72hr premature bbies
Congenital leukemia

Downloaded by Kato Caleb (katocalebnatamba@gmail.com)


lOMoAR cPSD| 41174528

39A 12 year old boy is brought to the emergency department because of fast developing itching
rash, edema of the lips and hoarse voice. These symptoms appeared half an hour after
cephalexin ingestion that has been prescribed 3 days ago for acute bronchitis and sinusitis.
What is the explanation of the symptoms that urged the admittance to the emergency
department?
(1 correct answer)
(2 Points)
anaphylaxis
subchordal laryngitis
epiglotitis
purlent sinusitis

40A 14-year-old girl became acutely unwell with fever, sore throat and malaise. Because
symptomatic treatment had no effect the family doctor prescribed her Ampicillin. The next day
she develops maculo-papular rash on the body. On examination she is febrile- 38°C, has mild
edema of the eyelids, severe nasal obstruction, neck lymphadenopathy, enlarged tonsils with
grayish exudate on them. Lab results show: leucocytes18х10^9, with 70% lymphocytes, one
third of them being similar to monocytes, Hgb and platelets within reference range. Mild
elevation of ASAT and ALAT. CRP- 5 mg/ml.
Which is the most likely diagnosis:
(1 correct answer)
(2 Points)
streptococcal tonsillitis
infectiousmononucleosis - gen maculopapular rash when ampicillin or amoxicillin is given to
EBV, monocytosis, fever, malaise, LAD, purulentish exudate on tonsils …..
herpangina
Hodgkin’s lymphoma

41A 15-year-old develops fever, malaise, sore throat, and a dry, hacking cough over several days.
He does not appear to be particularly sick, but on examination of his chest there are diffuse
rhonchi. ESR is 20mm. WBC 18.109/l , neutrophils are 60%. Which of the following is the most
likely pathogen?
(1 correct answer) (2
Points) Staphylococcus
aureus
Mycoplasmapneumoniae - atypical pneumonia = flu like Sx w/o Sx of lower RTI
Haemophilus influenzae
Pneumocystis carinii

42A 3-year-old girl presents with a history of watery stools for the past 2 months. Her mother
deniеs any additional symptoms as nausea, vomiting, abdominal pain, fever or presence of
blood in the stools. The girl drinks a lot of fruit juice. On examination, she is afebrile and in good
general condition. The abdomen is soft on palpation and below the level of the thorax. The rest

Downloaded by Kato Caleb (katocalebnatamba@gmail.com)


lOMoAR cPSD| 41174528

of the examination is unremarkable. ЕSR, CRP, fecal calprotectin, anti TTG, anti EMA and fecal
cultures are normal or negative. What is the most probable diagnosis:
(1 correct answer)
(2 Points)
acute gastroenteritis
Crohn’s disease
chronicnon-specificdiarrhea - loose stools in preschool age children w/normal physical exam --
excessive ingestion of fruit juice esp apple juice can be a cause
coeliac disease

43A three-month old infant is admitted because of pneumonia, that was treated for two weeks
at home without improvement. On the 3rd day after the birth a heart murmur was detected
and parents were advised to consult a pediatric cardiologist after discharge from the neonatal
unit, but so far they have not done it. On examination the child is pink, with tachydyspnoea and
bilateral crepitations. There is also a loud P2; rude pansytolic murmur 3-4/6, loudest at the
lower left sternal border. Liver is palpable 3-4 cm below the costal margin.
What is the most likely diagnosis:
(1 correct answer)
(2 Points)
Eisenmenger syndrome - cyanotic
tetralogy of Fallot - cyanotic
atrial septal defect - systolic ejection murmur at 2ICS
largeventricularseptaldefect(VSD) - holo/pan systolic murmur at left sternal border, acyanotic
ie pink
overor
reduacleertdness
44A 5 days old neonate is admitted to the intensive care unit because obtundation, paleness,
depressed neonatal reflexes and low muscle tone. Breathing rate is 75/min. Heart rate -
200/min. Heart murmur 2/6 is heard on the precordium. Liver is palpable 4 cm below the costal
margin. Blood pressure in the right arm is 98/70. No pulses are palpable on both femoral
arteries. What is the most probable diagnosis?
(1 correct answer)
(2 Points)
Transposition of great arteries
Aortic valvular stenosis
Atrio-ventricular channel (ventricular septal and atrial septal defects)
Coarctationoftheaorta - weak + delayed/no fem pulse, hypotension in lower limbs..

45. A 15-year-old boy with no past medical history of note has been vomiting and complaining
of dull lumbar pain for the last few days. He has had less frequent micturation then usual and
his urine has become cloudy. On examination, the child looks unwell, pale with mild periorbital
edema. No deviations from the remaining status except for blood pressure, which is 148/83
mmHg.

demoof
T
His test results are: Hb- 110 g/l; PLT – 210; creatinine 130 µmol / L ( 35-75 µmol / L) , urea 9.5
mmol / l ( 3,2-8,2 mmol/l); urine:
race
rich
widey

Downloaded by Kato Caleb (katocalebnatamba@gmail.com)


lOMoAR cPSD| 41174528

protein 2 (+), sediment - leukocytes 8-10 per field, erythrocytes -20-25 per field, specific gravity
- 1010 mOsm / l.
What is the most likely diagnosis?
(1 correct answer)
(2 Points)
Prerenal acute kidney injury
Renal acute kidney injury
Nephrolithiasis
Urinarytractinfection

46 A 18 days old formula fed boy is admitted to hospital because of projectile vomiting.
Reduced skin elasticity and turgor were found. His body weight is equivalent to the birth weight.
Lab data reveal hyponatremia with hyperkalemia and metabolic acidosis. Which is the most
likely diagnosis:
(1 correct answer)
(2 Points)
cow's milk intolerance
meconium ileus
congenital adrenal hyperplasia due to 21-hydroxylase deficiency - no aldosterone = no Na+/H20
reabsoprtion (hyponatremia), no K+ excretion (hyperkalemia)+ H+ excretion (metabolic acidosis)
hypertrophic pyloric stenosis

47An 11-year- old girl has had breast development from the age of 9 years, followed by
appearance of pubic hair growth. The onset of menarche is at the age of 11 years. Her bone age
corresponds to her chronological age. Her mother had her menarche at the age of 12 years.
What is the most likely diagnosis?
(1 correct answer)
(2 Points)
Premature thelarche
Gonadotropin-dependent / central premature puberty
Gonadotropin-independent / peripheral premature puberty
Physiologicalpuberty - normal age for all features (thelarche, adrenarche, menarche >9 yrs)

48A child from the first pregnancy is born via cesarian section at 28th week of gestational age
because of sponaneous rupture of the amniotic sac. His weight was 1050 g, height - 39 cm,
head circumference - 28 cm. Apgar - 5 on the first minute. What are the risks for the child?
(1 correct answer) L o 10
(2 Points)
Riskofrespiratorydistressduetosurfactantdeficiency - surfactants prod >20GW but
distributed throughout lungs >28-32 GW -- Sno surfactant -- alveolar collapse-- atelectasis ---
hypoxemia + hypercapnia (neonatal resp distress syndrome/hyaline membrane disease)
Risk of meconium ileus
Risk of congenital anomalies
Risk of hypothyroidism

Downloaded by Kato Caleb (katocalebnatamba@gmail.com)


lOMoAR cPSD| 41174528

49A 4 year old boy is brought to pediatric surgery department because of obstipation,
distended abdomen, decreased appetite and nausea since two weeks. Abdominal CT with oral
contrast reveals ileal obstruction by a mass that might be also fecaloma. CBC is normal. ESR is
45mm. LDH is 2800IU/l. Mantoux test and IGRA (T-spot) test are negative. What is the most
probable diagnosis?
(1 correct answer)
(2 Points)
Non-Hodgkinlymphoma
TB typhlitis
Acquired hypothyroidism
Hirschprung disease

50. A 9-year-old girl has been referred because her weight centile has increased in the last 10
months. She is an excellent student and has no other complaints. Parents report no dietary
change. On examination there is no acne, striae or hirsutism. Her weight is on the 97th centile
and her height is on the 91st centile.
What is the most adequate course of action:
(From 1 to 5 correct answers out of 5 possible answers; 1 point for each correct answer, minus 1
point for each wrong answers)
further tests to rule out hypothyroidism -- no puffy eyes, constipation, protruding belly,
neurodevelopmental probs etc
examine growth hormone and IGF1 - height - 91st centile -- tall stature >98th centike
adviceabouthealthydietandincreasedphysicalactivity - overweight (>91st centile) almost
obese
further tests to rule out Cushing's syndrome - no acne, striae or hirsutism
adviceabouttheincreasedriskofabnormallipidlevelandhighbloodpressure

51A 3-month- old infant is admitted to the hospital due to slow growth of the head. He was
born at the 37 week of gestational age, with birth weight 2100 gr. and head circumference of
30 cm. During early neonatal period he developed jaundice and was treated with phototherapy.
At home, the head circumference has increased by 1,5 cm in 3 months and the weight by 2.5 kg.
Examination reveals persistent jaundice, hepatosplenomegaly, and lab tests- thrombocytopenia,
anemia, elevated transaminases, mixed hyperbilirubinemia. Abdominal ultrasound shows
hepatomegaly with increased echogenicity of the parenchyma. Transfontanel ultrasound
revealed brain echogenicities, suspicious of calcifications. What additional tests will you suggest
to start with?
(From 1 to 5 correct answers out of 5 possible answers; 1 point for each correct answer, minus 1
point for each wrong answers)
MRIbrainformalformations - visualise calcifications in congenital infections
Genetic testing for autosomal recessive isolated microcephaly
Liverbiopsytoruleoutbiliaryatresia - see active inflammation w/bile duct degeneration +
fibrosis -- biliary atresia = malabsorption -- low weight?
Sweat test - CF does not have microcephaly, calcifications, HSM

Downloaded by Kato Caleb (katocalebnatamba@gmail.com)


lOMoAR cPSD| 41174528

Congenitalinfectionsscreenofmotherandchild.---microcephaly, calcifications seen in


congenital CMV, Toxoplasmosis etc

52А 2-year-old girl is admitted to hospital because of pneumonia. She was treated by her family
doctor with an antibiotic at home but with no effect. She was born on time with birth weight
3100 gr and length 51 cm. Uneventful perinatal period. She has been suffering from frequent
chest infections. She has good appetite , but frequent bulky, fatty stools. On examination she
looks unwell, has non-productive cough, tachypnoea at 50/min, emphysematous chest, wheeze
and crepitations bilaterally.Cardio-vascular system- no abnormalities detected. Abdomen- soft,
slightly distended, no palpable organomegaly. Current weight 10.5 kg, height 80 cm ( both are <
3rd centile). What tests would be most useful in making the diagnosis?
(From 1 to 5 correct answers out of 5 possible answers; 1 point for each correct answer, minus 1
point for each wrong answers)
chest X-ray - progression of pneumonia, any complications etc
Sweat test - CF - freq chest infections, obstructive disease = hyperinflation, steatorrhea due to
malabs, poor weight gain etc
fecal calprotectin
Abdominal X-ray - abdo is near normal, no organomegaly
anti-tTG antibodies - celiacs doesn't account for recurrent chest infection + steatorrhea

53A 4-year-old boy with normal development and no past medical history of note is admitted
to hospital because of mild petechial rash, mainly on the face, neck and torso that occurred
the previous day. Physical examination does not reveal any other abnormalities. Blood tests on
ɠ
admission are PLT - 10 x109/l (N- 150-450ࡣ10 /l)with normal levels of hemoglobin, WBC, RBC
and LDH – normal. What is the right management?
(From 1 to 5 correct answers out of 5 possible answers; 1 point for each correct answer, minus 1
point for each wrong answers)
Monitoringonly,asthepatientisnotcurrentlybleedingprofusely - could be immune
thrombocytopenia --- self limiting --- only supportive care needed
i.v. antibiotics - normal WBC, LDH = no inflammation
Treatment with corticosteroids or high-dose immunoglobulin after confirmation of the diagnosis
by bone marrow biopsy -- not needed if only plt is low
Splenectomy - only in severe cases + only >7 yrs age (risk of post splenectomy sepsis)
Immediate platelet transfusion - most children do not need Tx even if its as low as 10 x109/l in
immune thrombocytopenia
54A 13-month-old boy, previously healthy, no fever, no rash, appears pale. Examination reveals
a palpable liver 3 cm below the right costal margin and uncertainly palpable spleen 0.5 cm
below the left costal margin. Lab findings include: hemoglobin - 64 g/l (1-12 yrs - <110 =
anemia) ; mean corpuscular volume (MCV)-67 fL(80-96fl); reticulocytes 1%, normal WBC,
platelets and differential blood counts; mild indirect hyperbilirub
inemia. Of the following, the MOST valuable tests to confirm the diagnosis are:(From 1 to 5
correct answers out of 5 possible answers; 1 point for each correct answer, minus 1 point for
each wrong answers)

Downloaded by Kato Caleb (katocalebnatamba@gmail.com)


lOMoAR cPSD| 41174528

abdominal ultrasonography - no indication -- Slight HSM due to anemia


serumiron,totalironbindingcapacity,andserumferritin -- microcytic anemia = check these for
IDA vs anemia of chronic inflammation
blood group
bone marrow study - not needed to Dx microcytic anemia
hemoglobinelectrophoresis - thalassemia = microcytic anemia -- Hb electrophoresis = HbF,
>HbA2)
55A 15-year-old girl has a weight reduction of 10 kg in two months. She is generally an excellent
student, but during the last month her grades have deteriorated. She has become very irritable
and emotional. She also reports more frequent and sometimes watery stools. During the
examination her skin is warm and wet, she has slight hand tremor, heart rate - 160 / min, clear
sounds, blood pressure - 125/45mmHg. She blinks rarely and her look is strange - see the
picture.
What are the most appropriate initial investigations ?
(From 1 to 5 correct answers out of 5 possible answers; 1 point for each correct answer, minus 1
point for each wrong answers)
Consultation with a psychologist / psychiatrist to rule out eating disorder
Colonoscopy to rule out Crohnˋs disease
Ultrasound of the thyroid gland- hyperthyroidism graves = enlarged + hyperechoic
Thyroid hormones tests -- hyperthyroidism esp graves - ͫT3/T4 + ͭTSH
Abdominal CT scan for pheochromocytoma -- >catecholamines - HT! profuse sweating,
heachaches, tachycardia

56A previously healthy 15-month old girl is febrile, irritable and refusing to bear weight on her
right leg. On examination she looks unwell. Her right leg is held in semiflexion and there are no
spontaneous movements. She gets severely distressed on passive movement of the right leg.
Which of the following are particularly important for her management ?
(From 1 to 5 correct answers out of 5 possible answers; 1 point for each correct answer, minus 1
point for each wrong answers)
bone marrow aspiration for smear microscopy - joint prob
ultrasoundexaminationofthehipjointandaspiration - septic arthritis /sepsis? due to fever,
unwell, pain
lumbar puncture thought
urinary culture - UTI --dissemination?? peed
blood culture, ESR, CRP -- sepsis?

I
57An 8-year-old boy falls from a fence 2 meters high, after which he is unconscious for 2
minutes, followed by vomiting twice. On examination he is bradylalia and bradypsychic. Blood is
leaking from his right ear. The rest of his examination is normal. What is the initial
management:
(From 1 to 5 correct answers out of 5 possible answers; 1 point for each correct answer, minus 1
point for each wrong answers)
Lumbar puncture Explorative
craniotomy Antiepileptics
prophylactically

Downloaded by Kato Caleb (katocalebnatamba@gmail.com)


lOMoAR cPSD| 41174528

Hospital admission for observation


Computed tomography of the head

58A thirteen-year-old girl presents with history of recurrent liquid diarrhea for few months, 1 to
2 times a day, without presence of blood in stools or fever. No significant weight loss. On
examination, the child has no abdominal pain or distention. Which tests and procedures are
appropriate:
(From 1 to 5 correct answers out of 5 possible answers; 1 point for each correct answer, minus 1
point for each wrong answers)
skin-prick test for food allergens
anti-tTG (immunoglobulin A tissue transglutaminase antibodies) and EMA (endomysial
antibodies) There is recurrent watery diarrhoea so to rule out celiac disease.
gastroscopy and biopsy
esophageal pH monitoring - no Sx of GERD
contrast X-ray of the colon

59A one-year old boy is admitted to hospital because of fever up to 39oC in the last 24 hours.
Cough has appeared 4 days ago. The examination shows bad general condition, bluish lips,
breathing rate of 50/min, epigastic retractions, nasal flaring, absent breath sounds at the base
of the right lung. The following X-ray was produced 1 hour after admission.
A. What is the diagnosis?
(1 correct answer)
(2 Points)
Diaphragmatic hernia with penumonia in the compressed lung
Pneumonia with right-sided pneumothorax -- absent breath sounds only at base of right lung
Valvular emphysema in the left lobe
Pneumoniawithpleuraleffusionintherightlobe
60Q59 B. After 1 hour lab data revealed Hb 100g/l, WBC 35.109/l, neurtrophils 85%, platelets
110.109/l, CRP 110mg/l. What causative microorganism do you suggest?
(1 correct answer)
(2 Points)
Staphylococcusaureus -- tends to complicate w/pleural effusion
Micoplasma penumoniae - atypical pneumonia
Pneumocystis jirovecii (carinii)
Influenza virus - atypical

61Q59 C. Which procedures are necessary?


(1 correct answer)
(2 Points)
laryngoscopy
uroculture
bloodcultureandpleuralpuncture - to confirm Dx agent + Tx
fecal culture

Downloaded by Kato Caleb (katocalebnatamba@gmail.com)


lOMoAR cPSD| 41174528

62A 6-year-old girl presents to the pediatric department because of red urine. The girl has no
dysuria, frequency or abdominal pain and there is no history of trauma. She is otherwise well
but she had a sore throat 10 days ago for which she received only Paracetamol. She had a
urinary tract infection at the age of 4 years, but following a normal renal ultrasound she was
discharged from clinic. There is no family history of renal problems but her grandmother has
hypertension. On examination she looks well and apart from blood pressure of 130/85 mmHg
no other abnormalities are detected. Her FBC, urea, electrolytes, bilirubin, albumin,
transaminases and clotting are within reference range. Urine dipstick – blood 4+, protein 2+,
leucocytes-neg, nitrites – neg.
A. What further investigations should be performed?
(1 correct answer)
(2 Points)
Urine protein electrophoresis
Urine microbiology
plain abdominal X-ray
anti-streptolysintitre(AST),C3andC4 -- possible post strep glomerulo nephritis - Dx = evidence
of prev infection w/GAS (ASO ͫ or antiDNAse B or throat culture) + C3ͭbut normal C4

63Q62 B. What is the most likely diagnosis?


(1 correct answer)
(2 Points) Henoch–
Schönlein purpura
Acutepost-infectiousglomerulonephritis
steroid-sensitive nephrotic syndrome
urinary tract infection

64Q62 C. What is the most appropriate treatment?


(1 correct answer)
(2 Points)
corticosteroids
antibiotics for 7-10 days
High dose of immunoglobulins
saltandfluidrestrictionandantihypertensives - mostly self limiting --supportive Tx eg HT -
diuretics, ACEi
65A 5-year old girl complains of night bone pains in the two legs, malaise and unwillingness to
play with other children. She was previously healthy. On physical examination she is pale,
afebrile, no jaundice or rash, with splenomegaly at 3 cm below the costal margin. No
lymphadenopathy. No other pathological signs. Her CBC: HGB- 73 g/l, WBC- 2.3x109/l, PLT-
62x109/l.
A. What do you suspect?
(1 correct answer)
(2 Points)
Systemic form of Juvenile idiopathic arthritis

Downloaded by Kato Caleb (katocalebnatamba@gmail.com)


lOMoAR cPSD| 41174528

Acuteleukemia- anemia, leukopenia, thrombocytopenia, bone pain (Sx of BM infiltration) +


Splenomegaly, etc -- most common for her age ALL
Bone tumor
Osteomyelitis

66Q65 B. Which tests will be most useful in establishing the diagnosis?


(1 correct answer)
(2 Points)
Bonemarrowexamination -- BM biopsy + FBC --- hypercellular BM w/>20%blasts
CT scan
Blood culture
Antinuclear antibodies immunology

67Q65 C. What is the treatment?


(1 correct answer)Immersive Reader
(2 Points)
Surgery
Antibiotics
Immunosupressants
Chemotherapy

1.Mortality is highest in which age group?


(1 correct answer)
(1 Point)
Intheneonatalperiod
During puberty
From the end of the 1st month to the end of the 1st year
When the child starts attending nursery and kindergarten and is in contact with more people
2.The neonatal period is exactly :
(1 correct answer)
(1 Point)
From 1st to the 14th day
From1sttothe28thday
From 1st to the 7th day
The whole 1st month
>28-1 yrs - infant
1-3 - early childhood
4-6 - preschool
7-18 - school age

Downloaded by Kato Caleb (katocalebnatamba@gmail.com)


lOMoAR cPSD| 41174528

3.If the mother of a 3-month-old infant has hypogalactia, what is the most appropriate food to
top up the breast milk feeds
(1 correct answer)
(1 Point)
2% fat yoghurt - poor nutritional value
Formulamilk - modified to nutritional needs of infants e.g >iron
Unmodified cow’s milk -- never give unmodified cow’s milk --- low iron w/poor absorption
Vegetable puree - weaning only around 6 mo (17-26 weeks)
4.Which of the following endocrine glands is not part of the normal hormonal regulation of
puberty
(1 correct answer)
(1 Point)
Ovaries - Estrogen
Adrenal glands - Androgens
Parathyroidgland - PTH - ͫ Ca2+ in blood
Pituitary gland - FSH/LH
5.The first sign of puberty in girls is:
(1 correct answer)
(1 Point)
Breastenlargement - thelarche -- 1st in boys = testicular enlargement - 4ml >
Axillary hair growth
Menarche
Pubarche
6.If a newborn has a heart rate of 85 beats/min during sleep, this is:
(1 correct answer)
(1 Point)
Bradycardia
Tachycardia
Depends on the gestational age at birth
Normalheartrate - normal <1 yrs = 110-160 but w/sleep ͭup to 80 bpm
7.A 1-year-old child, who was born at term, with current weight 9 kg and height 75 cm, has head
circumference of 54 cm.
(1 correct answer)
34 12 46
(1 Point)
Thisismacrocephaly
The interpretation of the head circumference depends on the wight of the child
This is microcephaly
This head circumference is normal for this age
8.Direct hyperbilirubunemia may be present in all EXCEPT:
(1 correct answer)
(1 Point)
Chronic hepatitis - UCB+CB
Cystic fibrosis - cholestasis - CB
Atresia of the extrahepatic bile ducts - posthepatic - CB

Downloaded by Kato Caleb (katocalebnatamba@gmail.com)


lOMoAR cPSD| 41174528

Breastmilkjaundice- UCB
9.There is neonatal screening for which of the following :
A (1 correct answer)
(1 Point)
Hereditary spherocytosis
Progressive muscular dystrophy
Congenitaladrenalhyperplasia - Guthrie test = congenital hypothyroidism, CF (immunoreactive
trypsin), PKU, hemoglobinopathies e.g HbS , galactosemia etc
Monogenic diabetes mellitus
10.In the most common type of congenital hypothyroidism, according to its etiology, there is:
(1 correct answer)
(1 Point)
Elevated thyroid hormones and low thyroid-stimulating hormone (TSH) - hyperthyroidism
Normal thyroid hormones and low thyroid-stimulating hormone (TSH) - subclinical
hyperthyroidism
Low thyroid hormones and low thyroid-stimulating hormone (TSH) -
Lowthyroidhormonesandhighthyroid-stimulatinghormone(TSH) - most common acquired =
hashimotos , congenital = dysgenesis both -- low T3/4 but (-) fb = ͫTSH
11.The three “D” symptoms – dyspnea, dysphagia and dysphonia are present in:
(1 correct answer)
(1 Point)
Lacunar tonsillitis
Acuteepiglottitis - muffled voice (dysphonia), painful swallowing due to movement of inflamed
epiglottis (dysphagia)--drooling, dyspnea
Acute tracheitis - no dysphonia + dysphagia - lower
Acute pneumonia - no dysphonia + dysphagia
12.In pneumothorax there is:
(1 correct answer)
(1 Point)
Hyperresonantpercussiontone,absentbreathsoundsandnegativebronchophony
Hyperresonant percussion tone, bronchial breathing and positive bronchophony
Dull percussion tone, vesicular breath sounds with prolonged expiration and positive
bronchophony
Hyperresonant percussion tone, inspiratory dyspnea, bronchial breathing and negative
bronchophony
13.Purulent pleural effusion is characteristic of:
(1 correct answer)
(1 Point)
Tuberculosis
Mycoplasma pneumonia - atypical
Systemic Lupus Erythematodes
Staphylococcalpneumonia - S. aureus = rapidly progresive w/early complications eg. pleural
effusion, pneumatocele (tissue destruction) -- abscess
14.Which of the following symptoms is present in respiratory failure?

Downloaded by Kato Caleb (katocalebnatamba@gmail.com)


lOMoAR cPSD| 41174528

(1 correct answer)
(1 Point)
Chest deformity - pectus excavatum
Tachydyspnea
Persistent dry cough
Bradycardia
15.Hematuria in Henoch-Schonlein purpura is due to:
(1 correct answer)
(1 Point)
Glomerulonephritis-- immune complex deposition (small vessel vasculitis)
Thrombocytopathia
Hypofibrinogenemia
Thrombocytopenia -- HSP - normal or even ͫplt
16.Which blood sugar value makes the diagnosis of type 1 diabetes mellitus certain and
differentiates it from impaired glucose tolerance:
(1 correct answer)
(1 Point)
Fasting blood glucose >6mmo/l
Fastingbloodglucose>7.8mmol/l
Postprandial blood glucose >7.8 mmol/l --- prediabetic value - 7.8-11mmol/l
Postprandial blood glucose > 10 mmol/l . ->11.1?
17.What is typical for Cushing’s syndrome/disease in childhood:
(1 correct answer)
(1 Point)
Android (central) type obesity -- moon face, buffalo hump..
Arterial hypertension - Na/H20 retention, cortisol = potentiates effect of catecholamines..
Alloftheabove
Reduced growth velocity - cortisol = ͫbone resorption/destruction
18.All of the following is true for congenital pyloric stenosis EXCEPT:
(1 correct answer)
(1 Point)
Metabolic alkalosis with hyponatremia and hypochloremia -- due to persistent projectile
vomiting
Visible gastric peristalsis - L-R direction Diarrhea -
- PS = Gastric outlet obstruction Palpable
hypertrophic pylorus - olive shaped mass
19.Which disease does not cause liver cirrhosis?
(1 correct answer)
(1 Point)
Parainfluenzainfection - resp infection commonly causing croup in children
Atresia of bile ducts
Wilson’s disease
Cystic fibrosis
20.What are the typical hemorrhagic skin lesions in immune thrombocytopenia ?

Downloaded by Kato Caleb (katocalebnatamba@gmail.com)


lOMoAR cPSD| 41174528

(1 correct answer)
(1 Point)
Hemorrhagic, necrotic, star-shaped on the buttocks
Palpable hemorrhagic rash which is symmetrical on the extensor surfaces of the limbs - henoch
schonlein purpura
Petechialskinrash- mucocutaneous bleeding e.g petechiae, purpura, ecchymosis, epistasis etc
Transient erythematous macules mainly on the body, proximal parts of the limbs and face
21.Hemarthrosis is typical for:
(1 correct answer)
(1 Point)
Immune thrombocytopenia - superficial bleeding
HemophiliaA - esp at end of 1 yr when they start crawling + walking -- fall -- minor injury --
bleeding into joints
Fanconi’s anemia
Henoch – Schonlein purpura - arthritis/arthralgia
22.Transient hypogammaglobulinemia is due to:
1. (1 correct answer)
(1 Point)
Sepsis
Depletionofthetrans-placentallytransferredimunoglobulinsG
Increased levels of IgM
Low neutrophil count
23.Folate deficiency anemia in infants is more common in:
(1 correct answer)
(1 Point)
Infants fed only with goat’s milk - goats milk = no vit D, B12, iron + folic acid
Infants whose diet does not contain meat
Late weaning with potato puree
Infants fed only with cow’s milk
24.A 2-day-old neonate, born at 38 gestational weeks, has Hgb 125 g/l.
(1 correct answer)
(1 Point)
the neonate has anemia due to bone marrow immaturity
theneonatehasanemiaandfurtherinvestigationsarerequired- neonate anemia <140g/l,
<1yrs = <100g/l, 1-12yrs - ,110g/l
the hemoglobin level is normal for this age and no further investigations are required о
the neonate has iron-deficiency anemia
25.Which medication is NOT used for treating neonatal seizures?
(1 correct answer)
(1 Point)
Potassiumchloride15% - hypokalemia - cardiac arrhythmias
Calcium gluconate 10% - hypocalcemia = rare cause of seizure
Glucose 25% - hypoglycemia = common cause of seizures
Vitamin B6 - VitB6 deficiency = rare cause of intractable seizure in neonates

Downloaded by Kato Caleb (katocalebnatamba@gmail.com)


lOMoAR cPSD| 41174528

26.What are the characteristic abnormalities in the differential cell count (DCC) in Hodgkin’s
lymphoma?
(1 correct answer)
(1 Point)
lymphocytopenia, monocytosis, eosinophilia
neutrophilia with normal lymphocytes
lymphocytosis, monocytosis, normal eosinophil count
neutropenia,lymphocytosis -- hodgkin = monoclonal proliferation of reedstern berg cells = B ly
+ pushes other cell lines down
27.Which symptom is characteristic for peripheral motor neuron damage?
(1 correct answer)
(1 Point)
Tendon and periosteal hyperreflexia - central
Babinski sign - central
Increased muscle tone with spasticity - central
Musclehypotrophy- denervation atrophy
28.Idiopathic (pure) nephrotic syndrome is treated with:
(1 correct answer)
(1 Point)
Antibiotics
Immunoglobulin
Only with Human albumin infusions and Furosemide
Corticosteroids
29.Which of the following is TRUE regarding acute renal injury:
(1 correct answer)
(1 Point)
there is evidence for reduced bone mineralization from imaging studies
usuallyassociatedwitholiguria
there is permanent reduction in glomerular filtration rate
prognosis is usually bad in childhood
30.Which of the following is true for oligoarticular juvenile idiopathic arthritis?
(1 correct answer)
(1 Point)
affects 5 and more joints during the first 6 months - <5 joints
affects only the joints of the axial skeleton - med+ large joints e.g knee
affects1to4jointsinthefirst6months
affects the small joints of the hands and feet symmetrically - asymmetrical + large joints
31.Which of the following congenital heart defects has a right to left shunt?
(1 correct answer)
(1 Point)
TetralogyofFallot
Persistent ductus arteriosus - L-R
Isolated ventricular septal defect - L-R
Isolated atrial septal defect - L-R

Downloaded by Kato Caleb (katocalebnatamba@gmail.com)


lOMoAR cPSD| 41174528

32.Acute lymphoblastic leukemia may present with:


(1 correct answer)
(1 Point)
Meningosisleukemica - leukemic meningitis -- ca cells spread to meninges - inflammation (ALL -
likes to go to CNS, Testes)
Obstructive internal hydrocephalus
Acute disseminated encephalomyelitis
Macrocrania
33.Emergency treatment of anaphylactic shock requires emergency application of:
(1 correct answer)
(1 Point)
Infusion of electrolytes and glucose solutions
Antihistamine
Adrenalin
Parenteral corticosteroid
34.The FBC of a 1-year-old child with normal growth, who is fed mainly cow’s milk, shows Hgb
– 95g/l, Er 3.8х 10/12; MCV – 66. Mother had anemia during pregnancy. At the moment
maternal Hgb is 123g/l; Еr- 4.2; MCV 80. What is the most likely diagnosis in the child ?
(1 correct answer)
(2 Points)
Thalassemia minor
Irondeficiencyanemia -- cow’s milk -- >iron but poor absorption (10%) and 1 yr = much more
iron need
В12-deficiency anemia
Hemoglobin and erythrocytes are normal for age

35.Two children in one family ate soup, which had been out of the fridge for 12 hours, and
peaches, which the granddad brought from an orchard. In 30 minutes both children started
vomiting, had abdominal pain, diarrhea, hypersalivation, sweating and labored breathing. The
ambulance crew established that the children had tachydyspnea, vesicular breath sounds with
prolonged expiration, wide spread rhonchi, bradycardia and pinpoint pupils. What is the correct
management?
(1 correct answer)
(2 Points) cholinergic
TreatmentwithAtropinshouldbestartedimmediatelyasthisisorganophosphatepoisoning
This is acute food poisoning, may be with Salmonella
The children should be admitted and isolated in an infectious diseases ward
These children probably have neuroinfection
36.A 2-year-old child has 3 episodes of loss of consciousness within 1 week after arching of the
back, trismus and cyanosis for around 30 sec. The episodes are preceded by crying, followed by
breathing out but no breathing in and after this the child has a seizure. On all occasions the
parents refused to fulfill the child’s demands before the episodes occurred. These episodes are
most likely:
(1 correct answer)

Downloaded by Kato Caleb (katocalebnatamba@gmail.com)


lOMoAR cPSD| 41174528

(2 Points)
Hypoglycemic seizures
Breathholdingattacks -- when toddlers are angry, afraid, pain -- hold their breath in expiration
-- cyanosis --- hypoxia -- seizure
Epileptic seizures
Hysteria
37.A 1-year -old child with abdominal tumor in the right hypochondrium has recurrent episodes
of arterial hypertension up to 150/100 mmHg with tachycardia, pallor and sweating. The
abdominal tumor is most likely:
(1 correct answer)
(2 Points)
Non-Hogkin lymphoma
Neuroblastoma -- freq starts in adrenal medulla (SNS inn)--- ͫcatecholamines
Rhabdomyosarcoma
Hemangiendothelioma of the liver

38.A 3-year-old boy is admitted because of pain in the left shoulder. 2 weeks ago he had acute
tonsillitis, and 3 days ago complained of pain in the right inguinal area while walking. On
examination he has a soft systolic murmur at the apex. ECG shows first degree AV block. What
investigation would you request?
(1 correct answer)
(2 Points)
antistreptolysinantibodiestiter -- possible rheumatic fever -- migratory polyarthritis/arthralgia,
endocarditis = murmur, prolonged PR interval
antinuclear antibodies - JIA,SLE etc
blood culture
serological tests for Chlamydia

39.A 15-year-old boy, who is an active athlete, with no family and past medical history of note,
is found to have frequent supraventicular extrasystoles on the ECG during a routine check-up.
He has no symptoms. On examination he looks well, HR 62/min, RR 115/70 mmHg. What is the
right management?
(1 correct answer)
(2 Points)
Perform24-hourECG-- possible SVT?
There is no need for further tests, but he needs to reduce his training.
This is the so called “ athlete’s heart”. There is no need for further investigations, treatment or
limitation to his physical training. - cardiomegaly + low resting HR
Start treatment with anti- arrhythmic medication straight away.

40.A 1-month-old child with uneventful perinatal history has been having
difficulty feeding from birth-quickly lets go of the breast and sweats. For the last two days he
has been breathing fast, has become pale and is refusing to feed. Examination reveals

Downloaded by Kato Caleb (katocalebnatamba@gmail.com)


lOMoAR cPSD| 41174528

acrocyanosis, sinus tachycardia. He has good pulse on both radial arteries but no pulse is
palpated on the femoral arteries. What is the likely diagnosis?
(1 correct answer)
(2 Points)
Atrial septal defect
Coarctationoftheaorta -- weak+delayed or absent femoral pulse but normal on upper arms
Transposition of the great arteries
Tetralogy of Fallot

41.A 3-month-old infant is admitted to the infectious diseases department because of diarrhea.
The pregnancy was normal, it was born at term with birth weight of 2700 gr. and is breasfed..
Laboratory results do not show evidence of an infectious cause. The infant improves and is
discharged home. Two weeks later it is admitted again because of persistent diarrhea and
weight loss. The child has normal appetite and has 5 large, foul smelling stools/day. On
admission the child weighs 4050 gr. It is pale, with distended abdomen, edema on the back of
the hands and feet. Normal respiratory and cardio-vascular examination. Abdomen is soft, liver
is palpable 3 cm below the costal margin, the spleen is not palpable. Laboratory results show
mild anemia, hypoproteinemia and metabolic alkalosis. This is most likely:
(1 correct answer)
(2 Points)
Congenital nephrotic syndrome
Cysticfibrosis (due to diarrhoea & false-smelling stools)
Lactase deficiency
Immune deficiency
42.A 2-year-old boy had fever up to 38 °C and diarrhea with some blood in the stools for 2 days.
After some improvement, on the 5th day he suddenly became pale (Hgb 70 g/l), lethargic, his
urine was red-brown in colour and edema appeared around his ankles and eyes. What is the
most likely diagnosis?
(1 correct answer)
(2 Points)
acute poststreptococcal glomerulonephritis - typically after URTI or skin infection w/GAS
hemolytic-uremicsyndrome (bloody diarrhoea due to toxins released by E.coli. The toxins later
affect the kidneys => edema & red-brown urine)
Prerenal acute renal injury because of hypovolemia
nephrotic syndrome

43.An 8-year-old child with hay fever, which presents with sneezing, clear secretions from the
nose and red eyes during the blossoming of linden trees, has been complaining of shortness of
breath during play. During such an episode there is usually prolonged expiration and wheeze
without rhonchi. Which is the most probable cause for his complaints during physical play?
(1 correct answer)
(2 Points)
Thisisduetoexercise-inducedasthma - Hx of atopy (hay fever), prolonged expiration +
wheezing + SOB + episodic

Downloaded by Kato Caleb (katocalebnatamba@gmail.com)


lOMoAR cPSD| 41174528

This is due to undiagnosed cardiomyopathy


This is due to growing pains
This is due to his blocked nose because of allergic rhinitis

44.A 6-year-old child has been diagnosed with asthma but is having frequent and severe
episodes of bronchial obstruction despite regular asthma treatment. During the last such
episode Pseudomonas aeruginosa was found in her sputum. Chest X-rays shows bilateral
bronchiectasis. Which of the following is true:
(1 correct answer)
(2 Points)
Finding Pseudomonas aeruginosa in the sputum does not affect the diagnosis or the treatment
of the child.
This is not characteristic for asthma so further tests should be performed to rule out alternative
diagnoses -- this is typical for CF
This is characteristic of asthma in childhood so skin-prick test for allergies should be performed
and her asthma therapy should be increased
The development of bronchiectasis means the child was not receiving its asthma medications
regularly

45.A 3-month- old infant is admitted to the hospital due to slow growth of the head. He was
born at 37 gestational weeks, with birth weight 2100 gr. and head circumference of 30 cm. He
developed jaundice on the second day of life. At home, the head circumference has increased
by 2 cm in 3 months and the weight by 2.5 kg. Examination reveals persistent jaundice,
hepatosplenomegaly, and lab tests- thrombocytopenia, anemia, elevated transaminases, mixed
hyperbilirubinemia. Abdominal ultrasound shows hepatomegaly with increased echogenicity of
the parenchyma. What additional tests will you suggest ?
(1 correct answer)
(2 Points)
No further test are required as this is breast milk jaundice.
Tests for thrombophilia as the child likely had prenatal stroke
Cytogenetic tests as this is likely genetic syndrome, associated with microcephaly
Congenitalinfectionsscreenofmotherandchild. - microcephaly, hepatomegaly, hemolytic
anemia, persistent jaundice -- all typical for congenital infections
46.Which of these children should be investigated for precocious puberty?
(1 correct answer)
(2 Points)
a 5-month-old girl, breast fed, with premature thelarche stage 2 - premature puberty - benign if
no other Sx of puberty
a 9-year-old girl with thelarche stage 2 and pubarche stage 2 - normal
a 4-year-oldboywithtestessizecorrespondingtosecondstageofpubertyandpubarchestage2
-- should have testicular enlargement from 9-14 yrs
All of the above
47.The dyskinetic form of cerebral palsy is characteristic for:
(1 correct answer)

Downloaded by Kato Caleb (katocalebnatamba@gmail.com)


lOMoAR cPSD| 41174528

(2 Points) Porencephalic cyst


Periventricular leucomalacia
Downˋs syndrome
Kernicterus(bilirubinencephalopathy) -- UC is deposited in basal ganglia +/- brainstem ---
dyskinesia

48.A 1-month-old infant, born at term, is consulted because of restlessness, inconsolable crying,
arching of the body backwards. This was mum’s first pregnancy and she had contractions and
was admitted to hospital because of the danger of premature labor. The child did not cry
immediately at birth. Apgar at 1 min - 4, 5th min – 7. Received oxygen for 7 days.The
neonatologist found muscle hypotonia and suppressed neonatal reflexes.The newborn had
multifocal seizures in the first 3 days of life, which were treated with Phenobarbital. On
examination there is severe axial hypotonia, the child does not react to loud noises and does
not have visual fixation. Transfontanel ultrasound shows multiple periventricular and subcortical
cysts i.e multicystic encephalomalacia. What is the cause of the child’s condition?
(1 correct answer)
(2 Points)
Congenital infection
Hypoxic-ischemicencephalopathygradeIII -- severe presentation - absent neonatal reflex,
hypotonia, seizures etc
Hypoxic-ischemic encephalopathy grade I - aka mild -- irritable/hyperaltert, normal or increased
tone + reflexes, no seizures etc
Intraventricular hemorrhage grade I

49.An-8- month old infant of Roma origin, born at term with birth weight 3000 gr, length 51 cm
and head circumference 34 cm with optimal Apgar scores At 8 months the infant is still not
sitting independently, recently started turning from back to front. From the first months of life
the relatives noticed poor facial expressions, droopy eyelids. The infant does not follow moving
objects just with the eyes but has to move the head as well. Frequently chokes while feeding.
On examination there is eyelid ptosis, weak cry and diminished pharyngeal reflex and the range
of eye movements is limited. Slightly diminished tendon reflexes. Normal reaction of pupils to
light. What is the most likely diagnosis?
(1 correct answer)
(2 Points) SMA
Neural muscular atrophy
Spinalmuscularatrophy - hypotonia + floppy baby, unable to sit independently, poor pharyngeal
muscle tone etc
Congenital myasthenia
Chronic botulism

50.Which disorder/group of disorders is part of the differential diagnosis of spastic cerebral


t palsy?
(1 correct answer)

Downloaded by Kato Caleb (katocalebnatamba@gmail.com)


lOMoAR cPSD| 41174528

(2 Points)
Duchenne /Becker progressive muscular dystrophy - LMN
Steroid-induced myopathy - LMN
Leukodystrophy - degeneration of white matter in brain (inherited)
Autistic-spectrum disorder-- no spastic Sx

51.A 2-month-old- infant from first normal pregnancy with birth weight 3150 gr, length 51 cm.
Breast fed. Referred because of frequent vomiting after feeds which started during the first
month of life and happens more often when the child is lying down.The vomiting is not forceful
and covers an area of around 7-8 cm. There is no bile in the vomits. The infants feeds “greedily”,
has 4-5 stools/daily . On examination: normal amount of subcutaneous fat, weight 4600, length
57 cm. Normal somatic and neurological examination. Small umbilical hernia. What do you
think?
(From 1 to 5 correct answers out of 5 possible answers; 1 point for each correct answer, minus 1
point for each wrong answers)
Thesearelikelyregurgitationsandanti-refluxpositionsshouldbetried -- GERD =noon-forceful,
non-bilious vomiting esp when lying down. Near normal stool freq, normal weight etc
This is congenital hypertrophic pyloric stenosis - projectile vomiting, after feeding, dehydration,
poor weight gain etc
The child has a congenital metabolic disorder
ThesymptomsareNOTduetotheumbilicalhernia
The child should promptly have contrast imaging study of the esophagus and stomach

52.Gowersˋsign may be found in?


(From 1 to 5 correct answers out of 5 possible answers; 1 point for each correct answer, minus 1
point for each wrong answers) - gowers = muscle weakness in prox muscles
Spinal muscular atrophy type III - walkers but with proximal + symmetrical muscle weakness
Duchenne/ Becker progressive muscular dystrophy
Spinal muscular atrophy type I
Cerebral palsy
Juvenileidiopathicarthritis- esp w/arthritis of hip

53.Severe inspiratory dyspnea in a newborn may be a sign of:


(From 1 to 5 correct answers out of 5 possible answers; 1 point for each correct answer, minus 1
point for each wrong answers)
Congenital pneumonia
Laryngealpolyp
Congenital atresia of the choanae - mouth breathers
Congenital lobar emphysema
Pulmonary cyst

54.Which of the following is true for Henoch-Schonlein purpura:


(From 1 to 5 correct answers out of 5 possible answers; 1 point for each correct answer, minus 1
point for each wrong answers)

Downloaded by Kato Caleb (katocalebnatamba@gmail.com)


lOMoAR cPSD| 41174528

ItisaIgА-mediatedvasculitis - infection of mucosal tracts e.g URTI -- >IgA


Always starts with renal involvement
The rarest type of vasculitis in childhood
Frequently affects brain blood vessels
Affectsthesmallbloodvessels

55.A 2-year-old boy became ill 2 days ago with runny nose and fever of 38 C. Last night he woke
up scared, restless, with barking cough and had difficulty breathing. The symptoms became
worse on crying. On examination he has hoarse voice, inspiratory dyspnea with supraclavicular
recession, loud stridor. Normal examination of the lungs. What emergency treatment is
required?
(From 1 to 5 correct answers out of 5 possible answers; 1 point for each correct answer, minus 1
point for each wrong answers)
start immediate treatment with antibiotics - most common cause of croup = parainflu aka viral
Inhaled salbutamol - laryngotracheitis = min response --- used in bronchiolitis possible
emergency tracheostomy as this is a life threatening condition -- no, unlike epiglottis = slow
progression + self limiting
oxygen therapy -- risk of hypoxia == humidified O2
nebulised adrenalin and/or corticosteroids -- croup = good response to Epi unlike epiglottitis
56.Which of the following is true for diabetes mellitus (DM)?
(From 1 to 5 correct answers out of 5 possible answers; 1 point for each correct answer, minus 1
point for each wrong answers)
Type I DM does not present with diabetic ketoacidosis -- DKA can be a new DM T1 onset
Type II DM is usually associated with obesity and children do not usually develop diabetic
ketoacidosis - true -- DMT2 = > hyperosmolar hyperglycemic state
The predominat type in childhood has monogenic inheritance - DM 1+2= polygenic -- multiple
gene defect
TypeIIDMstartsusuallyaroundpubertyorlater
Type I is treated only with diet - insulin def = exogenous insulin needed
57.An 8-year-old girl had Mycoplasma pneumonia 2 weeks ago and was treated with
Clarythromycin till 5 days ago. For the last day she has had abdominal pain, acute palor and
jaundice , fatigue and her urine has become darker. No other symptoms. On examination she is
pale and jaundiced, has wesicular breath sounds, tachycardia with heart rate 110/min,
rhythmic and clear heart sounds. Abdomen is soft, spleen is palpable at 0.5 cm below the costal
margin sith soft consistency. Succussio renalis is negative. Lab results: FBC – Hgb 70g/l; RBC- 3.2
x10/12/l, WBC – 6.8 x10/9/l, PLT- 348 x10/9/l, АSАТ – 50U; АLАТ- 40U; bilirubin – 87 μmol /l;
direct bilirubin – 10 μmol/l .What additional tests would you request to clarify the diagnosis?
(From 1 to 5 correct answers out of 5 possible answers; 1 point for each correct answer, minus 1
point for each wrong answers)
Electophoresis of hemoglobin
I LDH,freehemoglobin,haptoglobin - haptoglobin - clears free Hb in circulation
glucose-6-phosphate dehydrogenase level - X-linked recessive i.e in boys
Bone marrow biopsy - only anemia is present
Coombstest(DAT) - rule out autoimmune hemolytic anemia

Downloaded by Kato Caleb (katocalebnatamba@gmail.com)


lOMoAR cPSD| 41174528

58. A 6-month-old infant falls ill for the first time with fever and refusal to feed. There is no
vomiting and diarrhea. Normal physical examination. Laboratory results show leucocytosis with
neutrophilia, raised ESR.
A, Which of the following test would you request next to clarify the diagnosis?
(1 correct answer)
(2 Points)
Urinetest-biochemistry,microscopyandculture - UTI infants = fever,poor feeding, lethargy etc
Stool culture
Tests for immune deficiency
Pharyngeal aspirate for virology studies - neutrophilia = bacterial
59.B. If you establish leucocyturia, bacteriuria, normal pH and negative virology, what would
you perform next?
(1 correct answer)
(2 Points)
I.v urography
Abdominalultrasound - in 1st infections = US to rule out congenital urinary tract abnormalities
e.g VUR
Micturating cystourethrogram
abdominal CT
60.C. If one of the imaging studies suggests vesico-ureteral reflux(VUR), what other
investigation would you perform after the acute illness is treated , to assess the severity of
VUR.
(1 correct answer)
(2 Points)
Abdominal ultrasound
I.v urography
abdominal CT
Micturatingcystourethrogram - fluoroscopy w/real time x-ray --assess VUR
61.A 9-month-old infant had had subfebrile temperature for 1 day. The next day he had a spike
of fever up to 39.5 dg, had a generalized tonic-clonic seizure for about 5 mins, after which fell
asleep. He woke up 1 hour later and vomited twice. On examination there are no abnormalities
from the respiratory and cardio-vascular system. The childˋs consciousness is preserved, but he
is extremely irritable, the fontanelle is slightly above the level of the cranial bones and Lesage
sign is positive. No focal neurological signs.
A. Which is the likely diagnosis?
(1 correct answer)
(2 Points)
Febrile seizure
Meningitis - lasegue sign = nerve root irritation, building fontanelle = ͫICP, high grade fever etc
Brain tumor
Epilepsy
62.B. What test would you perform to confirm the diagnosis ?
(1 correct answer)
(2 Points)

Downloaded by Kato Caleb (katocalebnatamba@gmail.com)


lOMoAR cPSD| 41174528

EEG
FBC and blood biochemistry
head CT
LumbarpuncturetoobtainCSFformicroscopy,biochemistry,virologyandculture
63.C. What is your management ?
(1 correct answer)
(2 Points)
treatmentwithantibioticsandmedicationstodecreasecerebraledema
Neuro-surgical treatment
Intermittent prophylaxis with Diazepam
Antipyretics and parental reassurance

64.A 4-month-old infant with genetically proven Downˋ s syndrome has had all the required
immunizations. Her pediatrician established preserved consciousness, pale skin, respiratory rate
60/min, subcostal recession, vesicular breathing with spread crackles bilaterally , heart rate
190/min, 2-3/6 systolic murmur at the apex, liver palpable at 4 cm below the costal margin, dry
nappy and axillary temperature 36.7. Chest X-ray – increased pulmonary vascular markings,
possible cardiomegaly, cardio-thoracic index 0.61.
А. Which test will be most useful for making the diagnosis?
(1 correct answer)
(2 Points)
FBC, urea, creatinine, creatine kinase, troponin
Abdominal ultrasound
Echocardiography -- probs VSD
Sweat test
O
65.B. Currently only an ECG is available and it shows severe conduction abnormalities. This
clinical presentation is characteristic for:
(1 correct answer)
(2 Points)
Isolated atrial septal defect
Diphtheria
Staphylococcal pneumonia
Atrio-ventricularseptaldefect-- Sx of VSD+ conduction probs = ASD invol -- AVSD
66.C. What treatment would you recommend?
(1 correct answer)Immersive Reader
(2 Points)
treatment with specific immunoglobulin
Medicaltreatmentforheartfailure,followedbysurgicaltreatment
Emergency surgical treatment
oxygen therapy and anti-staphylococcal antibiotics

Bulgarian Variant 1

Downloaded by Kato Caleb (katocalebnatamba@gmail.com)


lOMoAR cPSD| 41174528

1. The period of the newborn covers exactly: (1 correct answer)

(1 point)

From the first to the 14th day

Fromthefirsttothe28thday (first 4 weeks of life)

The whole first month

From the first to the 7th day

2. In case of hypogalactia of a mother of an infant at 3 m.v. the most suitable food to add to breast milk
is: (1 correct answer)

(1 point)

Vegetable puree

Adaptedmilk(infantmilk)

Fresh cow's milk

Yogurt with 2% fat

MOCK TEST 1

1. All of the following are true for Type 2 Diabetes EXCEPT (1 correct answer): (0/1 Points)
- result of insulin resistance.
- debut mainly in puberty
- in obese patients
- oftendebutswithdiabeticketoacidosis (common for DM1)

2. Which are the Criteria for Diabetic ketoacidosis (1 correct answer): (1/1 Points)
- hyperglycemia above 20 mmol / l, acetonemia / acetonuria; pH <7.37; HCO3 <22
- hyperglycemia over 15 mmol / l, acetonemia / acetonuria; pH <7.32; HCO3 <17
- hyperglycemiaover11mmol/l,acetonemia/acetonuria;pH<7.30;HCO3<15
- hyperglycemia greater than 15 mmol / l, acetonemia / acetonuria; pH <7.33; HCO3 <20

3. The main factors determining growth during puberty are (1 correct answer):(1/1 Points)
- sex hormones
- sexhormonesandgrowthhormone
- insulin and insulin-like growth factors (Insulin only => for fetal period)
- growth hormone

Downloaded by Kato Caleb (katocalebnatamba@gmail.com)


lOMoAR cPSD| 41174528

4. The most common cause of short stature in puberty is (1 correct answer): (1/1 Points)
- Hypogonadism
- ConstitutionalDelayinGrowthandPubertalDevelopment
- Acquired hypothyroidism (Thyroid hormone doesn’t have major role in growth &
development during puberty. It mainly affects the infantile & pre-pubertal phase)
- Protein-energy malnutrition

5. The first symptom of pubertal development in girls is (1 correct answer): (1/1 Points)

- menarche

- thelarche

- pubarche

- growth spurt

6. Premature puberty in a boy is a manifestation of any pubertal sign before (1 correct answer):
(1/1 Points)
- 10 years (true for girls => if they have menarche before 10 yrs of age => precocious
puberty)
- 9years
- 8 years (true for girls=> if they develop secondary sex characteristics before 8 yrs =>
premature puberty)
- 7 years

7. Which of the following is an Emergency (1 correct answer): (1/1 Points)


- Congenital hypothyroidism
- Hashitoxicosis
- Neonatalhyperthyroidism
- All listed

8. A 1- year- old boy is with a length at the 3rd centile and a weight adequate for its length. Born
small for his gestational age. Growth rate for the first year of life- 28 cm. Normal
neuropsychological development. What is the correct management in this situation? (1 correct
answer) (2/2 Points)
- workup for celiac disease
- growth hormone deficiency tests

Downloaded by Kato Caleb (katocalebnatamba@gmail.com)


lOMoAR cPSD| 41174528

- watchfulwaiting (because everything is normal)


- Investigation of thyroid hormones

9. A 1 year 6 months old girl, with thelarche 2-3 st. with a duration of about a month. Normal
height. Obese. Normal bone age. What would you recommend: (1 correct answer) (2/2 Points)
- Workup for premature puberty
- Watchfulwaiting,mostlikelyitisaprematurethelarche (the other secondary sex
characterisitcs haven’t been mentioned=> are probably normal hence it’s probably just a
premature thelarche).
- Ultrasound to distinguish breast enlargement from subcutaneous adipose tissue
accumulation
- Stop eating chicken

10. A 15 days old child that lives in a small village 100 km away from the nearest medical institution.
The GP physician of the baby was informed of the positive result of the baby’s neonatal
screening for congenital hypothyroidism, with TSH-90 IU /L (the cut off is 15 IU / L). The child is
in a good general condition. What is the proper management: (1 correct answer) (0/2 Points)
- Referral for consultation with a specialist in a planned non-emergency manner
- Send a second filter paper with dry blood spot to the neonatal screening laboratory and
wait for the result
- Ifpossible,takeanewbloodTSHtestandstartatreatmentwithL-thyroxine
immediately.
- Referral for emergency hospitalization ( no need because the child has good general
condition).

11. A 14 year old girl, with oligomenorrhea - last regular menstruation 2 months ago, gained weight-

l 15 kg for three months. She complains of increased fatigue and drowsiness. Family history of
thyroid disease and diabetes. On examination she is apathetic, with poor facial expressions, BP
110/60; heart rate 55 / min. No significant deviations in the rest of the physical status. What will
you do: ( 0 -5 correct answers) (0/6 Points)
- Pregnancy test
- Thyroidhormonestestsandspecificthyroidautoantibodies (weight gain & slight
bradycardia could be due to hypothyroidism. Since there is a family history of thyroid
disease too, it could be inherited)
- Thyroidultrasound
- Consultation with a psychologist for depression
- ECG

Downloaded by Kato Caleb (katocalebnatamba@gmail.com)


lOMoAR cPSD| 41174528

j
12. A 25-day-old boy of normal pregnancy and delivery. Positive result obtained from the neonatal
screening for congenital adrenal hyperplasia (17OH- 250 nmol / l; at cut off <30 nmol / l). The
child is in a good general condition, weight gain of 450 g, well-formed external male genitalia
with penile hypospadias, and no palpable testicles bilaterally in the scrotum and in the inguinal
canals. What will you do? ( 0 - 5 correct answers) (6/6 Points)
- Re-examination of 17OHP, but also of cortisol and androgens
- X-rayofthekneeforboneage - advanced bone age due to precocious puberty --- bone
age <2 mo
- X-ray of the wrist for bone age - if >2 mo
- Cytogenetictest - females w/CAH = pseudohermaphroditism w/well formed male
external genitalia but w/uterus/ovaries
- A small pelvis CT to search for testicles

13. 7 - year-old girl, suffers from a runny nose, cough and fever. Diagnosed with Acute viral infection
and a treatment was initiated. On the third day the girl had abdominal pain, on which occasion
she was again examined by the GP. A history of increased thirst for about a month, polyuria and
nocturia with episodic nocturnal enuresis. From the outpatient lab tests: CBC-normal, urine:
glucose 2+ and traces of ketones. The girl was directed for emergency hospitalization. A/ What
emergency tests will you order, for the need of emergency treatment: ( 0 - 5 correct answers)
(6/9 Points)
- Capillaryorserumglucose - hyperglycemia >11.1 mmol/l in DKA
- Blood-gasanalysis (to check for metabolic acidosis- low pH <7.3)
- Radiography of the lung
- Abdominal ultrasound
- Serumelectrolytes- Anion gap met acidosis, K+ imbalance

14 Q13 B/ Given the presumed diagnosis of Question 13, where do you expect to hospitalize the child
after receiving the results / 1 correct answer /: (0/3 Points)

- Pediatric surgery; with a diagnosis of an acute abdomen syndrome


- PediatricClinic-IntensiveCareUnit;withadiagnosisofDiabeticKetoacidosis (DKA because the
patient has abdominal pain; polyuria; glycosuria together with ketones in the urine)
- Pediatric Clinic - Endocrine Sector; with a diagnosis of Diabetes mellitus, newly discovered,
without ketoacidosis

15.Q13 C/ Results obtained for hyperglycemia- 25 mmol / l and Blood gas analysis with pH -7.35 and
HCO3-19 mEq / l. What treatment will you start / 1 correct answer/: (0/3 Points)

- Rehydration and intravenous insulin


- Rehydrationandafteronehour-intravenousinsulin

Downloaded by Kato Caleb (katocalebnatamba@gmail.com)


lOMoAR cPSD| 41174528

- Rehydration and subcutaneous insulin


- Antispasmodics
- Bicarbonate, once, intravenously
- Oncohematology

16. The physiological ratio (in%) of segmented neutrophils to lymphocytes in children over 4 years

of age is as follows (1 correct answer):

(1/1 Points)

- 60:40 - Adults, >3-5 yrs N L


- 40:60 - 5 day - 2-5yrs
- 70:30
- 75:25 L N
Babies take the L
17. Neutropenia in an infant is diagnosed when the absolute neutrophil count is below (1 correct
answer): (0/1 Points)

- 250 /ml
- 500 /ml
- 1000/ml
- 2000 /ml

18. The lowest physiological hemoglobin values in healthy term infants are reached at (1 correct answer):
(1/1 Points)

- 1 month
- 2 months
- 3months- physiological anemia around 8-10 weeks
- 6 months

19.The most instructive diagnostic laboratory test for a child with normocytic normochromic anemia is:
F (1 correct answer): (1/1 Points)

- serum iron and serum ferritin; (Iron deficiency anemia is microcytic hypochromic)
- electrophoresis of hemoglobin;
- bone marrow biopsy;
- reticulocytecount. (normocytic normochromic anemia include aplastic anemia & hemolytic
anemia. In aplastic anemia, reticulocyte is decreased whereas in hemolytic anemia, it is
increased.)

20.The most common extracranial solid tumor in childhood is (1 correct answer): (1/1 Points)

- Nephroblastoma (Wilms tumor)


- Neuroblastoma

Downloaded by Kato Caleb (katocalebnatamba@gmail.com)


lOMoAR cPSD| 41174528

- Osteosarcoma
- Rhabdomyosarcoma

21.The treatment of the immune thrombocytopenia includes routine infusions of platelet

concentrates (1 correct answer):

(1/1 Points)

- True
- False (Platelet infusion is done only in severe cases. In majority of the cases, no treatment is
needed)

22.Prolonged clot retraction is specific for: (1 correct answer)(0/1 Points)

Clot retraction refers to shrinking of the blood clot & is driven by contraction; knotting & twisting of the
fibrin mesh.

- Thrombocytopenia
- Haemophilia
- Willebrand’s disease
- Glanzmann’sthrombasthenia (rare platelet disorder in which the platelets have qualitative or
quantitative deficiencies of the fibrinogen receptor αIIbβ3)

23. At a prophylactic lab test of a clinically healthy 12-year-old girl, the following were identified: Hgb
105 g / l, Er- 5.55 x1/ 120 / l, MCV 65 fl, WBC- 6.8 x10 / 9 / l, PLT-243 x10 / 9 / l. The father mentions that
years ago, in a prophylactic CBC test, he also had low hemoglobin, but he had no complaints, too, and
has not done any other tests since. What is the most likely diagnosis (1 correct answer)? (2/2 Points)

- Iron deficient anemia


- Acute haemolytic anemia
- Wilms tumor
- Thalassemiaminor (MCV is low in both iron deficiency anemia & thalassemia but the fact that
the patient is clinically healthy & her father also had low hemoglobin suggests that the anemia
could have been inherited ).

24. А 3-year-old child is admitted to the clinic with complaints of fatigue and refusal to play and to

take food. The examination revealed generalized lymphadenomegaly, pale skin and mucosa, no

jaundice. Hepatosplenomegaly is present, at 4 and 6 cm below the costal margin, both the organs

being firm, with a smooth surface. No hemorrhagic manifestations. No previous illnesses, regularly

immunized. No diarrhea or constipation, no dysuria. From the lab tests: CBC- hemoglobin 85 g / l,

erythrocytes 2.4 x 10/12 / l, leukocytes-35 x 10/9 / l, platelets-88 x 10/9 / l; DBC shows atypical

Downloaded by Kato Caleb (katocalebnatamba@gmail.com)


lOMoAR cPSD| 41174528

lymphocytes; LDH-1250 U, ASAT-46 U, ALAT-39 U. What is the most important study to clarify the

diagnosis (1 correct answer)?(0/2 Points)

- Plain abdominal X-ray


- Bonemarrowexamination (because the RBC & platelet is decreased by WBC is increased)
- Mononucleosis serology
- Coagulation status

25. Five months old boy, born on term, breastfed. So far it has been healthy, no vomiting, no diarrhea
and no acute infections. For two days, the mother noticed prolonged bleeding from the dentition sites of
the lower incisors and two bruises on his knees. The family history: the mother had a brother who often
had "swollen knees" and who died as a child from a head injury when he fell from a bicycle. What is the
most likely diagnosis (1 correct answer)? (0/2 Points)

- Acute Thrombocytopenia
- Hemophilia (the mother had a brother who had a similar condition so probably the disease was
inherited. Since, Hemophilia is X-linked recessive disorder boys are more commonly affected. His
mother must be a carrier).
- Dentition difficilis
- Brain tumor

26. You are checking a young boy of 16 years, due to a bump of the right calf, which appeared about 1
month ago and grew rapidly, with severe difficulty in stepping on the same leg. The patient reports pain
in the same area that has lasted for at least 2-3 months. The patient attributed the symptoms to a fall
while playing basketball, but also admits that the pain at that place had occurred sometime before the
fall. He is not often getting ill, does not follow special diets and has not had any serious illnesses before,
except for bronchitis and pneumonia, respectively, 3 and 5 years ago. No allergies. Grandpa - a
passionate smoker - died of lung cancer. What workup will you offer to clarify the diagnosis? ( 0 - 5
correct answers) (3/9 Points)

- CBC,DBC,ESR,LDHandAlkalinePhosphatase
- X-rayofthecalf
- Lung CT for metastases
- A test of the C1- esterase inhibitor for angioneurotic edema of the foot
- Skin swab for erysipelas microbiology

27. A 3-year-old boy, previously healthy, enters the clinic as a matter of urgency in the afternoon, due to
nausea, shortness of breath and jaundice. The grandmother reports that he did not do anything different
from the previous days, and all the family had broad bean stew for lunch 2 hours ago. On physical
examination you find a markedly pale skin and prominent jaundice, with dark urine. Afebrile. CVS -
tachycardia up to 150 beats / min, clear sounds; RS - vesicular breathing, no wheezing. Abdomen- soft on
palpation, liver 1 cm below the ribs, elastic; spleen- 3 cm below the ribs, elastic. Given the emergency,
what will you do? ( 0 - 5 correct answers) (6/9 Points)

Downloaded by Kato Caleb (katocalebnatamba@gmail.com)


lOMoAR cPSD| 41174528

- EmergencyCBC,ErythrocyteMorphologyandDAT(CoombsTest) (To check if there is anemia &


rule out autoimmune hemolytic anemia)
- Testingfortransaminases,LDHandbilirubin,andurineforhemoglobinuria (to check for liver
function since there is jaundice)
- Emergency blood transfusion (thinking of G6PD here => transfusions are rarely required for
G6PD)
- Emergency initiation of corticosteroids because the child has autoimmune hemolytic anemia
- Clarifythefamilyhistoryintermsofcongenitalhemolyticanemia

28.A 3-year-old child with a 2-week history of abdominal pain presented with a palpable abdominal
mass. The child was otherwise healthy and the parents reported no other complaints. An abdominal CT
scan showed this polar renal tumor with some calcifications extending beyond the renal capsule to the
adrenal gland. There was no thrombus in the renal vein and the chest CT scan was normal. A/ Which of
the following would be the more likely diagnosis? (1 correct answer) (2/2 Points)

- Neuroblastoma (abdo pain with palpable abdominal mass is typical for neuroblastoma)
- Rhabdomyosarcoma
- Wilms tumor
- Non-Hodgkin lymphoma

29 Q28 B/ What would be your next diagnostic step? (1 correct answer) (2/2 Points)

- Referthechildforbiopsy. (to confirm the diagnosis of tumor)


- Start preoperative chemotherapy without further testing.
- Evaluate for mycobacterial infection.
- Perform bone marrow biopsy.

30. This peripheral blood smear is from a 5-day-old child with anemia (hemoglobin 90 g/l) and jaundice
(indirect bilirubin of 175 mmol/l). The Coombs test is negative and there is no other clinically evident
sign. A/ What do you see on the slide? (1 correct answer) (3/3 Points)

- Macrocytosis
- Fragmentocytes (a.k.a. Schistocytes)
- Elliptocytes
- Spherocytes

31.Q30 B/ What is your probable diagnosis? (1 correct answer) (3/3 Points)

- Thalassaemia beta
- ABO incompatibility
- Hereditaryspherocytosis
- Aplastic anemia

32.Q30 C\ What confirmatory test would you order? (1 correct answer) (3/3 Points)

Downloaded by Kato Caleb (katocalebnatamba@gmail.com)


lOMoAR cPSD| 41174528

- osmoticfragility
- chromosome fragility
- hemoglobin electrophoresis
- Repeat the Coombs test

33.The 2-year-old boy in this photo exhibits a typical presenting sign of which conditions? ( 0 - 4 correct
answers) (2/9 Points)
unilateral
- Orbital rhabdomyosarcoma (swelling around the eyes)
- Childabuseandfractureofthecranialbase(youwillseeraccooneyes)
- Neuroblastoma(youwillseeraccooneyes)
- Acute myeloid (M3 subtype) leukemia stated
Mock Test 2

1.Craniotabes is a symptom of: (1 out of 4 correct answer) (1/1 Points)

Vit A deficiency

VitDdeficiency (it is an early sign for rickets)

Hypervitaminosis D

Hypervitaminosis A

2. A complex of VSD, overriding of the aorta with respect to the ventricular septum, subpulmonary
stenosis causing right ventricular outflow tract obstruction and right ventricular hypertrophy are signs of:
(1 out of 4 correct answers) (1/1 Points)

TetralogyofFallot

Persistent ductus arteriosus

Transposition of the great arteries

Adult-type coarctation of the aorta

3.Puberty starts between 7-9 yr in males and 10-14 yr in females. (1 out of 4 correct answers)

(0/1 Points)

- True
- False (Girls => 8-13 years & boys => 9-15 years)

4. Microcytic,hypochromic anaemia(low MCV and MCH) with low serum ferritin are the diagnostic

Downloaded by Kato Caleb (katocalebnatamba@gmail.com)


lOMoAR cPSD| 41174528

clue to: (1 out of 4 correct answers)

(1/1 Points)

Β-thalassaemia trait

Irondeficiencyanaemia

Glucose-6-phosphate dehydrogenase deficiency

Fanconia anaemia

5.Clinical features of hypothyroidism are: (1 out of 4 correct answers) (1/1 Points)

,cold,3
- I Pale2 mottled dry skin4 , hoarse cry6
, large tongue5 , delayed development
- Pale,cold, mottled dry skin, large tongue,hoarse cry, tachycardia (tachycardia =>
hyperthyroidism)
- Pale,cold, mottled dry skin,large tongue, hoarse cry, loose stools (loose stools => In
hyperthyroidism. In hypothyroidism we have constipation)
- Pale, cold, mottled dry skin, large tongue,hoarse cry, hyperthermia (hyperthermia=>
hyperthyroidism)

6.Breast feeding is contraindicated in: (1 out of 4 correct answers) (1/1 Points)

Mother'sHIVinfection

Mother's upper respiratory tract infection

Mother's obesity

Low calories of mother's milk

7.Bronchiolitis goes with: (1 out of 4 correct answers) (0/1 Points)

Inspiratory dyspnea (In foreign body obstruction??)

Expiratorydyspnoea

Unilateral pleuritis

Cardiomegaly

8.A Febrile seizure may occur at the age of: (1 out of 4 correct answers) (1/1 Points)

3mo-6yr (Typically occurs between 6 months to 6 year) - atypical = <6 mo

3 mo - 3 yr

1mo - 1yr

Downloaded by Kato Caleb (katocalebnatamba@gmail.com)


lOMoAR cPSD| 41174528

Any age

9.A child can run and jump, draw, understand two joined commands at: (1 out of 4 correct answers) (0/1
Points)

6 months

12 months

15 months

30months (can run around 18 months; jump at 2 years; copies a circle & a cross around 3 years; 25% of
the speech understandable around 2 years & 50% of the speech understandable around 3 years)

10. In acute pyelonephritis there is: (1 out of 4 correct answers) (1/1 Points)

Haematuria

Bacteriuriaandleukocyturia

Proteinuria

Hematuria and proteinuria

11. At what age does the infant start to pronounce the first meaningful words? (1 out of 4 correct
answers) (1/1 Points)

at around 7-8 months of age

ataround11-12monthsofage (around 12 months=> few words + mama/dada)

at around 21 months of age

at around 36 months of age

12.The most common cause of acute epiglottitis is: (1 out of 4 correct answers) (0/1 Points)

Influenzae virus

HaemophilusInfluenzae

type B Streptococcus beta-hemolytic group A (S. pyogenes) (Causative agent of tonsillitis)

Epstein-Barr virus (Causative agent of tonsillitis)

13.Which disease are the values of sweat test above 60 mEq /l evidence for: (1 out of 4 correct answers)
(1/1 Points)

Tuberculosis

Downloaded by Kato Caleb (katocalebnatamba@gmail.com)


lOMoAR cPSD| 41174528

Cysticfibrosis

Aspergillosis

Idiopathic pulmonary hemosiderosis

14.Which substance is used for measuring immunity against Mycobacterium tuberculosis by Mantoux
test ? (1 out of 4 correct answers) (0/1 Points)

Bacillus Calmett -Guerin (BCG)

Purifiedproteinderivative(PPD)

Interferon gamma release assays like T-spot

All of the above

15.Which of the following symptoms are characteristic of respiratory failure? (1 out of 4 correct
answers) (1/1 Points)
persistent dry cough
tachydyspnoea
Bradicardia
chest deformity of the pectus carinatus type

16.Atelectasis of the right upper lobe in a young infant with respiratory failure is typically due
to: (1 out of 4 correct answers) (0/1 Points)
Congenital lobar emphysema
Tuberculosis
Foodaspiration - infant = supine position --- upper lobe affected
Meconium aspiration

17.Reduced prothrombin time (%) is found in: (1 out of 4 correct answers) (1/1 Points) NB - PT
in % is inversely proportional to PT in seconds
DICsyndrome,severevit.Kdeficiency - reduced PT in % (Increased PT in seconds)
Thrombocytopenia - normal PT
Glanzmann thrombasthenia
Thrombophilic state - increased PT in % (reduced PT in s)

18.Which antidote is used in organophosphate poisoning: (1 out of 4 correct answers) (1/1


Points)
Prostigmin
V it. K
Atropine
Ahypnon

Downloaded by Kato Caleb (katocalebnatamba@gmail.com)


lOMoAR cPSD| 41174528

19.Werlhof's thrombocytopenic purpura (a.k.a. ITP) presents with: (1 out of 4 correct answers)
(1/1 Points)
Jaundice
PetechiaeandecchymosesHaemarthrosis - only superficial bleeding
Lymphadenomegaly

20.The most likely diagnosis of a child with acute onset of pallor, oliguria, azotemia and
petechiae without hematuria and arterial hypertension is: (1 out of 4 correct answers) (1/1
Points)
Acute hemolytic anemia
Acute glomerulonephritis
Hemolytic-uremicsyndrome (here we have destruction of blood platelets=> petechiae & kidney
failure leading to build up of nitrogen containing compounds in the blood; oliguria & arterial
hypertension).
Both a/ and b/ are correct

21.The normal urine output of an infant is: (1 out of 4 correct answers)


(1/1 Points)
0.5ml/kg/hr
2ml/kg/hr (1-3 ml/kg/hr)
5ml/kg/hr
10ml/kg/hr

22.The key hormone which determines the height after the the first three years of life is: (1 out
of 4 correct answers) (1/1 Points)
Insulin
Growthhormone (3 yrs-puberty => GH & Thyroid hormone)
Insulin and thyroid hormones (Only Insulin => Fetal period)
Sex hormones (During puberty together with GH)

23.Compared to cows' milk, breast milk contains more proteins, less carbohydrates and more
vit.D (1 out of 4 correct answers) (0/1 Points)
True
False (Cow’s milk contains more proteins (3.5g in Cow’s milk & 1.2g in Breast milk). Breast milk
is deficient in vitamin D)

24.Thalassemia is inherited in : (1 out of 4 correct answers) (1/1 Points)


Autosomal dominant mode
Autosomalrecessivemode
X-linked mode
Codominant mode

25.The most common cause of urinary tract infections in childhood is: (1 out of 4 correct
answers) (1/1 Points)

Downloaded by Kato Caleb (katocalebnatamba@gmail.com)


lOMoAR cPSD| 41174528

E.coli (90% of the cases)


Proteus mirabilis
Enterococcus
Pseudomonas aeruginosa

26.Periventricular leukomalacia in preterms is due to? (1 out of 4 correct answers) (1/1 Points)
Genetic brain malformation
Congenital infection
Hypoxiaand/orischemiaofthebrain
Brain hemorrhage

27.Which tool gives a functional assessment for perinatal asphyxia? (1 out of 4 correct answers)
(0/1 Points)
Apgarscore
Glasgow coma scale (used to assess for consciousness)
Gross motor function
measurement scale Ballard score (Based on baby’s physical & neuromuscular maturity)

28.Jaundice in the newborn is characterized by the following features: (1 out of 4 correct


answers) (0/1 Points)
Inhemolyticjaundice,flavinicterisobserved
Indirect (unconjugated) bilirubin is found in the urine (Unconjugated bilirubin is not water soluble
so it is not excreted in the urine)
Jaundice in the second week after birth is most likely due to iso-immune hemolytic anemia
(Usually due to ABO incompatibility or G6PD deficiency. Baby’s immune system is not very well
developed by 2 weeks to have iso or autoimmune hemolytic anemia)
Brain damage is due to direct bilirubin. (due to unconjugated bilirubin)

29.In the first months after birth, constipation, large abdomen and malnutrition are characteristic
t of: (1 out of 4 correct answers) (1/1 Points)
Celiac disease (occurs usually after weaning; so typically around 6 months when gluten
containing food is introduced)
Intolerance to cow's milk proteins (would have diarrhoea)
Hirschprung'sdisease
Duodenal stenosis (They would have bilious vomiting from day 1)

30.Which disease is caused by maldigestion of fats and proteins in general food and is
characterized by increased appetite and bulky fatty and very foul-smelling feces? (1 out of 4
correct answers)
(0/1 Points)
Celiac disease
Cysticfibrosis
Exudative enteropathy
Lactase deficiency

Downloaded by Kato Caleb (katocalebnatamba@gmail.com)


lOMoAR cPSD| 41174528

31.Ventricular septal defect is characterized by: (1 out of 4 correct answers) (1/1 Points)
pulmonarycongestionandfrequentpneumonias
hypoxemic spells (characteristic for tetralogy of fallot)
small boot-shaped heart (characteristic for tetralogy of fallot)
ECG conduction disorders - ASD primum

32.What is true for infantile mortality rate? (1 out of 4 correct answers) (0/1 Points)
It reflects the number of deaths in the ages from 0 to 17 years (Between 0-14 years)
In the developed world, it is between 50 and 100 per 1000.
Inthedevelopedworld,itisdeterminedmainlybycongenitalmalformations,neonataldiseases and
pneumoniasandotherrespiratorydiseases
All of the above is correct

33.What is typical for systemic lupus erythematodes?: (1 out of 4 correct answers) (1/1 Points)
malarrash
nuchal rigidity (true for meningitis)
positive blood culture
allergies are common

34.Arterial hypertension is a majоr clinical sign in: (1 out of 4 correct answers)


(1/1 Points)
Rhabdomyosarcoma
Pheochromocytoma (Produce catecholamines => vasoconstriction=> Increase in BP)
Acute pyelonephritis
Ventricular septal defect

35.Which of the following is typical for tetralogy of Fallot: (1 out of 4 correct answers)
(1/1 Points)
bootshapedheart
left-to-right shunt (right to left shunt)
absent cyanosis
pulmonary valvular insufficiency

36.An infant at 3 months of age becomes ill with fever, decreased appetite and single vomiting.
The mother reports an altered urine odor. Blood tests performed by the family doctor showed
evidence of mild anemia, leukocytosis and increased ESR, as well as an increase in C-reactive
protein. What additional tests do you suggest? (1 out of 4 correct answers) (2/2 Points)
Urineandurineculture (due to altered urine odour)
Lung graph
Fecal culture
Renal scintigraphy

Downloaded by Kato Caleb (katocalebnatamba@gmail.com)


lOMoAR cPSD| 41174528

37.Newborn is born by cesarean section at 37 g.w with birth weight 4800 g, height - 51 cm,
head circumference - 36 cm. Apgar - 8; 9. Difficult postpartum adaptation, placed on oxygen for
two hours, then given for breastfeeding to the mother. On the second day, the mother registered
irritability, backwards posturing and trembling of the limbs. The neonatologist observed focal
clonic seizures of the left hand, clonus of the feet, chin tremor, oral automatisms. Hgb - 160 g / l,
RBC - 5.5x1012/l, WBC - 12.2x109/l, Tr - 410x109/l, ESR - 12 mm. CRP - 0.1 g/l, blood sugar -
1.2 mmol/l, calcium - 2.16 mmol/l, ionized calcium - 1.2 mmo/l. What is the cause of the
seizures? (1
out of 4 correct answers) (0/2 Points)
Neonatal sepsis (In sepsis we would have other systemic symptoms as well such as fever;
increased BP etc)
Neonatalhypoglycaemia
Cerebral hemorrhage
Hypoxic-ischemic encephalopathy

38.A 5-year-old child is admitted to the emergency department in an extremely serious condition
with impaired consciousness (Glasgow coma scale - 6-7 points), pronounced pallor of the skin
and mucous membranes, blood pressure - 60/20, pulse - 160/minute, weak pulsations on the
peripheral arteries, with recoloration time of 6 sec. The same day he was riding a bicycle with
friends, and according to the children, he hit his abdomen in the frame of the bicycle during one
abrupt stop. He did not lose consciousness, but later started complaining of dull pain in his
abdomen and left shoulder. What is the probable cause of the condition? (1 out of 4 correct
answers) (2/2 Points)
most common abdominal injury
Ischemic heart stroke
Ruptureofthespleen y
Intracranial hemorrhage (there is no hit to the head)
Acute pancreatitis

39.A 10-year-old girl is complaining of severe abdominal pain in the last days. From this
morning, the pain intensified and she began to vomit. She was examined by a pediatric
gastroenterologist, who revealed by ultrasound an infiltrative process in the area of the terminal
ileum. Lab data revealed mild normocytic anemia and thrombocytopenia, but very high serum
lactate dehydrogenase - 1500 E/L; chest X -ray was normal. What is the most likely cause of
these complaints? (1 out of 4 correct answers) (0/2 Points)
Fecaloma
Non-Hodgkinlymphoma (due to anemia & thrombocytopenia so there is probably bone marrow
infiltration too. The high lactate dehydrogenase suggests a destructive process).
Phlegmonous appendicitis
Crohn's disease

40.A 14-year-old girl is diagnosed with cystic fibrosis at age 2 years, but is not checked regularly
in the last 3-4 years. Now she is admitted to the clinic for pneumonia. abdominal wall,
Hepatomegaly - liver at 3 cm below costal margin, firm, with sharp edge and uneven surface.
Laboratory data were hypoproteinemia, hypoalbuminemia, elevated transaminases, prolonged

Downloaded by Kato Caleb (katocalebnatamba@gmail.com)


lOMoAR cPSD| 41174528

prothrombin time, direct hyperbilirubinemia. Which extrapulmonary complication of cystic fibrosis


you consider most likely? (1 out of 4 correct answers) (0/2 Points)
Bile sludge syndrome
Livercirrhosis ( hypoproteinemia; hypoalbuminemia; increased transaminases & increase in
direct bilirubin => suggest liver pathology) + firm liver w/sharp edge + uneven surface
Chronic viral hepatitis C.
Non-alcoholic steatosis of the liver

41.A 6-month-old infant that fell asleep in full health wakes at night with screaming, bents his
legs to the abdomen, refuses to eat being restless. Very strong colic-like pain was observed,
combined with pallor and repeated biliary vomiting.On examination: impaired general condition,
pale skin, sunken eyes. The abdomen is distended and elongated, sausage-like structure is
found by palpation. Intestinal peristalsis is enhanced, but there is no actual passage. Rectal
examination reveals bloody mucus on the fingertips. What is the diagnosis and what is the next
management step? (1 out of 4 correct answers) (2/2 Points)
Ileocecalintussusception,pneumocolonoscopy (due to presence of sausage-like structure on
palpation & biliary vomiting)
Mekel's diverticulum, scintigraphy, surgical treatment
Infectious enterocolitis, fecal culture to find etiology
Allergy to cow's milk protein, specific diet

42.A 1year 7 months old boy from normal pregnancy and delivery does not respond when called
by name and says no distinct words. He is able to walk, scribble with pen and feed himself. He
points to the objects he wants.He seeks comfort in his mother’s lap looking at her. What is the
most appropriate initial investigation? (1 out of 4 correct answers)
(0/2 Points)
Hearingtest (because he has normal development. Maybe the reason why he is not responding
is due to hearing defect)
Assessment for autism spectrum disorder
Reassurance of the parents
Perform a full developmental assessment

43.A 2-year-old boy became ill with gastroenteritis presenting with repeated vomiting during the
day and watery stools, fever up to 38 degrees. A few hours later he became lethargic, sleepy
and the mother noticed that he had not urinated for the last 4-5 hours. Heart rate 160 / min.
Blood pressure 85/50. What is the most likely diagnosis of the child's condition? (1 out of 4
correct answers) (2/2 Points)
Hemolytic-uremic syndrome
Dehydration
Acute meningitis
Acute glomerulonephritis

44.This 3-month-old infant was brought to a doctor for constipation. She was born from a
seventh pregnancy in a family with low social status. Born at home. She has not been examined

Downloaded by Kato Caleb (katocalebnatamba@gmail.com)


lOMoAR cPSD| 41174528

by a doctor so far. She defecated every 3-4 days. Good appetite and weight gain. She is said to
be very calm and sleeps most of the time. Besides the appearance seen in the photo, the girl
has dry skin,generalized muscular hypotension, no hepatosplenomegaly, heart rate 72 / min,
clear heart sounds, no murmurs. She lags behind in her neuropsychological development. What
is the probable diagnosis? (1 out of 4 correct answers)
(0/2 Points)
Mucopolysaccharidosis
Hirschsprung's disease
Congenitalhypothyroidism (dry skin & delay in neuropsychological development suggests
hypothyroidism) + lethargy, very calm + sleeps most of the time, muscular hypotension
Down syndrome

45.A 14 year old girl with normal development up to now suffers from oligomenorrhea - last
regular menstruation 2 months ago, gained weight- 15 kg for three months. She complains of
increased fatigue and drowsiness. Family history of thyroid disease and diabetes. On
examination she is apathetic, with poor facial expressions, BP 110/60; heart rate 55 / min. No
significant deviations in the rest of the physical status. What will you do first? (1 out of 4 correct
answers) (2/2 Points)
CheckTSHandfT4 (increased weight; bradycardia & history of thyroid disease suggests that
the patient may have hypothyroidism)
Check DHEA-S
perform heart ultrasound
send for psychiatric consultation

46.A few days after an upper respiratory tract infection, a 3 years old boy goes pale, lethargic
and with abdominal pain and dark red urine. On examination – heart rate 140/min, liver 2 cm,
spleen –0,5 cm bellow the costal margin. Hb. 60 gr/L. Normal leucocyte and platelet counts.
High reticulocyte count - 5%. Urine is urobilnogen positive, bilirubin negative, with erythrocytes
2/ml.What is this disease? (1 out of 4 correct answers) (0/2 Points)
Acutehemolyticanemia (due to increased reticulocytes. It could be triggered by an infection).
Acute hepatitis
Acute glomerulonephritis
Wilms tumor

I ays
w 47.A four-year-old child was sent to grandmother's village to be isolated from chickenpox. It was
look at healthy when it went out to play in the yard in the sun. When he returned home, the
ne Ux grandmother noticed that his speech is inadequate and blurred, the child staggers as if drunk
and has a flushed face. The doctor found a temperature of 37.8 °C, tachycardia 125 / min.,
blood pressure 120/80, large pupils and dry tongue and mucous membranes of the mouth.
Which condition explains these symptoms best? (1 out of 4 correct answers) (0/2 Points)
Onset of chickenpox, possibly complicated by encephalitis
Heatshock (due to flushed skin; tachycardia; dry mouth; altered speech & large dilated pupils)
Atropine poisoning
Alcohol intoxication

Downloaded by Kato Caleb (katocalebnatamba@gmail.com)


lOMoAR cPSD| 41174528

48. An 18-month-old boy has had a fever up to 39 C for the past 7 days. His GP prescribed
Cefuroxime for upper respiratory tract infection with sore throat 5 days ago but there is no
improvement so far. Three days ago he developed a rash and his parents noticed a swelling on
the right side of his neck. On examination you also establish bilateral conjunctivitis, red and
cracked lips, erythema and oedema of his palms and soles as well as intense erythema with
desquamation in the nappy area. There is only unilateral neck lymphadenopathy. No
hepatosplenomegaly.. Normal blood counts and transaminases Which of the following tests will
help you most to make the diagnosis? (1 out of 4 correct answers) (0/2 Points)
echocardiographyofthecoronaries (to rule out Kawasaki disease => bilateral conjunctivitis;
erythema & oedema of the palms & soles; neck lymphadenopathy all point towards Kawasaki
disease)
conjunctival swab for culture
computed tomography of neck
PCR for enteroviruses in stool

49.A baby girl at 4 months is admitted because of diminution of consciousness and right-sided
clonic seizures after a mild head injury. From the status - left oculomotor nerve palsy (ptosis of
the left eyelid and dilated left pupil), hypotonicity with reduced active movements of the right
limbs. Eye fundi - retinal hemorrhages in both eyes. Computed tomography of the head - fresh
and old subdural hematomas, midline shift to the right. What is the most probable cause of
intracranial bleeding? (1 out of 4 correct answers) (2/2 Points)
Cerebral micropolygyria (neuronal migration disorder where there is development of several
small microgyri causing intellectual disability).
Shakenbabysyndrome (there is presence of fresh & old subdural hematomas=> could be due
to persistent abuse).
Brain tumor
Acute traumatic brain injury after the mild head injury

50.A five-year-old boy has a normal premorbid history. For 2 weeks he has been complaining of
night pains in his legs - the bones of his calves, general malaise, reluctance to play. On
examination: afebrile, pale skin and mucous membranes, no rash, Normal respiratory and
cardiovascular status. Liver 3.5 cm; spleen 3 cm below the costal arch with a elastic
consistency. No other pathology. HGB- 73 g / l, WBC- 2.3x109 / l, PLT- 62x109 / l. Which is the
most likely diagnosis listed? (1 out of 4 correct answers) (0/2 Points)
Growth pain
Acuteleukemia (night pain suggests a malignant process; anemia; leucopenia &
thrombocytopenia because the blast cells in leukemia infiltrate the bone marrow leading to
decreased production of other cell lines).
Osteosarcoma
Systemic form of juvenile idiopathic arthritis

51.In the well-baby clinic, a 7 months old infant from first normal pregnancy with birth with a
weight 3500 g, height 51 cm and head circumference 35 cm is examined. It currently weighs 8,0

Downloaded by Kato Caleb (katocalebnatamba@gmail.com)


lOMoAR cPSD| 41174528

kg, is 66cm long and has a head circumference of 39 cm. When assessing its
neuropsychological development, it is established that the child raises his head in a prone
position, has good leg support, does not roll over and does not sit alone, turns his head in the
direction of the sound, reaches for a toy and holds it for a short time, smiles, coos rarely, does
not pronounce words or syllables. Which of the following statements is true? (From 1 to 5
correct answers out of 5 possible answers; 1 point for each correct answer, minus 1 point for
each wrong answers)
There is a delay in weight and height
There is an acceleration in head growth.
Thereisadelayinmotordevelopment (baby should roll over by 4 months of age & sit by 6
months)
Thereisadelayinspeechdevelopment (at 6 months baby should be babbling & initiating
sounds)
There is an acceleration in motor development.

52.A 2-year-old boy who fell ill two days ago with a runny nose and a temperature of up to 38
degrees. That night he woke up frightened, restless, coughing with a hoarse, barking cough. His
parents noticed difficulty breathing. Complaints intensified when crying. Examination revealed
blue lips, hoarse voice, inspiratory dyspnea with jugular retraction, rough stridor. No
abnormalities in lung status. What treatment needs to be taken? (From 1 to 5 correct answers
out of 5 possible answers; 1 point for each correct answer, minus 1 point for each wrong
answers)
Start antibiotic treatment immediately (barking cough & inspiratory dyspnea with stridor
suggests croup. The causative agent for croup is mainly virus such as parainfluenza; influenza
& adenovirus so antibiotic is of no use)
Administration of corticosteroid by inhalation or intravenously (For croup, you would initially give
corticosteroids, if it doesn’t get better => epinephrine).
Oxygensupply (to treat cyanosis)
Administration of ventolin by inhalation
Tracheotomy immediately

53.A full-term newborn develops jaundice within the first 24 hours. The newborn doesn’t wake
up for feeding. Jaundice is mainly due to indirect bilirubin. Anemia, mild hepatosplenomegaly
and the presence of erythroblasts in the peripheral blood are found. Which of the following
statements is true? (From 1 to 5 correct answers out of 5 possible answers; 1 point for each
correct answer, minus 1 point for each wrong answers)
ThisjaundiceismostcommoninnewbornswithbloodtypeRh+whosemothersareRh- (Rhesus
incompatibility usually presents within 1st 24 hrs of birth).
No treatment is needed; just monitor bilirubin levels
Gastric lavage is necessary
Phototherapyshouldbeapplied (Apply phototherapy first, if it doesn’t work or if there is risk of
kernicterus => exchange transfusion)
In case of bil. > 150 mmol/l an exchange transfusion should be undertaken (exchange
transfusion if bilirubin is > 300 mmol/l)

Downloaded by Kato Caleb (katocalebnatamba@gmail.com)


lOMoAR cPSD| 41174528

54.Infant at 4 months of second normal pregnancy, born full term weighing 3100 g. Older sister
of 5 years, with bronchial asthma. On a diet with formula and mother's milk since age 2 months.
With regurgitation from birth. In the last month with unsatisfactory weight gain and an increase
in the number of bowel movements (9 times a day), which became more loose, with fresh blood
in some of them. Upon admission, the child weighed 5100 g, with no signs of dehydration, with
erythemo-exudative rash on the face and crustae lactea on the scalp (see photo). No
abnormalities in lung and heart status. Abdomen - without organomegaly. Hemoglobin - 92 g / l,
MCV -70 fl, leukocytes - 10 x 10 ^ 9 / l, platelets - 150 x 10 ^ 9 / l, normal coagulation tests,
transaminases, urea, creatinine, uric acid, serum zinc, total protein and albumin. The mother
had scars on her right breast 2 weeks ago. What do you think should be undertaken? (From 1 to
5 correct answers out of 5 possible answers; 1 point for each correct answer, minus 1 point for
each wrong answers)
Carryingoutaskin-pricktestwithcow'sandformulamilk (to see if the child is allergic to cow’s milk
protein in the formula milk that was introduced at 2 months).
Replacementoftheformulawithproteinhydrolysate ( protein hydrolysate is more digestible &
less immune provoking so better for a child with CMPA).
Referral of the child to a surgeon, most likely for intussusception
Switch to lactose-free milk, as there is lactase deficiency.
The diagnosis is acrodermatitis enteropathica. Zinc therapy should be initiated

55.A one month old boy is born with the defect, shown in the picture below. Discharged from the
Maternity Ward after consultation with a neurosurgeon for surgical treatment after reaching a
body weight of 4000 grams, and until then regular dressings of the defect are made. The child
was hospitalized because of two-days fever, refusal of food, vomiting and sleepiness. The lower
limbs were limp. The anus was patent and urine was dripping from the penis. What urgent
examinations corresponding to the child's condition would you order? (From 1 to 5 correct
answers out of 5 possible answers; 1 point for each correct answer, minus 1 point for each
wrong answers)
Bloodandurinecultures (to check for any signs of infection)
Transfontanelandabdominalultrasonography
Radiography of the feet
Abdominal contrast enhanced magnetic resonance imaging
Toxicological screening

56.A 9-year-old girl has been referred because her weight centile has increased in the last 10
months. She is an excellent student and has no other complaints. On examination there is no
acne, striae or hirsutism. Her weight is in the 97th centile and her height is between 75th and
90th centile. Heart rate of 95/min. Blood pressure 135/95. Normal eye fundi. What is the most
adequate course of action? (From 1 to 5 correct answers out of 5 possible answers; 1 point for
each correct answer, minus 1 point for each wrong answers)
adviceabouthealthydietandincreasedphysicalactivity
further tests to rule out Cushing's syndrome (she has no striae & other symptoms that would be
associated with Cushing’s)

Downloaded by Kato Caleb (katocalebnatamba@gmail.com)


lOMoAR cPSD| 41174528

further tests to rule out hypothyroidism (no other symptoms suggestive of hypothyroidism)
adviceabouttheincreasedriskofabnormallipidlevelandhighbloodpressure
evaluation for brain tumor

57.A 14month-old child, vaccinated for measles, mumps and rubella 1 week ago, was brought
to hospital because he had started stepping on his left toe since yesterday. Normal history of
pregnancy and birth. He can walk held by one hand since age 12 months. Speaks 2-3 words.
On examination- looks well, no fever, normal lung, cardiac, abdominal and oral status. Normal
muscle tone and reflexes. Only limited and probably painful abduction of the left leg was found.
A\ Which is the most likely diagnosis? (1 out of 4 correct answers ) (2/2 Points)
Hemiparetic cerebral palsy (normal muscle tone & reflexes & in case of hemiparetic cerebral
palsy, one complete side of the body would be paretic)
Reactivearthritis
Septic arthritis ( false because there is no signs of inflammation & fever)
Congenital dysplasia of the hip joint

58.Q57 B\ Which test is the most informative in this case? (1 out of 4 correct answers) (2/2
Points)
Blood culture (Usually culture of synovial fluid is performed)
Radiography of the left knee
X-ray of the skull
C-reactiveprotein (ESR & CRP are increased in reactive arthritis)

59Q57 C\ If all of the above tests are normal, what is the most correct treatment ? (1 out of 4
correct answers )
(2/2 Points)
Start antibiotic treatment.
StarttreatmentwithibuprofenorotherNSAIDs. (for pain)
Start physiotherapy.
Refer the child for plaster immobilization.

60.A 20 days old preterm neonate was admitted to hospital because of irritabilty and 2-3 spells
of right hand trembling that last for several seconds since 6-7 hours. It was alert, sucking well.
Fontanel was 2x2cm above the cranium, but not tense. Breathing rate was 55/min, heart rate
was 195/min, arterial pressure 55/30 mmHg. Weak Robinson reflex and reduced active

compensatory
increase in MR
0
movements were found for the right side. Emergency CT showed the following picture: A\ What
is the problem? (1 out of 4 correct answers ) (2/2 Points)
Brain edema
Intracranial hemorrhage
Ischemic stroke
Brain malformation

61. Laboratory test on admission were: Hemoglobin 99g/l, leucocytes 19x109/l, neutrophils
35%, eosinophils 3%, monocytes 6%, lymphocytes 56%, platelets 160x109/l, ESR 20mm.

Downloaded by Kato Caleb (katocalebnatamba@gmail.com)


lOMoAR cPSD| 41174528

Prothrombin time measurement showed no coagulation. D-dimers and blood gas analysis were
normal. What is the diagnosis? (1 out of 4 correct answers ) (2/2 Points)
Brain abscess
Hemorrhagicdiseaseofthenewborn (maybe due to presence of intracranial hemorrhage??)
Sepsis
Inborn error of metabolism - pyruvate dehydrogenase deficiency

62.Which of the following treatment strategies is the most urgent and mandatory? (1 out of 4
correct answers ) (0/2 Points)
ApplyVit.Kandfreshfrozenplasma.
Firstly,startdiureticstoreduceintracranialpressure. (raised fontanelle suggests there is
increased ICP)
Intubate and start hyperventilation because that is the best way to lower the intracranial
pressure.
Refer immediately for neurosurgery, no time for conservative treatment

63.A one-year-old child coughed vigo rously while ea ting chicken soup and vomited once . A fte r a
few minutes he calmed down . Two days la te r the cough reappea red . The examina tion revealed
a satisfactory general condition, spastic cough, hype rresonan t pe rcussion tone in the righ t lung
with softer than normal vesicular breathing and mild wheezing . Pa raclinica l s tudies revealed
moderate leukocytosis and mildly eleva ted ESR . The g randmo the r su ffe rs from b ronchia l
asthma. A\ What is the probable diagnosis? (1 ou t o f 4 co rrec t answe rs )(0 /2 Poin ts )
Right-sided pneumonia (dull sound on percussion )
For eign body in main rb onchus( duet ot he hist ro y &t he hype
rr esonant sound is localized int r he
t igh lung
)
Bronchial asthma (the hyperresonant sound would be hea rd all ove r the lungs )
Pulmonary edema due to ven tricula r sep ta l de fec t

64.Q63 B\ The same child underwent radiography, which showed the following finding. Where is
the primary injury? (1 out of 4 correct answers ) (0/2 Point s)

In the left lung

Downloaded by Kato Caleb (katocalebnatamba@gmail.com)


lOMoAR cPSD| 41174528

Intherightlung (Increased intercostal space & radiolucent on the right => due to hyperinflation.
In foreign body obstruction initially we have hyperinflation then atelectasis)
In the heart
In the diaphragm

65.Q63 C\ What is the correct behavior? (1 out of 4 correct answers ) (0/2 Points)
Salbutamol by inhalation
Corticosteroids intravenously
Bronchoscopy
Intubation and artificial lung ventilation

Extra Questions no wit D in breastmilk

y
1. Breast milk contains more proteins, less carbohydrates, more vitamin D compared to
cow's milk
a) True
b) False
the
a ram
L y2. 1 year old child has the following blood results: leucocytes 12,000, neutrophils 32%,
lymphocytes 60%, eosinophils 3%, basophils 1%, monocytes 4%
a) Thesearenormalbloodresults
b) There is leucocytosis with neutropenia
c) There is leucocytosis with monocytosis
d) There is leucocytosis

3. A child with lacunar tonsillitis who develops a non-itchy rash after the first dose of
Ampicillin is
a) Allergic to Ampicillin, with lacunar tonsillitis
b) A child with infectious mononucleosis and allergy to Ampicillin
c) Achildwithinfectiousmononucleosis
d) Both b/ and c/ are correct

4. The commonest cause for recurrent bronchial obstruction in infants is


a) Reoccurent viral infections of the lungs
b) Gastro-oesophageal reflux disease
c) Botha/andb/arecorrect
d) Bronchial asthma

5. The key hormone which determines the height during the first year of life is
a) insulin
b) growth hormone
c) thyroidhormone (nutrition & thyroid hormones)
d) both a/ and b/ are correct

Downloaded by Kato Caleb (katocalebnatamba@gmail.com)


lOMoAR cPSD| 41174528

6. The latest age at which a child should start talking is


a) The end of the 10th month
b) The end of the 12th month
c) The end of the 16th month
d) Theendofthe18thmonth

7. A healthy term newborn has


a)Length46cm,weight3200,anteriorfontanelle2.5/2.5cmandposteriorfontanelle
0.5cm
b) Weight 3200, anterior fontanelle 2.5/2.5cm and small fontanelle 0.5 cm
c) Weight 4700, length 51cm, anterior fontanelle 2.5/2.5cm
check
d) Length 50cm, weight 3800, anterior fontanelle 0.5/0.5cm

10 30 8. A 5 year old child with respiratory rate of 30/min has tachypnoea


I go us a) True
Y 25 35 b) False (normal=> 25-35)
0 20 29 9. A 4 year old child has blood pressure of RR 106/59 mmHg and it is
a) Normalbloodpressure
b) Arterial hypotension
c) Arterial hypertension

10. An 8 year old child has a heart rate of 120/min. Is this tachycardia?
a) True
b) False (for 8 years old=> 80-120 is normal)

11. A 1 year old child has urine output of 500ml/24h


a) This is normal urine output
b) The child has polyuria
c) Thechildhasoliguria (normal urine output=> 1-3 ml/kg/hr)

12. The most likely diagnosis in a child with the following symptoms - septic temperature
of more than 2 weeks duration, skin rash during the febrile peak, hepatosplenomegaly,
O
lymphadenomegaly, good general condition is:
a) Systemic juvenile idiopathic arthritis (characterized by fever> 2 weeks; salmon pink
rash & hepatosplenomegaly. Occurs in the absence of other causes such as infection)
b) Streptococcal sepsis
c) Haematological malignancy (blood results are not given so difficult to say)
d) Rheumatic fever (usually occurs after group A streptococcal infection such as
pharyngitis or Scarlet fever)

13. During a prophylactic check up of an infant a urine test is done which shows 4 Leuc.
in the sediment, Webb’s —10 leuc. What is the recommended action?
a) Reassuremotherthattheresultisnormal

Downloaded by Kato Caleb (katocalebnatamba@gmail.com)


lOMoAR cPSD| 41174528

b) Order urine culture as you suspect urinary tract infection


c) Offer admission to a paediatric ward
d) Both b/ and c/ are correct

14. Microcytic anaemia is found in


a) Thalassemia,irondeficiency,chronicinfection
b) Thalassemia, iron deficiency, aplastic anaemia (aplastic anemia is normocytic)
c) Infants with folate deficiency, thalassemia, iron deficiency (folic acid deficiency
anemia=> macrocytic)
d) Both a/ and b/ are correct
reduction in circulatingblood pie
purpuraassociated w a
15. Thromboca ytkoa
peunpia is characteristic in
a) Werlhof’s disease, leukaemia, sepsis
b) Haemophillia, leukemia (In Hemophilia => platelet count is normal)
c) Haemolytic ureic syndrome
d) Botha/andc/arecorrect

16. Kawasaki’s disease should be considered when the following symptoms are present:
a) Fever for 1 week, diarrhoea, conjunctivitis, rash, strawberry tongue, jaundice
b) Fever for 1 week, diarrhoea, conjunctivitis, rash, strawberry tongue, bright red and
shiny lips, blocked nose.
c) Fever for 1 week, diarrhoea, conjunctivitis, rash, strawberry tongue, bright red and
shiny lips, blocked nose and jaundice
d)a/,b/,c/areallcorrect

17. Atelectasis of the right upper lobe in an infant with respiratory failure may be due to:
a) cystic fibrosis (as a result of mucus plug)
b) food aspiration
c) congenital lobar emphysema
d) botha/andb/arecorrect

18. Bronchiolitis presents with


a) Tachypnea, dyspnea, inspiratory stridor, bronchial obstruction (there will be expiratory
wheezing in Bronchiolitis).
b)Tachypnea,dyspnea,bronchialobstruction expiratory wheezing
c) Tachypnea, dyspnea, inspiratory stridor, dysphonia, bronchial obstruction
d) both a/ and b. are correct

19. Reduced prothrombin time is found in:


a) Haemophilia
b) DICsyndrome,severevitKdeficiency c)
Thrombocytopenia
d) both b/ and c/ are correct

Downloaded by Kato Caleb (katocalebnatamba@gmail.com)


lOMoAR cPSD| 41174528

20. Tetralogy of Fallot/fallot does NOT include:


a) Pulmonaryvalveinsufficiency (subpulmonary stenosis)
b) Hypertrophy of the right ventricle
c) Ventricular septal defect
d) overring aorta.

21. Hypoxemic spells are characteristic for


a) Foramen ovale
b) Tetralogy of Fallot
c) Stenosis v.mitralis
d) Atrial septal defect

22. Which antidote is used in organophosphate poisoning


a) Prostigmin
b) Vit K
c) Atropin
d) Ahypnon

23. Absence of femoral pulses is typical for:


a) Tetralogy of Fallot
b) Atrial septal defect
c) Coarctationoftheaorta
d) Tricuspid valve atresia

24. Werlhof’s thrombocytopenia purpura presents with:


a) jaundice
b) Petechiaeandecchymoses
c) Haemarthrosis (In Hemophilia)
d) Lymphadenomegaly

25. The most likely diagnosis in a child with acute onset pallor, oliguria, azotem…
petechiae is:
a) Acute haemolytic anaemia
b) Acute glomerulonephritis
c) Hemolyticuremicsyndrome
d) Both a/ and b/ are correct

What most often kills children aged 1 to 14 years? (1 correct answer)

(1 point)

from late diagnosed congenital anomalies

for reasons of an exogenous nature that cannot be controlled

Downloaded by Kato Caleb (katocalebnatamba@gmail.com)


lOMoAR cPSD| 41174528

fromaccidents

from errors in diagnosis and treatment

The body of the infant is characterized by all physiological features with the exception of: (1 correct
answer)

(1 point)

Hydrolability

Increasedresistancetobacterialinfections

Thermolability

Tropholability

Feeding the child starts from: (1 correct answer)

(1 point)
ouldearly
after reaching the age of one
gethis
wrong thebirth - breastmilk

immediately after the neonatal period

4-5 months of age

Hydrops foetus universalis manifests itself with: (1 correct answer)

(1 point)

thrombocytosis

hydrocephalus

Anazarka-- abnormal fluid build up in tissues around lungs, heart, or abdomen and underskin.

congenital pneumonia

Kernicter can develop in: (1 correct answer)

Downloaded by Kato Caleb (katocalebnatamba@gmail.com)


lOMoAR cPSD| 41174528

(1 point)

Atresia of the biliary tract -- ͫCB

Neonatal hepatitis -- UCB+CB

Hemolyticdiseaseofthenewborn-- Kernicterus = ͫUCB (fat sol - cross BBB -- basal ganglia brain stem)

Hemorrhagic disease of the newborn - Vit K def -- bleeding

Shortened/dull percussion tone is detected in all except: (1 correct answer)

(1 point)

Emphysema

Atelectasis - collapsed lung -- dullness

Pleural effusion - dull

Pulmonary infiltrate - dull

Obstructive bronchitis is characterized by the following physical finding: (1 correct answer)

(1 point) - air trapping (cant get air out - prolonged espiration) -- hyperinflation of lungs

Hypersonicpercussiontone,vesicularbreathingwithprolongedexpiration,impairedbronchophonia

Shortened/dull percussion tone, vesicular breathing with a focus of small moist rales, increased
bronchophonia

Shortened percussion tone, vesicular breathing with prolonged expiration, increased bronchophonia

Hypersonic percussion tone, bronchial breathing, increased bronchophonia (consolidation)

Expiratory dyspnea is observed in: (1 correct answer)

(1 point)

Acute subchordal laryngitis

Bronchialasthma - obstructive lung disease -- cant get air out

Retropharyngeal abscess

Downloaded by Kato Caleb (katocalebnatamba@gmail.com)


lOMoAR cPSD| 41174528

Epiglottitis

Maintenance treatment of bronchial asthma includes the following medications: (1 correct answer)

(1 point)

Theophylline

Phytotherapy

Inhaledcorticosteroids-- +LABA as controllers

Antibiotics

10

In an infant at 4 months of age, artificial feeding is proven to be allergic to cow's milk protein. What diet
- would you recommend? (1 correct answer)

(1 point)

Take only lactose-free formula - prob is milk protein i.e whey, casein

To start power supply ?

To be fed with adapted milk with locust bean glue - anti-reflux - GERD

To take only soy milk - not appropriate nutritionally

11

At what time is the Mantoux Test reported? (1 correct answer)

(1 point)

At 24 o'clock

At72o'clock- reaction evaluated 48-72 hours >injection -- (+) 5-17mm of induration

On the 7th day

At 36 o'clock

12

Iron deficiency anemia can occur in all cases except: (1 correct answer)

(1 point)

Downloaded by Kato Caleb (katocalebnatamba@gmail.com)


lOMoAR cPSD| 41174528

Gastroesophageal reflux with reflux esophagitis - microhemorrhages in lower esophagus due to acidity

Intestinal giardiasis - watery diarrhea -- malabsorption

Eating meatless food with a predominance of cow's milk - yep

Hereditaryspherocytosis - normocytic anemia

13

Which statement about glucose-6-phosphate dehydrogenase deficiency is true :? (1 correct answer)

(1 point)

Microcytic, hypochromic anemia - IDA (G6PDH - normocytic anemia)

It runs with a positive Coombs test - Autoimmune hemolytic anemia

Theanemicsyndromeisprovokedbyspecificnoxa - drugs can trigger G6PDH hemolytic anemia

It is transmitted autosomal recessively - G6PDH = X-linked recessive

14

Which of the following statements is true for diabetes mellitus / DM /: (1 correct answer)

(1 point)

The second type of diabetes usually manifests itself in early childhood - older

The first type of TA does NOT start with DKA. -- can present as DKA

Type2diabetesisusuallyassociatedwithobesityandhasnopredispositiontodiabeticketoacidosis
(DKA). -more prone to hyperosmolar hyperglycemic nonketotic syndrome

The treatment of the first type of diabetes includes only dietary nutrition - insulin injection

15

Which of the following children should be examined for short stature? (1 correct answer)

(1 point)

Noneoftheabove

A child with a height below 3 percentiles when targeted at 3 percentiles - Short stature = <2nd centile

A child with a height of 50 percentile at a target of 50 percentile

A child with a height of 50 percentile with a target between 50 and 85 percentile

Downloaded by Kato Caleb (katocalebnatamba@gmail.com)


lOMoAR cPSD| 41174528

16

Premature Telarhe is: (1 correct answer)

(1 point)

Breastenlargementbeforetheageof8ingirls

Abundant hair on androgen dependent areas in girls

Occurrence of the first menstruation - without other data on pubertal development - precocious
puberty in girls

Premature testicular growth - precocious puberty in boys

17

Cause of adrenal insufficiency is: (1 correct answer)

(1 point)

Autoimmune adrenalitis - most common cause of addisons/adrenal insuff

Kidney tumor destructive and adjacent adrenal gland

Hemorrhage in the right adrenal gland

Alllisted

18

In which of the listed age periods is the fastest growth calculated in cm / year? (1 correct answer)

(1 point)

Between1and2yearsofage . -10-12 cm/yr

In preschool age . - 5-6cm/yr

Between 2 years and 3 years of age - 8cm/yr

Puberty - girls (8cm/yr), boys (10cm/yr)

19

The onset of puberty is determined by activation of the axis: (1 correct answer)

(1 point)

Downloaded by Kato Caleb (katocalebnatamba@gmail.com)


lOMoAR cPSD| 41174528

Hypothalamus-Pituitary-Gonads - ͫTestosterone/estradiol

Pituitary-adrenal-gonads

Hypothalamus-Pituitary-Adrenal

Pituitary gonads

20

- Celiac disease is characterized by everything EXCEPT: (1 correct answer)

(1 point)

There is a lag in growth- malabsorption due to flattened villi

It manifests itself after the introduction of flour foods in the diet.- around 8 mo-2yrs

Thesymptomspassspontaneouslyafter5-6years. -- chronic disorder

There is chronic diarrhea and behavioral changes, - GI inflammation - abdo pain, distention, diarrhea etc

21

The most common cause of urinary tract infections in childhood is: (1 correct answer)

(1 point)

Pseudomonas aeruginosa - in congenital malformations

Proteus mirabilis - > in males

E.coli - 90% cases

Enterococcus

22

T What is the most common symptom in children with SCM with a right-left shunt? (1 correct answer)

(1 point) L
Cyanosis - R-L in cyanotic CHD

hepatomegaly

rhythm and conduction disorders

swollen jugular veins

Downloaded by Kato Caleb (katocalebnatamba@gmail.com)


lOMoAR cPSD| 41174528

23

In SCM with left-right shunt the most common syndrome is: (1 correct answer)

(1 point)

Chronic hypoxia with fingers such as drumsticks and clock glass claws - R-L

Hypoxemic crises - R-L

Heartfailure

Pulmonary hypovolemia - R-L

24

Innocent / accidental / heart murmurs are characterized by the following features: (1 correct answer)

(1 point)

Theyareusuallysystolic- innocent murmurs = aSx, systolic, normal S1+2, sternal border left, soft

They are combined with a Fremisman in the precordium

They are combined with cardiomegaly - benign = no cardiomegaly

By force they reach the 4th degree

25

The hemolytic-uremic syndrome includes the triad: (1 correct answer)

(1 point)

Arterial hypertension, microangiopathic hemolytic anemia, thrombocytosis

Acuterenalimpairment,microangiopathichemolyticanemia,thrombocytopenia

Acute renal impairment, microangiopathic hemolytic anemia, thrombocytosis

Gastroenteritis, microangiopathic hemolytic anemia, thrombocytopenia

26

Which of the following tests is in favor of glomerular hematuria: (1 correct answer)

(1 point)

Presence of glucosuria - doesn't give info about hematuria

Downloaded by Kato Caleb (katocalebnatamba@gmail.com)


lOMoAR cPSD| 41174528

Presenceofover30%dysmorphicerythrocytes - dysmorphic = upper/glomerular only

Presence of proteinuria - doesn't give info about hematuria

Presence of low relative weight of urine - proteinuria?

27

Trypsin testing in bowel movements is shown in: (1 correct answer)

(1 point)

Achildwithchronicdiarrheaandsuspectedcysticfibrosis. - trypsin = specific to pancreas -- ͭin stool as


its blocked in ducts

Chronic ulcerative colitis

Intestinal parasites

Allergy to cow's milk protein

28

What is characteristic of central muscular hypotension? (1 correct answer)

(1 point)

Severe atrophy of the muscles - LMN

Fasciculations of the tongue - LMN

Leading muscle weakness - muscle weakness later - 1st spastic

Positivepathologicalreflexes

29

Which of the diseases is coagulopathy? (1 correct answer)

(1 point)

Schonlein-Henoch disease - small vessel systemic vasculitis

Hyaline-membrane disease - premature baby <37GW - ͭsurfactant -- resp distress syndrome (esp
<20GW)

Cushing's disease - hypercortisolism

Hemophilia - coagulation disorder

Downloaded by Kato Caleb (katocalebnatamba@gmail.com)


lOMoAR cPSD| 41174528

30

What deviation in erythrocyte morphology do you expect to find in a child with verified folic acid
deficiency: (1 correct answer)

(1 point)

Plenty of target cells - hemoglobinopathies (central spot of Hb in hypochromic cells)

Macrocytosis

There are no morphological deviations

Microcytosis - IDA

31

Which of the symptoms is characteristic of Wilms' tumor (nephroblastoma)? (1 correct answer)

(1 point)

Accidentallypalpatedtumorformation - as it is aSx

Polyuria

Leukocyturia

Arterial hypotension with tachycardia

32

The most common type of leukemia in children is: (1 correct answer)

(1 point)

Chronic myeloid leukemia

Acutelymphoblasticleukemia - newborns -14 yrs

Acute myeloid leukemia

Chronic lymphocytic leukemia

33

The persistence of primitive reflexes / automatic walking, automatic crawling, grasping / after 5 m.
points to: (1 correct answer)

(1 point)

Downloaded by Kato Caleb (katocalebnatamba@gmail.com)


lOMoAR cPSD| 41174528

Riskofdevelopingcerebralpalsy - UMN

Spinal muscular atrophy - LMN

Normal mental development

Normal motor development

34

A 20-day-old baby, born from a normal pregnancy, at 37 g.s., Tr -2600 g and height 49 cm, on the
occasion of a "light" heart murmur heard after birth, comes for a check-up. On examination,
double-measured blood pressure 110/85 mmHg on the left arm and 90/60 mmHg on the lower limb.
What is the most probable reason: (1 correct answer)

(2 points)

Congenital anomaly of the urinary system

Blood pressure was not measured correctly

Neuroblastoma

Aorticcoarctation - hypertension in upper but hypo in lower limbs

35

A 5-year-old boy with normal physical and neuropsychological development. Often suffering from
respiratory infections. On examination by a pediatrician, a fixed bifurcated 2nd tone and protomezo
systole noise 2 / 6st, with a puncture maximum of the 2nd left intercostal space, with propagation to the
back, were auscultated. An incomplete right femoral block is found on the ECG. Which SCM is most likely
involved? (1 correct answer)

(2 points)

Persistent arterial canal -continuous machine like murmur blow left clavicle, widened pulse pressure

Atrialdefect - Midsystolic ejection murmur on 2LISC, RBBB (proximity of AV node)

Mitral regurgitation - holosystolic murmur at apex

Interventricular defect - holosystolic murmur at lower left sternal border (Erbs)

36

A 3-month-old infant born to blood-related parents - third cousins. From the second week after birth he
has generalized edema. From studies with evidence of severe proteinuria, hypoproteinemia.
hyperlipidemia, What is the most likely diagnosis: (1 correct answer)

Downloaded by Kato Caleb (katocalebnatamba@gmail.com)


lOMoAR cPSD| 41174528

(2 points)

Congenital hydronephrosis

Mesangioproliferative glomerulonephritis - nephritic syndrome

Congenitalnephroticsyndrome

Infantile nephrotic syndrome

37

A boy who had been ill for several days with fever, heart pain, tachypnea, tachycardia. Examination
revealed pericardial friction, and from the next day ultrasound data for exudative pericarditis. Which
symptom points to threatening pericardial tamponade? (1 correct answer)

(2 points)

Higher blood pressure in the left arm than in the left leg - Coarctation of aorta

Swollenjugularveins - Beck’s triad - distended jugular vein, decreased BP, muffled heart sounds.

ECG data for ST elevation - MI, pericarditis etc

Well filled pulse.

38

The girl in the photo is 7 years old from the first normal pregnancy and birth at the age of 40, weighing
2,500 grams on the 3rd percentile and 48 cm long on the 3rd percentile. Operated for coarctation of the
aorta. It is monitored by a pediatric cardiologist and has normal heart function. He is currently taller and
weighs less than 3 percentiles. She walked and spoke on time, but had a hard time dealing with the
study material. What is the test that will help you the most in making the diagnosis? (1 correct answer)

(2 points)

Growth hormone deficiency tests - always has been smaller -- GH only for growth in childhood

Magnetic resonance imaging of the central nervous system - normal motor and lingual development

Cytogeneticexamination - for genetic abnormalities as woman >35 yrs

Wrist radiography for bone age - no signs of precocious puberty

39

In a child of 1 1/2 years of age with spastic quadriparesis, but without risks in the perinatal period, which
examination must be performed in the differential diagnostic plan of quadriparesis? (1 correct answer)

Downloaded by Kato Caleb (katocalebnatamba@gmail.com)


lOMoAR cPSD| 41174528

(2 points)

PAC

EMG

MRIofthebrain - rule out CP, UMN lesions etc

EEG

40

A 3-year-old child who has frequent infections of the respiratory system is still unable to walk stably, for
which he was examined by a neurologist, who suggested that the unstable gait was ataxia. Dilated
vessels in the conjunctiva also began to appear. What is the most likely diagnosis? (1 correct answer)

(2 points)

Ataxia-Telangiectasia - AR due to cerebellar generation, telangiectasia, immunodef, >risk for cancer,


radiation sensitivity

Wiskott-Aldrich Syndrome - eczema, thrombocytopenia, immune def + bloody diarrhea

Congenital syphilis

Bruton's disease - agammaglobulinemia

41

Infant born from the first normal pregnancy and birth weighing 3500 g and height 51 cm. Normal
physical development. When assessing his neuropsychological development, it is established that the
child reaches for a toy and grabs it, follows it, laughs out loud and turns his head in the direction of
sound, sound. It does not turn from the back to the abdomen. The child is responsible for the
neuro-psychological development of: (1 correct answer)

(2 points)

1 month of age

4monthsofage

6 months of age

8 months of age

42

A 2-month-old boy, born at the age of 36, from a second normal pregnancy, with a body weight of 2200
g, height 49 cm and a head circumference of 33 cm, with a normal Apgar. During pregnancy, the mother

Downloaded by Kato Caleb (katocalebnatamba@gmail.com)


lOMoAR cPSD| 41174528

felt less movement of the fetus. Twisted hard. He made the impression that the child was relaxed and
had difficulty breathing. She cries with a weak cry. During the examination it is established that head -
37.5 cm, normal fontanelle. Respiratory rate 66 / min, rhythmic heart rate with a frequency of 160 / min.
MN Without hepatosplenomegaly. Severe muscular hypotension, very limited movements - only proximal.
Tendon and periosteal reflexes are missing bilaterally. Good emotional contact, follow with a look. In a
T
relaxed state, fasciculations of the tongue are observed. The mother's first child is a healthy girl. What is
the most likely diagnosis? (1 correct answer)

(2 points) Isma
Duchenne muscular dystrophy - presents around 3 yrs w/initial normal motor development

Hypoxic-ischemic encephalopathy - UMN signs missing

Cerebral palsy - hypotonic form - umn


Spinalmuscularatrophy - ͭfetal movements, weak cry, severe muscle hypotension (1st-proximal),
fasciculation etc - SMAT1

43

A 5-month-old child, born according to a normal mechanism, with T - 3000 years, in a slightly depressed
state. With a slight delay in the CPD - hooks and traces only from 4m.v., still does not turn from back to
abdomen. From 2 weeks there are attacks of sudden spreading of the legs and arms for seconds, then
sometimes rinsing. The attacks are repeated every 30-40 seconds 10-15 times and there are 2-3 sets a
day, usually after waking up. He began to coo less often. There is no change in appetite and defecation.
The temperature did not rise. Which test is mandatory to determine what these seizures are? (1 correct
answer)

(2 points)

EEG - West syndrome aka infantile spasm - EEG = Hypsarrhythmia A intellectual disability
Transfontaneal ultrasound

Thyroid-stimulating hormone

Abdominal ultrasound

44

About Down Syndrome is true: (1 correct answer)

(2 points)

Premature puberty is typical - delayed puberty -gonadal unresponsiveness

Development of hydrocephalus

Downloaded by Kato Caleb (katocalebnatamba@gmail.com)


lOMoAR cPSD| 41174528

There is tendon areflexia

Presenceofmacroglossia

45

A 10-year-old boy with a normal premorbid history. From the last 2 weeks he has lost weight, he has
become paler. He did not raise the temperature. Objectively: pale skin and visible mucous membranes,
unenlarged peripheral lymph nodes, hepar - 4 cm below the costal arch, spleen 3 cm with a dense elastic
consistency. No pathology in the remaining status. From PKK - Hb 80g / l; Levko - 35x10 / 9, Platelets -
100x 10/9; DCC - lymphocytosis 85% - atypical lymphocytes. SUE 35mm. AST - 35 U; TOOL -40U; bilirubin
- 18 μmol / l; C-reactive protein -50 mg / l; LDH - 1500 U, urea, creatinine, ionogram normal. What is the
most likely diagnosis? (1 correct answer)

(2 points)

Sepsis

Tuberculosis

Infectious mononucleosis

Acutelymphoblasticleukemia

46

When examining a 6-year-old child with unilateral cervical lymphadenopathy, which symptom most
points to a malignant process? (1 correct answer)

(2 points)

Removable with soft elastic consistency

Fluctuating painful lymph nodes

Fixedlymphnodeswithafirmconsistency

Presence of systemic manifestations - fever, rash

47

A 3-day-old infant has repeated vomiting soon after feeding from birth. There is an impurity of bile in the
vomit. The child sucks actively. There is no bowel movement in the last 2 days. The X-ray examination
found two water-air levels in the upright position - one in the stomach and the other in the proximal part
of the duodenum. Your diagnosis is: (1 correct answer)

(2 points)

Downloaded by Kato Caleb (katocalebnatamba@gmail.com)


lOMoAR cPSD| 41174528

Duodenalatresia- double bubble sign, bilious vomiting early >birth, vomiting after feeding

Congenital pyloric stenosis - >2-7 weeks, non-bilious, projectile vomiting

Congenital hyperplasia of the adrenal glands with salt loss - i.e lack of aldosterone -- hypotension

Hirshprung's disease - failure to pass meconium in 24 hrs, meconium ileus -- abdominal distention --
later bilious vomiting

48

In a 6-month-old infant, with prolonged jaundice and marked pallor with adequate weight gain and
height. PKK study was performed: hemoglobin: 51 g / l, erythrocytes -6.2x10 / 12, mcv: 59 fl, leukocytes:
6.85 x 109 / l, platelets: 258 x 109 / l. The examination showed evidence of hepatosplenomegaly, and
laboratory tests revealed total bilirubin-114 in direct-6.2 micromol / l, elevated ferritin. The child is fed
meat purees and is on a natural diet. What is your probable diagnosis? (1 correct answer)

(2 points)

Iron deficiency anemia - of chro

Glucose-6-phosphate dehydrogenase deficiency


not mentioned
HomozygousBetathalassemia- microcytic anemia, ͫbilirubin, HSM etc

Foil deficiency anemia Q


49

A 6-year-old child of three days complains of pain in the left hypochondrium, without vomiting,
constipation or diarrhea. He urinates without pain, but sometimes the urine is red. Arterial hypertension
up to 140/90 has been registered. Examination reveals a formation in the left hypochondrium. Urine
examination was also performed - without data for proteinuria and leukocyturia, but with 55
erythrocytes according to Webb. What is the probable diagnosis? (1 correct answer)

(2 points)

Wilms'tumor - pain left hypochondrium (uncommon but possible), HT, hematuria

Microspherocytic anemia

Nephrolithiasis

Splenomegaly

50

What deviation is characteristic of Bruton's congenital agammaglobulinemia? (1 correct answer)

Downloaded by Kato Caleb (katocalebnatamba@gmail.com)


lOMoAR cPSD| 41174528

(2 points)

Lack of T-lymphocytes

CompleteabsenceofB-lymphocytes

Positive NBT test - impaired phagocytosis

Neutropenia

51

A 4-month-old baby who is formula-fed with infant formula. For about a month he has had diarrhea -
heavy oily stools, with an unpleasant odor and alkaline pH. She was born with T - 3000g, height 50 cm,
and currently she is with T 4500g, height -61 cm. He has not had a fever and has not been ill so far.
Reduced subcutaneous adipose tissue is found, without other pathology. What notes should be
excluded? (from 1 - 5 correct answers; for correct answer +1 point, for wrong answer -1 point)

Congenital lactase deficiency - appear earlier?

Allergytocow'smilkprotein

Cysticfibrosis

Celiac disease - 4mo - only milk -- >intro gluten -- around 8 mo-2 yrs

Chronic hepatitis

52

West syndrome is characterized by: (from 1 - 5 correct answers; for correct answer +1 point, for wrong
answer -1 point)

Delayinneuropsychologicaldevelopment

Metabolic acidosis

EEG-dataonhypsarrhythmia

Hypoglycaemic seizures

Attacks of severe abdominal colic

53

Acute respiratory failure may occur in? (from 1 - 5 correct answers; for correct answer +1 point, for
wrong answer -1 point)

acute tonsillitis - upper resp

Downloaded by Kato Caleb (katocalebnatamba@gmail.com)


lOMoAR cPSD| 41174528

acutebronchiolitis

acute rhinosinusitis - upper resp

Guillain-Barrésyndrome- resp m weakness

acute tracheitis - upper resp

54

Which of the following clinical manifestations are characteristic of systemic juvenile arthritis: (from 1 - 5
correct answers; for correct answer +1 point, for wrong answer -1 point)

gallbladder hydrops

intermittentsepticfeverlasting2weeksormore

encephalitis

glomerulonephritis

intermittentrashassociatedwithfebrilepeaks - high fever + salmon pink rash

55

Which of the following are characteristic of pneumocystis pneumonia? (from 1 - 5 correct answers; for
correct answer +1 point, for wrong answer -1 point) - pneumocystis pneumonia - pneumocystis jiroveci =
fungus --opportunistic in immunosupp . --usually aSx --dyspnea,tachypnea --can progress to resp failure

X-ray revealed destructive pneumonia and pneumatocele - S.aureus

Itmainlyaffectschildrenwithmalnutritionandimmunedeficiency
disease
It is typical for school age

fungal
Extreme tachypnea and radiological changes such as frosted glass. - symmetrical diffuse interstitial
infiltrates

A sharp start

56

Splenectomy is a method of treatment at a certain stage of the course: (from 1 - 5 correct answers; for
correct answer +1 point, for wrong answer -1 point)

Glucose-6-phosphate dehydrogenase deficiency - most severe - blood transfusions

Fanconi's anemia - BM suppression/impaired function

Downloaded by Kato Caleb (katocalebnatamba@gmail.com)


lOMoAR cPSD| 41174528

Hereditaryspherocytosis

Homozygousbetathalassemia

Vitamin B12-deficiency anemia

Splenectomy - autoimmune, thalassemia, hereditary spherocytosis, HbS

57

Prognostic factors for high risk in Acute lymphoblastic leukemia are: (from 1 - 5 correct answers; for
correct answer +1 point, for wrong answer -1 point)

CNSengagement

iƒ Presence of hepatosplenomegaly

Presenceofperipherallymphadenomegaly

Presence of Philadelphia chromosome in blast cells - poor

Joint engagement

58

A 7-year-old child with attacks of bronchial obstruction 4-5 times a year, starting from 3 years of age. The
attacks are described by a doctor as mild. The reason for the consultation is another attack with rapid
breathing, coughing and difficulty breathing. The physical and psychomotor development of the child is
normal. He has a brother with hay fever, his parents are also healthy. A. Which of the following studies is
most appropriate? (1 correct answer)

(2 points)

X-rayofthelung - rule out infection -- then test asthma

Study of IgA, IgM and IgG - asthma = IgE

PPD 5ME

Sweat test

59

B58 B. All tests are normal. What is the most likely diagnosis? (1 correct answer)

(2 points)

Cystic fibrosis

Downloaded by Kato Caleb (katocalebnatamba@gmail.com)


lOMoAR cPSD| 41174528

Bronchialasthma - multiple episodes of bronchial obstruction, fam Hx of atopy

Bruton's disease

Tuberculosis of the tracheobronchial lymph nodes

60

Q58 Q. What is the best treatment for the condition? (1 correct answer)

(2 points)

IncaseofBOS-inhaledsalbutamol

Immunoglobulin infusion treatment for seizures

Tubocin treatment for 6 months.

Pancreatic enzymes and inhalations with tobramycin

61 hanstouterelle Echo

I
A 27-day-old infant was brought to the outpatient clinic. He was born in 32 BC. weighing 1900 g, head
circumference 31 cm. It was on oxygen therapy. There were double clonic twitching of the limbs. From
performed TFE - data on intraventricular hemorrhage. It is written on the 12th day with jaundice,
continuing until now, On artificial nutrition. In the last 2 days the child becomes drowsy, has difficulty
eating, lacks fever, From the status - icteric skin and sclera, bombed fontanelle, distant sagittal suture.
About. head -37cm. Respiration - 24- arrhythmic vesicular, RSD 90 / min, clear tones, good pulse, RR 95 /
70. No hepatosplenomegaly. Reacts sluggishly, drifts off quickly. Generalized muscular hypotension, live
CHP. PKK- Hb - 140 g / l, Er- 4, 8 X10 9, Leukocytes and Reticulocytes - normal, indirect
hyperbilirubinemia, normal C reactive protein and other biochemical parameters, CAS. Normal ARTT,
thrombin time and fibrinogen. A. What do you think is the reason for the deterioration of the condition?
(1 correct answer)

(2 points)

Sepsis

Post-hemorrhagichydrocephalus

Hemorrhagic disease of the newborn

Congenital Metabolic Disease

62

Q61 B. What additional examination will help to clarify the diagnosis? : (1 correct answer)

Downloaded by Kato Caleb (katocalebnatamba@gmail.com)


lOMoAR cPSD| 41174528

(2 points)

EEG

Blood culture

Metabolic screening.

TFE - transfontanelle echo to check for hydrocephalus

63

Q61 Q. What do you think is the cause of prolonged jaundice in a child? (1 correct answer)

(2 points)

Hemolytic disease of the newborn

Resultoflysisoferythrocytesinaclosedspace/intraventricularhemorrhage/

Combined cause of sepsis

Glucoronyltransferase immaturity

64

A 6-month-old boy from the first normal pregnancy, born full-term with T-2900 g, without evidence of
asphyxia. Breastfeeding until 3 months of age. Powered by 4m.v. with vegetable puree. Although the
child has a good appetite and does not vomit, he does not gain weight and has twice suffered from
pneumonia. I often cough with a paroxysmal dry cough. The child was admitted to an infectious disease
clinic for diarrhea - the stools were profuse, foul-smelling, without impurities of blood and mucus, with
alkaline pH, negative co-cultures. The mother has Hashimoto's thyroiditis. During the examination, the
personal doctor found severely reduced subcutaneous adipose tissue of the chest, abdomen and limbs.
Currently weighing 4700g. Lung - vesicular respiration 40 / min, slightly prolonged expiration, no
wheezing. RSD, clear tones. Without hepatosplenomegaly. A. What is your opinion? (1 correct answer)

(2 points)

The child was born with low weight and that is why he is with this weight at the moment

ThechildhasgradeIImalnutrition - poor weight gain -- severely reduced subcutaneous adipose tissue,


freq infections etc

The weight is according to age

The child was fed improperly

65

Downloaded by Kato Caleb (katocalebnatamba@gmail.com)


lOMoAR cPSD| 41174528

Q64 B. Which diagnosis should you rule out? (1 correct answer)

(2 points)

Gastroesophageal reflux

Cysticfibrosis- freq resp infections, foul smelling stools, malabsorption etc

Pertussis

Celiac disease

66

Q64 Q. Which of the following tests will you order to clarify the diagnosis? (1 correct answer)

(2 points)

PCR for Pertussis from throat secretions - whooping cough

Sweattest

Fibrogastroscopy

Tissue transglutaminase test - coeliac

States Paper 2

1. “Sitting cat position” is a symptom of: (1correct answer) . Single choice. (1 Point)

Vit A deficiency

VitDdeficiency

Hypervitaminosis A

Hypervitaminosis D

2. A febrile seizure may occur at the age of: (1correct answer) . Single choice. (1 Point)

Downloaded by Kato Caleb (katocalebnatamba@gmail.com)


lOMoAR cPSD| 41174528

4mo-4yr (Occurs usually between 6 months-6 year)

1mo - 1yr

3 mo - 6 yr

Any age

3. A child can walk, scribble, use a spoon as early as at: (1correct answer) . Single choice. (1 Point)

18months (walks well by 15 months; scribbles by 15 months & feeds themself by 18 months)

30 months

6 months complex

12 months

4. In pyelonephritis you search for: (1correct answer) . Single choice. (1 Point)

Proteinuria and hematuria

Proteinuria and leucocyturia

Bacterturia and leucocyturia

Haematuria and leukocyturia

5. The most common cause of acute tonsillitis is: (1correct answer) . Single choice. (1 Point)
Influenzae virus

Haemophilus influenzae type B

Streptococcusbeta-hemolyticgroupA(S.pyogenes)

Epstein-Barr virus

6. Chronic pneumonia and chronic diarrhoea are found in: (1correct answer) . Single choice. (1
Point)

Hypogammaglobulinemia

Alpha 1-antitrypsin deficiency

Mycobacterium bovine infection

Cystic fibrosis

7. Which of the following about the BCG vaccine is valid? (1correct answer) . Single choice. (1 Point)

Downloaded by Kato Caleb (katocalebnatamba@gmail.com)


lOMoAR cPSD| 41174528

It is a purified protein derivative heated to 150 ° C

Immunity is due to primed B-cells

It is not given to newborns in the neonatology ward

Containsattenuatedbovinetuberculosisbacteria

8. Which of the following drugs are of major importance in treatment of anaphylaxis? (1correct
answer) . Single choice. (1 Point)

Dexametazon

Adrenaline

Levocetirizine

glucose 10%

9. Elevated D-dimers in the absence of internal bleeding are found in: (1correct answer) . Single
choice. (1 Point)

Hemophilia

DICsyndrome (D-Dimer is a product of fibrinogen breakdown and is increased when there is


thrombosis. DIC is characterized by both thrombosis & bleeding event so D-Dimer is increased
here)

Glanzmann thrombasthenia

Thrombocytopenia

10. Werlhof's thrombocytopenic purpura presents typically with: (1correct answer) . Single choice.
(1 Point)

Palpable purpura

Jaundice

Petechiae

Haematomas

11. The key hormone which determines the height after 3 years of life is: (1correct answer) . Single
choice. (1 Point)

Both a/ and b are correct

Insulin

Downloaded by Kato Caleb (katocalebnatamba@gmail.com)


lOMoAR cPSD| 41174528

Thyroid hormones

Growthhormone

12. Generalized epilepsy with febrile seizures plus syndrome (GEFS+) is inherited

by : ( 1 correct answer) . Single choice.

(1 Point) Codominat mode

Autosomal recessive mode

X-linked mode

Autosomaldominantmode

13. Which anti-biotic is suitable for urinary tract infection: (1correct answer) . Single choice. (1 Point)
clarithromycin

clindamycin

cefalosporin3rdgeneration (Mainly Cephalosporin; co-amoxiclav & trimethoprim is used)

vancomycin

14. Diarrhoea, large abdomen and malnutrition are characteristic of: (1correct answer) . Single
choice. (1 Point)

Intolerance to cow's milk proteins

Duodenal stenosis (you would have bilious vomiting)

Celiac disease

Hirschprung's disease (delayed passage of meconium & bilious vomiting)

15. Which disease is characterized by severely impaired digestion of the fats and proteins of the
general food, causing bulky fatty and very foul-smelling feces? (1correct answer) . Single choice.
(1 Point)

Cysticfibrosis Lactase

deficiency Celiac

disease Exudative

enteropathy

Downloaded by Kato Caleb (katocalebnatamba@gmail.com)


lOMoAR cPSD| 41174528

16. Newborn to a mother with gestational diabetes is born by cesarean section at 37 p.m. with birth
weight 3900 g, height - 51 cm, head circumference - 36 cm. Apgar - 8; 9. Difficult postpartum
adaptation, placed on oxygen for two hours, then given for breastfeeding to the mother. On the
second day, the mother registered irritability, posturing backwards and trembling of the limbs.
The neonatologist observed focal clonic seizures of the left hand, clonus of the feet, chin tremor,
oral automatisms. Hgb - 160 g / l, RBC - 5.5x1012/l, WBC - 12.2x109/l, Tr - 410x109/l, ESR - 12
mm. CRP - 0.1 g/l, blood sugar - 3.2 mmol/l, calcium - 1.96 mmol/l, ionized calcium - 0.7 mmo/l.
What is the cause of the seizures? (1 correct answer) . Single choice. (2 Points)

Neonatalhypocalcemia ( Normal value for total calcium => 2- 2.25 mmol/l & ionized calcium =>
1.1 to 1.35 mmol/l)

Neonatal sepsis

Cerebral hemorrhage

Neonatal hypoglycaemia

17. A 2-year-old boy became ill with gastroenteritis - repeated vomiting during the day and watery
stools, fever up to 38 degrees. A few hours later he became lethargic, sleepy and the mother
noticed that he had not urinated for the last 4-5 hours. Heart rate 140 / min. Blood pressure
85/50. What treatment is necessary? (1 correct answer) . Single choice. (2 Points)

Startintravenousrehydrationwith0,9%NaCl (signs of dehydration => start with i.v. hydration)

Start intravenous rehydration with only glucose 5%

Apply furosemide for acute renal impairment

Apply mannitol for brain oedema

18. A 4-year-old girl is diagnosed with cystic fibrosis at age 2 years, but was not checked regularly in
the last years. Now she is admitted to the clinic for jaundice, pneumonia, liver at 3 cm below
costal margin. Paraclinical data were elevated transaminases, gamma-glutamyl transpeptidase
and alkaline phosphatase, direct hyperbilirubiemia, normal albumin and prothrombin time.
Which extrapulmonary complication of cystic fibrosis you consider most likely? (1 correct
answer) . Single choice. (2 Points)

Bile sludge syndrome

Non-alcoholicsteatosisoftheliver (albumin is normal)

Liver cirrhosis

Chronic viral hepatitis C.

Downloaded by Kato Caleb (katocalebnatamba@gmail.com)


lOMoAR cPSD| 41174528

19. This 3-month-old infant was brought to a well -baby cli nic. She was born from a seventh
pregnancy in a family with low social status. Born at home. She has not been examined by a
doctor so far. She had good appetite and wei ght gai n. Besi des the appearance seen in the photo,
the girl had generalized muscular hypotension, no hepatosplenomegal y, heart rate 160 min,
clear tones, 2/3 endoapical heart murmur. She l agged behind in his neuropsychological
development. What is the probable diagnosi s? (1 correct answer) . Single choice. (2 Points)

Mucopolysaccharidosis

Congenital hypothyroidism

Zellweger syndrome

Downsyndrome

20. A 14 year old girl with normal development up to now suffers from oligomenorrhea - last regular
menstruation was 2 months ago. She gained weight- 15 kg for three months. She complains of
increased fatigue and drowsiness. Family history of thyroid disease and diabetes. On
examination she is apathetic, with poor facial expressions, BP 110/60; heart rate 55 / min. No
significant deviations in the rest of the physical status. What will you do first? (1 correct answer) .
Single choice. (2 Points)

examine DHEA-S

send for psychiatric consultationия 1

examineTSHandfT4

perform heart ultrasound

21. Afour-year-oldchildwassenttograndmother'svillagetobeisolatedfromchickenpox.Itwas
healthywhenitwentouttoplayintheyardinthesun.Whenhereturnedhome,the grandmother
noticedthathisspeechisinadequateandblurred,thechildstaggersasifdrunk
andhasaflushedface.Thedoctorfoundatemperatureof37.8oC,tachycardia125/min.,blood
pressure120/80,largepupilsanddrytongueandmucousmembranesofthemouth.Which condition
explainsthesesymptomsbest?(1outof4correctanswers).Multiplechoice.(2
Points)

Downloaded by Kato Caleb (katocalebnatamba@gmail.com)


lOMoAR cPSD| 41174528

Onset of chickenpox, possibly complicated by encephalitis

Heatshock

Belladonna poisoning

Onset of an acute viral infection

22. Aone-montholdinfantiswithlumbosacralspinabifidaapertadetectedatbirth.Discharged fromthe


MaternityWardafterconsultationwithaneurosurgeonforsurgicaltreatmentafter reachingabody
weightof4000grams,anduntilthenregulardressingsofthedefectaremade. Thechildwas
hospitalizedbecauseoftwo-daysfever,refusaloffoodanddifficultawakening. Whichofthe
followingfindingsismostIMPROBABLE?(1outof4correctanswers).Multiple choice.(2Points)

Positive blood culture

Positive urine culture

Buldging fontanel

PulmonaryconsolidationonX-ray

23. A 13-year old girl was admitted for midface rash, fever up to 37,8oC. ESR 120mm, leukocytes
2,8x109/l, platelets 110x109/l and proteinuria was 1g/l . Which of the following additional
results are most probable? (1-4 correct answers) . Multiple choice. (1 Point)

Anti-streptolysin O (ASLO) antibodies in blood

Anti-dsDNAantibodiespositiveinblood

Bone marrow infiltration with abnormal cells

Leukocytoiclastic vasculitis in skin biopsy

24. A 3-month old girl was admitted in the pediatric department for difficulties in feeding. The
objective examination revealed diminished active movements, head lag in traction maneuver,
diminished tendon reflexes, weak pharyngeal reflex and constant mild movements inside the
tongue. The baby looked bright, smiled and cooed. Creatinkinase was 800IU/l. Which is the most
probable diagnosis? (1-4 correct answers) . Multiple choice. (1 Point)

Progressive muscular dystrophy (Duchenne & Baker affect males & usually occur after 3 years
old)

Congenitalmyopathy

Guillain-Barre syndrome (there will be no rise in creatinine kinase)

Downloaded by Kato Caleb (katocalebnatamba@gmail.com)


lOMoAR cPSD| 41174528

Spinal muscular atrophy (Creatine Kinase is normal)

25. A 9-year-old girl has been referred because her weight centile has increased in the last 10
months. She is an excellent student and has no other complaints. Parents report no dietary
change. On examination there is no acne, striae or hirsutism. Her weight is on the 97th centile
and her height is between 75th and 90th centile. What is the most adequate course of action?
(1-4 correct answers) . Multiple choice. (2 Points)

furtherteststoruleouthypothyroidism

start metformin treatment immediately

evaluation for hypersomatotropism (elevated growth hormone)

further tests to rule out Cushing's syndrome

26. In the well-baby clinic, a 8 months old infant from first normal pregnancy with birth with a
weight 3500 g, height 51 cm and head circumference 35 cm is examined. It currently weighs 6.0
kg, is 60 cm long and has a head circumference of 39 cm. When assessing its neuropsychological
development, it is established that the child raises his head in a prone position, has good leg
support, rolls over, sits alone, does not crawl or stand, turns his head in the direction of the
sound, reaches for a toy and holds it, smiles, coos rarely, does not pronounce syllables and
words. Which of the following statements is true? (1-5 correct answers, +1 point for a correct
answer, -1 point for a wrong answer) . Multiple choice.

There is an acceleration in physical development.

Thereisadelayinspeechdevelopment

There is a delay in motor development

There is an acceleration in motor development.

Thereisadelayinphysicaldevelopment.

27. What are among the 3 most common causes of death of children under 1 year? (1-5 correct
answers, + 1 point for correct answer, -1 point for wrong): . Multiple choice.

Traffic accidents

Poisoning

Congenitalanomalies

Birth injuries

Downloaded by Kato Caleb (katocalebnatamba@gmail.com)


lOMoAR cPSD| 41174528

Infectionsoftherespiratorysystem

28. A full-term newborn develops jaundice within the first 24 hours. Jaundice is mainly due to
indirect bilirubin. Anemia, mild hepatosplenomegaly and the presence of erythroblasts in the
peripheral blood are found. Which of the following statements is true? (1-5 correct answers, + 1
point for correct answer, -1 point for wrong): . Multiple choice.

No treatment is needed; just monitor bilirubin levels

It is probable the newborn to be Rh+ and the mothers Rh---


300mn11
Phototherapy should be applied

The bilirubin level is monitored every 4 hours and in case of increase > 150 mmol/l an exchange
transfusion should be undertaken

Gastric lavage is necessary

29.

Infant at 4 months of second normal pregnancy, born full term weighing 3100 g. Older sister of 5
years, wi th bronchial asthma. On a diet wi th formula and mother' s mil k si nce age 2 months.Wi th
regurgitati on from birth. I n the l ast month wi th unsati sfactory wei ght gai n and an i ncrease i n the
number of bowel movements (5 times a day), which became more fluid, with presence of blood
in some of them . Upon adm iss ion, the chi ld we ighed 5100 g , w ith no s igns of dehydrat ion, w ith
erythemo-exudative rash on the face and crustae l actea on the scal p (see photo). No
abnormalities in lung and heart status. Abdomen - without organomegaly. Hemoglobin - 92 g / l,
MCV -70fl, leukocytes - 10 x 10 ^ 9 / l, platelets - 120 x 10 ^ 9 / l, normal coagulation tests,
transaminases, residual nitrogen fractions, serum zinc, total protein and albumin. The mother
had scars on her right breast 2 weeks ago. What do you think should be undertaken? (1-5 correct
answers, + 1 point for correct answer, -1 point for wrong): . Multiple choice.

Downloaded by Kato Caleb (katocalebnatamba@gmail.com)


lOMoAR cPSD| 41174528

why
Feedwithformulawithproteindegradedtopeptides

Switch to lactose-free milk, as there is lactase deficit.

Search for elevated eosinophils

SearchforelevatedIgE

Search for elevated CRP

30. Baby-boy at 4 months is admitted because of diminution of consciousness and right-sided


clonic seizures after a mild head injury. From the status - paresis of the n.oculomotorius sin
(ptosis of the left eyelid and dilated left pupil), hypotonicity with reduced active movements of
the right limbs. Eye fundi - retinal hemorrhages in both eyes. Which is the most probable site of
injury? (1 of 4 correct answers) . Multi ple choi ce. (2 Poi nts)

Pons

Right brain hemisphere

Left orbit

Left brain hemisphere (coz the seizure is on the right side)

31.

Q32 - Emergency CT revealed the following scan (right side of the patient is on the left side of
the image). What is the abnormality? (1 of 4 correct answers) . Multiple choice. (2 Points)
Symmetrical skull fractures

Right-sided acute subdural hematoma

Left-s ided acute subdural hematoma

Frontal subarachnoid haemorrhage

Q33 - A consulting radiologi st suggested bil ateral ol d and new i njuri es. What i s the most
probable diagnosis? (1 of 4 correct answers) . Multi ple choice. (2 Poi nts)

Downloaded by Kato Caleb (katocalebnatamba@gmail.com)


lOMoAR cPSD| 41174528

AV malformation

Chronic meningitis

Werlhof's disease

Shakenbabysyndrome

Paediatrics’ State Exam 09.11.2020


Kidney tubulopathy is a sign of: (1correct answer)
. Single choice. (1
Point)
Hypervitaminosis A
VitDdeficiency
Hypervitaminosis D
Vit A deficiency

2
2,

A complex of VSD, comprised of overriding of the aorta with respect to the


ventricular septum, subpulmonary stenosis causing right ventricular outflow
tract obstruction and right ventricular hypertrophy are signs of: (1correct
answer)
. Single choice.
(1 Point)
Adult-type coarctation of the aorta
Persistent ductus arteriosus
TetralogyofFallot

Downloaded by Kato Caleb (katocalebnatamba@gmail.com)


lOMoAR cPSD| 41174528

Transposition of the great arteries

3
3,

The earliest clinical sign of hypothyroidism in infancy is: (1correct answer)


. Single choice.
(1 Point)
Warm dry skin, hyperthermia - cold dry skin
Growth delay - later
Mental retardation
Prolongedjaundice - around 2 weeks

4
4,

Absolute contraindication for breast feeding is: (1correct answer)


. Single choice.
(1 Point)
Maternal upper respiratory tract infection
History of maternal tuberculosis after complete treatment - already Tx
Galactosemia
Maternal HIV infection - Syphilis/HIV contracted after conception -- < = ok

5
5,

Which statement is valid or subchordal laryngitis (croup): (1correct answer)


. Single choice.
(1 Point)
Stridor is predominantly expiratory - inspiratory
Nebulizedadrenalinereducestheseverityoftheobstruction - good response to epi unlike
epiglotitis
Presents with productive cough - most common = RSV=Viral
Irritability should be relieved by mild sedation - do not sedate! = resp suppression

6
6,

A febrile seizure may occur at the age of: (1correct answer)


. Single choice.
(1 Point)
4 mo - 4 yr
Any age

Downloaded by Kato Caleb (katocalebnatamba@gmail.com)


lOMoAR cPSD| 41174528

3mo-6yr
1mo - 1yr

7
7,

At what age you would expect a child to start reaching and grasping?
(1correct answer)
. Single choice.
(1 Point)
4months
6 months
15 months
10 months

8
8,

The typical age of onset for minimal changes nephrotic syndrome is: (1correct
answer)
. Single choice.
(1 Point)
Infancy
2-5years
5-10 years
>15 years

9
9,

At what age the infant starts to pronounce first meaningful words other than
“mama” and “dada”? (1correct answer)
. Single choice.
(1 Point)
at around 36 months of age
at around 8-9 months of age
at around 21 months of age
ataround11-12monthsofage

10
10,

The most common cause of acute tonsillitis is: (1correct answer)


. Single choice.

Downloaded by Kato Caleb (katocalebnatamba@gmail.com)


lOMoAR cPSD| 41174528

(1 Point)
Streptococcusbeta-hemolyticgroupA(S.pyogenes)
Epstein-Barr virus
Influenzae virus
Haemophilus influenzae type B

11
11,

Delayed passage of meconium in the newborn period is a common feature of:


(1correct answer)
. Single choice.
(1 Point)
Duodenal atresia - bilious vomiting
Cysticfibrosis - meconium ileus -- abdo distension --later - bilious vomiting
Breast-fed children
Coeliac disease - Sx 8 mo-2 yrs >gluten introduced

12
12,

Which of the following is NOT a contraindication for vaccination? (1correct


answer)
. Single choice.
(1 Point)
Electivesplenectomyforhemolyticanemia needs to be vaccinated against encapsulated
organisms e.g. meningitis c, Hib, S. pneumoniae etc
Steroid therapy - immunosuppressed
Immunoglobulin replacement therapy
Acute respiratory illness

13
13,

Which of the following drugs are of major importance in treatment of


anaphylaxis? (1correct answer)
. Single choice.
(1 Point)
adrenalin
dexametazon
glucose 10%
levocetirizine14

14,

Downloaded by Kato Caleb (katocalebnatamba@gmail.com)


lOMoAR cPSD| 41174528

Which of the following findings makes the diagnosis of asthma UNLIKELY? :


(1correct answer)
. Single choice.
(1 Point)
Cough only during vigorous exercise
Failuretoimproveafter2weeksonoralsteroids - should improve as oral CS - Tx esp in asthma
attack
Absence of a family history of asthma - Fam Hx is not compulsory
Emphysema on chest X-ray - most normal but can present w/hyperinflation

15
15,

Which of the following is TRUE for Haemophilia A: (1correct answer)


. Single choice.
(1 Point)
SeverityisdeterminedbytheleveloffactorVIII
Presents with petechial rash - deep bleeding e.g muscles + joints
Treatment is intramuscular injections of Factor VIII concentrates - never do IM injection for
hemophilia along w/NSAIDs
Inheritance is autosomal dominant - X-linked recessive

16
16,

Which antidote is used in atropin poisoning: (1correct answer)


. Single choice.
(1 Point)
Prostigmin or
Vit. K
pilo ionpine
Protamin sulfate
Ahypnon

17
17,

What is true for Idiopathic thrombocytopenic purpura: (1correct answer)


. Single choice.
(1 Point)
Isprecededbyaviralupperrespiratorytractinfectionin>60%ofcases - also >MMR vaccine
Requires urgent platelet transfusion to prevent intracranial hemorrhage - self-limiting -- only
observe
Presents with haematomas - mucocutaneous bleeding

Downloaded by Kato Caleb (katocalebnatamba@gmail.com)


lOMoAR cPSD| 41174528

Results from decreased platelet production - >plt destruction due to anti-plt antibodies
18
18,

Which of the following carries most unfavorable prognosis in systemic lupus:


(1correct answer)
. Single choice.
(1 Point)
Nephritis -- also HT, Nephrotic syndrome, diffuse lupus nephritis etc
Fever and leukocytosis
Pleural and pericardial effusion
Anti-ds DNA antibodies

19
19,

The normal urine output of an infant is: (1correct answer)


. Single choice.
(1 Point)
1ml/kg/hr - child
2ml/kg/hr
4ml/kg/hr
3ml/kg/hr
Neonates - 2-3

20
20,

A newborn baby with XX karyotype has ambiguous genitalia. Which is the


possible cause?: (1correct answer)
. Single choice.
(1 Point)
Klinefelter syndrome - XXY, well differentiated external genitalia
Congenital adrenal hyperplasia - ͫandrogens
Impaired peripheral androgen metabolism - (-) fb?
Both b/ and c are correct

21
21,

Which of the following signs is NOT found in In kwashiorkor (1correct answer)


. Single choice.
(1 Point)
Hair discoloration - dry, red/yellow/white hair - protein def = <keratin

Downloaded by Kato Caleb (katocalebnatamba@gmail.com)


lOMoAR cPSD| 41174528

Hepatomegaly - fatty infiltration


Generalized oedema
Splenomegaly

22
22,

Which of the following statements about neuroblastoma is correct? :


( 1 correct answer)
. Single choice.
(1 Point)
Hematuria is a common finding - nephroblastoma
Adrenal insufficiency is the most common presenting complaint -most common is painful abdo
pain + constitutional Sx
Survival is best in children beyond age of 2 years at the timepoint of the diagnosis - best in
infants due to chance of spont regression
Spontaneousregressioncouldoccurinsomeinfants
23
mis is 23,
he
Which is the most common causative agent for urinary tract infection:
(1correct answer)
. Single choice.
(1 Point)
Staph. Aureus
E.coli
Str. pneumoniae
viruses

24
24,

What is the typical type of hypoxic-ischemic encephalopathy in premature


infants? (1correct answer)
. Single choice.
(1 Point)
Subdural hematoma
Bilateral damage to the basal ganglia
Periventricularleukomalacia - PVL secondary to ischemia of/-severe intraventricular
hemorrhage or venous infarction is common in premature babies
Stroke in a main cerebral artery

25
25,

Downloaded by Kato Caleb (katocalebnatamba@gmail.com)


lOMoAR cPSD| 41174528

What is NOT true for brachial plexus nerve palsy in neonates? (1correct
answer)
. Single choice.
(1 Point)
Itiscommoninpretermneonates - risk = >larger
It may occur after deliveries with shoulder dystocia - dystocia - when should gets stuck and cant
exit pelvis - cause of brachial palsy + also macrosomia (large baby)
Upper nerve root (C5-C6) is more common It involves the lower roots (C8, T1) of the brachial
plexus.
It can be accompanied by phrenic nerve palsy
26
26,

Jaundice in the newborn is characterized by the following features: (1correct


answer)
. Single choice.
(1 Point)
Jaundice in the second week after birth is most likely an expression of hemolytic disease - <24
hrs
Brain damage is due to direct bilirubin.- kernicterus = UCB
Inhemolyticjaundice,flavinicterisobserved - yellow/flavin = hemolytic, rubin - hepatitis, verdin -
post-hepatic
Indirect (unconjugated) bilirubin is found in the urine - no bilirubin in urine (UCB - not water sol),
stool darker
27
27,

Which of the following conditions is causing melaena in childhood: (1correct


answer)
. Single choice.
(1 Point)
Meckel'sdiverticulum- most commonly 2Tfh eeet s
from ileocecal valve
Polyposis coli
Anal fissure - hematocheztia
Ulcerative colitis - hematochezia

28
28,

Which disease is characterized by severely impaired digestion of the fats and


proteins of the general food, causing bulky fatty and very foul-smelling feces?
(1correct answer)
. Single choice.

Downloaded by Kato Caleb (katocalebnatamba@gmail.com)


lOMoAR cPSD| 41174528

(1 Point)
Celiac disease
Exudative enteropathy
Lactase deficiency
Cysticfibrosis

29
29,

Perinatal mortality is equal to: (1 correct answer, 2 points):


. Multiple choice.
(1 Point)
Stillbirths per 1000 total births
Stillbirths+firstweekdeathsper1000totalbirths
Deaths in first 28 days per 1000 live births
Deaths in first year per 1000 live births

30
30,

In a child with atrial septal defect (ostium secundum defect): (1correct answer)
. Single choice.

(1 Point)
heart failure is common - in large VSD
major limitation of exercise tolerance is likely - often aSx due to minor pressure + vol shunt
closure should be performed by school age - around 3-5 yrs
there is systolic murmur due to mitral regurgitation - Primum

31 this gives it away


31,

t
distension and palpable soft mass above the pubic symphysis. By bimanual
A male newborn baby presents with oligohydramnion, oliguria, abdominal
palpation bilaterally enlarged kidneys are detected. The ultrasound shows also
ascites. The lab tests show creatinine: 110 mcmol/l and BUN: 23 mmol/l. What
is your probable diagnosis? (1 correct answer)
. Single choice.
(2 Points)
Hydrops fetus universalis
Bilateral nephroblastoma - older children around 2-3
Congenital neuroblastoma
Posteriorurethralvalves - flap of tissue in male obstructing urethra -- blocking urine flow --
oligohydramnion, oliguria, abdo distention etc

Downloaded by Kato Caleb (katocalebnatamba@gmail.com)


lOMoAR cPSD| 41174528

32
32,

Newborn to a mother with gestational diabetes is born by cesarean section at


37 p.m. with birth weight 3900 g, height - 51 cm, head circumference - 36 cm.
Apgar - 8; 9. Difficult postpartum adaptation, placed on oxygen for two hours,
then given for breastfeeding to the mother. On the second day, the mother
reported irritability, posturing backwards and trembling of the limbs. Soon
afterwards the baby became lethargic. Hgb - 160 g / l, RBC - 5.5x1012/l, WBC
- 12.2x109/l, Tr - 410x109/l, ESR - 12 mm. CRP - 0.1 g/l, blood sugar - 1.8
mmol/l, calcium - 2.1 mmol/l. What is the cause of the seizures? (1 correct
answer)
. Single choice.
(2 Points)
Cerebral hemorrhage
Neonatal sepsis
Neonatalhypoglycaemia
Neonatal hypcalcemiaя 1

33
33,

A 5-year-old girl has been unwell for 2 weeks with fever, lethargy and pain in
her wrists and knees. On examination she has bouts of Ifever of 38.5°C, during
which a subtle2erythematous rash on her trunk is observed.3Her wrists and triadcomplete
knees are warm, swollen and painful to move. Her liver is 3 cm below the
costal margin and her spleen is palpable. Her blood tests show: Hgb 98 g/l,

WBC 18x109 L, neutrophils 80%, Plt 360x 109/L, ESR 70 mm/h; blood film-
normal, no atypical lymphocytes; ASAT, ALAT, LDH – within reference range;

ANA and anti-dsDNA- negative. What is the most likely diagnosis?

Downloaded by Kato Caleb (katocalebnatamba@gmail.com)


lOMoAR cPSD| 41174528

(1 correct answer)
. Single choice.
(2 Points)
Infectious mononucleosis
Systemic-onsetjuvenileidiopathicarthritis- fever, salmon pink rash- typical for systemic JIA,
also arthritis Sx - ANA + Anti-dsDNA negative but not specific for JIA -- only for SLE
Food allergy
Kawasaki's disease - red rash, joint not affected
Systemic Lupus Erythematosus - ANA + Anti-dsDNA negative

34
34,

A 5-year-old child is admitted to the emergency department in with impaired


conscioness (Glasgow coma scale - 6-7 points), pallor of the skin and mucous
membranes, blood pressure - 80/40, pulse - 130/minute, palpated on all four
extremities, recoloration time of 5 sec., diffuse abdominal ternderness and
vague peristalsis. The same day he was riding a bicycle with friends, and
according to the children, fell from the bike hitting his abdomen. What is the
probable cause of the condition? (1 correct answer)
. Single choice.
(2 Points)
Aortic dissecation
Adrenal gland haematoma
Intracranial hemorrhage
Ruptureofthespleen

Downloaded by Kato Caleb (katocalebnatamba@gmail.com)


lOMoAR cPSD| 41174528

35
35,

A 5-year old girl complains of night bone pains in the two legs, malaise and
unwillingness to play with other children. She was previously healthy. On
physical examination she is pale, afebrile, no jaundice or rash, with
splenomegaly at 3 cm below the costal margin. No lymphadenopathy. No

other pathological signs. Her CBC: HGB- 73 g/l, WBC- 2.3x109/l, PLT-
62x109/l. What is the most likely cause of these complaints? (1 correct

answer)
. Single choice.
(2 Points)
Systemic form of Juvenile idiopathic arthritis
Acuteleukemia
Growing pains
Bone tumor

36
36,

A 4-year-old girl is diagnosed with cystic fibrosis at age 2 years, but was not
checked regularly in the last years. Now she is admitted to the clinic for
jaundice, pneumonia, liver at 3 cm below costal margin. Paraclinical data were
elevated transaminases, gamma-glutamil transpeptidase and apkaline
phosphatase, direct hyperbilirubiemia, normal albumin and prothrombin time.
Which extrapulmonary complication of cystic fibrosis you consider most likely?
(1 correct answer)
. Single choice.
(2 Points)
Chronic viral hepatitis C.
Bile sludge syndrome
Liver cirrhosis
Non-alcoholicsteatosisoftheliver - + focal biliary cirrhosis - common hepatobiliary
manifestations of CF

37
37,

A 6-month-old infant that fell asleep in full health wakes at night with
screaming, bents his legs to the abdomen, refuses to eat being restless. Very
strong colic-like pain was observed, combined with pallor and repeated biliary
vomiting. Objectively: impaired general condition, pale skin, sunken eyes. The

Downloaded by Kato Caleb (katocalebnatamba@gmail.com)


lOMoAR cPSD| 41174528

abdomen is distended and elongated, sausage-like structure is found by


palpation. Intestinal peristalsis is enhanced, but there is no actual passage.
Rectal touche reveals bloody mucus of the fingertips. What is the diagnosis
and what is the behavior? (1 correct answer)
. Single choice.
(2 Points)
Mekel's diverticulum, scintigraphy, surgical treatment
Infectious enterocolitis, fecal culture to find etiology
Allergy to cow's milk protein, specific dietция 1
Iliocecalintussusception,pneumocolonoscopy
38
38,

A one-year-old male patient was referred to the Gastroenterology Department


due to an unusual chronic constipation, associated with abdominal distension,
since six months of age. The baby was born at home and had not been seen
by a medical professionalist up to now. Objectively: failure to thrive, severe
developmental delay, bradycardia, rarefied hair and eyebrows, hoarse cry,
and macroglossia, unable to sit without support or say simple words. What
laboratory tests and procedures will you consider:

(1 correct answer)
. Single choice.
(2 Points)
Suction rectal biopsy
Sweat test
Coeliac disease screening
Thyroidhormones - typical Sx - pot belly, abdo distention, constipation, macroglossia etc

Downloaded by Kato Caleb (katocalebnatamba@gmail.com)


lOMoAR cPSD| 41174528

39
39,

A 2-year-old boy became ill with gastroenteritis - repeated vomiting during the
day and watery stools, fever up to 38 degrees. A few hours later he became
lethargic, sleepy and the mother noticed that he had not urinated for the last 4-
5 hours. Heart rate 140 / min. Blood pressure 85/50. What treatment is
necessary? (1 correct answer)
. Single choice.
(2 Points)
Start intravenous rehydration with only glucose 5%
Startintravenousrehydrationwith0,9%NaCl
Apply mannitol for brain oedema
Apply furosemide for acute renal impairment
40
40,

This 3-month-old infant was brought to a well-baby clinic. She was born from a
seventh pregnancy in a family with low social status. Born at home. She has
not been examined by a doctor so far. Mother complained of poor sucking.
The weight gain was aromd 400g per month. Besides the appearance seen in
the photo, the girl had generalized muscular hypotension, heart rate 160 min,
clear tones, 2/3 endoapical heart murmur, blood pressure 75/40,
hepatomegaly 4cm. She lagged behind in his neuropsychological
development. What is the probable diagnosis? (1 correct answer)
. Single choice.

(2 Points)

Downloaded by Kato Caleb (katocalebnatamba@gmail.com)


lOMoAR cPSD| 41174528

Williams syndrome with pulmonary stenosis


Atrial septal deffect in Down syndrome
VentricularseptaldefectinDownsyndrome - apical murmur
Mitral valve regurgitation in mucopolysaccharidosis
41
41,

A 14 year old girl with normal development up to now suffers from


oligomenorrhea - last regular menstruation was 2 months ago. She gained
weight- 15 kg for three months. She complains of increased fatigue and
drowsiness. Family history of thyroid disease and diabetes. On examination
she is apathetic, with poor facial expressions, BP 110/60; heart rate 55 / min.
No significant deviations in the rest of the physical status. What will you do
first? (1 correct answer)
. Single choice.
(2 Points)
send for psychiatric consultationия 1
examineTSHandfT4 - prob hypothyroidim
perform heart ultrasound
examine DHEA-S

42
42,

At a prophylactic lab test of a clinically healthy 12-year-old girl, the following


were identified: Hgb 105 g / l, Er- 5.55 x10 / 12 / l, MCV 65 fl, WBC- 6.8 x10 /
9 / l, PLT-243 x10 / 9 / l. The father mentions that years ago, in a prophylactic
CBC test, he also had low hemoglobin, but he had no complaints, too, and
has not done any other tests since. What is the most likely diagnosis (1
correct answer)?
. Single choice.
(2 Points)
Acute hemolytic anemia
Thalassemiaminor - aSx, father has same pathology
Vitamin B12 deficiency
Iron deficiency anemia

43
43,

A 4 years old boy returns home in the afternoon after playing in the
countryside during the summer vacation. He has fever 37,6oC, pulse rate 140/
min, dry mouth and mydriais. What is the probable disease? (1 correct
answer)
I Belladonna

Downloaded by Kato Caleb (katocalebnatamba@gmail.com)


lOMoAR cPSD| 41174528

. Single choice.
(2 Points)
Heat shock
Onset of chickenpox, possibly complicated by encephalitis
Belladonnapoisoning
Onset of an acute viral infection

44
44,

A-10-year-old boy has been complaining of fever and sore throat for the last 2
days. On examination there is pharyngitis. The rest of his examination is
unremarkable. His rapid streptococcal antigen test is positive, so he received
oral antibiotic for 10 days. 4 weeks later he returns because of palpitations
and fatigue, especially after physical exercise. His knees are swollen, red and
very painful. It becomes clear that his mother gave him the prescribed
antibiotic for only 2 days as his fever and sore throat subsided quickly. On
examination he has HR 130/min and systolic murmur at the apex, radiating to
the left axilla. His liver is palpable 3 cm below the costal margin. He also has a
wide spread rash. (1 out of 4 correct answers)
. Single choice.
(2 Points)

ƒ
New P
A
F does M

Rheumaticfever - arthritis, murmur, rash (erythema marginatum) etc


Post-streptococcal arthritis
F J
Infective endocarditis A
Infectious mononucleosis R
N
45 M E
45,

A 13-year old girl was admitted for midface rash, fever up to 37,8oC. ESR
120mm, leukocytes 2,8x109/l, platelets 110x109/l and proteinuria was 1g/l .
Which of the following additional results are most probable? (1 correct
answer)

Downloaded by Kato Caleb (katocalebnatamba@gmail.com)


lOMoAR cPSD| 41174528

. Single choice.
(2 Points)
Bone marrow infiltration with abnormal cells
Anti-streptolysin O (ASLO) antibodies in blood
Leukocytoiclastic vasculitis in skin biopsy
Ant-dsDNAantibodiespositiveinblood - SLE - midface rash (butterfly rash) fever, elevated
ESR, leukopenia, thrombocytopenia
46
46,

A 3-year-old child with a 2-week history of abdominal pain presented with a


palpable abdominal mass. The child was otherwise healthy and the parents
reported no other complaints. An abdominal CT scan showed this polar renal
tumor with some calcifications extending beyond the renal capsule to the
adrenal gland. There was no thrombus in the renal vein and the chest CT
scan was normal. (1 correct answer)

. Single choice.
(2 Points)
Neuroblastoma
Non-Hodgkin lymphoma
Rhabdomyosarcoma
Wilms tumor

47
47,

A 6-year-old girl has been referred because of weight loss without food
reduction in the past 4 weeks. Several times she wetted the bed during the

Downloaded by Kato Caleb (katocalebnatamba@gmail.com)


lOMoAR cPSD| 41174528

night after having stopped for 4 years. The last day she vomited several times.
On physical examination her face was red, her buccal mucosa - dry and she
had breathing rate 30 breaths/min without pulmonary findings. Which are the
most adequate lab tests for making diagnosis? (1-5 correct answers, +1 point
for correct answer, -1 point for wrong answer)
. Multiple choice.
urineanalysisforketonesandglucose
bloodglucoselevel
urine culture renal
ultrasound acid
baseanalysis
Probs DKA
48
48,

A 8 month-old infant from first normal pregnancy with birth weight 3500 g,
height 51 cm and head circumference 35 cm. The GP referred the patient
because of a current body weight of 6.0 kg. The mother reported that she
weaned the baby from breastfeeding at 4 months of age and after that the
child was fed on demand and preferred cereals. On physical examination
there is decreased skin turgor and subcutaneous fat on the body and the
extremities of the child. Which of the following statements is true? (1-5 correct
answers, +1 point for a correct answer, -1 point for a wrong answer)
. Multiple choice.
The growth rate of the child is normal
There is first degree of undernutrition in this case - ͭsubcut fat in body + extremities = 2nd
degree
The feeding should start with formula at least 6x daily
Workupforceliacdiseaseismandatory- introduction of gluten, well fed but poor weight gain
The amount of the single portion should be 140 ml at the start of appropriate diet

49
49,

A 10 month old boy of a 42 year old mother from first unfollowed pregnancy.
He was born at term by normal vaginal delivery. He can sit with support. He is
not crawling or pulling up to stand. On examination he has muscle hypotonia
with hyperreflexia. He also has dysmorphic features – flat occipit, mongoloid
eyelids, medial epicanthal fold. Which of the following investigations are most
appropriate? (1-5 correct answers, +1 point for a correct answer, -1 point for a
wrong answer)
. Multiple choice.
Congenital infections screen
EEG

Downloaded by Kato Caleb (katocalebnatamba@gmail.com)


lOMoAR cPSD| 41174528

poor development

Cranialultrasoundscan y
Screeningforinbornerrorsofmetabolism dysmorphicCenter
Chromosomekaryotype
down
50
50,

A full-term newborn develops jaundice within the first 24 hours. Jaundice is


mainly due to indirect bilirubin. Anemia, mild hepatosplenomegaly and the
presence of erythroblasts in the peripheral blood are found. Which of the

following statements is true? (1-5 correct answers, + 1 point for correct


answer, -1 point for wrong):
. Multiple choice.

ItisprobablethenewborntobeRh+andthemothersRh-
Phototherapyshouldbeapplied
No treatment is needed; just monitor bilirubin levels
Thebilirubinlevelismonitoredevery4hoursandincaseofincrease>150mmol/lanexchange

i - incorrect units -- should be microƧŰՖ


transfusionshouldbeundertaken
Gastric lavage is necessay

51
51,

Which of the following characteristics EXCLUDES the diagnosis functional


abdominal pain (irritable bowel syndrome) (1-5 correct answers, + 1 point for
correct answer, -1 point for wrong):
. Multiple choice.
Pain has variable characteristics: sharp, blunt, permanent or colic-like - varies bet patients
Painawakensthepatientatnight.
Objectiveexaminationandlaboratoryparametersareabnormal - clinical Dx
Painislocalizedaroundtheumbilicus
Pain is manifested more often at the age between 4 - 13 years.

52
52,

An 18-month-old boy has had a fever up to 39 C for the past 7 days. His GP
prescribed Cefuroxime for upper respiratory tract infection with sore throat 5
days ago but there is no improvement so far. Three days ago he developed a
rash and his parents noticed a swelling on the right side of his neck. On
examination you also establish bilateral conjunctivitis, red and cracked lips,

Downloaded by Kato Caleb (katocalebnatamba@gmail.com)


lOMoAR cPSD| 41174528

erythema and oedema of his palms and soles as well as intense erythema
with desquamation in the nappy area. There is only unilateral neck
lymphadenopathy. No hepatosplenomegaly.. Normal blood counts and
transaminases Which of the following tests will help you most to make the
diagnosis? (1-5 correct answers, + 1 point for correct answer, -1 points for
wrong):
. Multiple choice.
serology for EBV
conjunctival culture
neck computed tomography
bone marrow biopsy
Echocardiography- coronary a aneurysm

53
53,

A 2-years old boy was taken to the Emergency ward for high fever 39oC.
Upon admission he was found with pale face and mottled skin. The capillary
refill time was 5 sec., heart rate 180/min, arterial pressure 75/30. ESR was
80mm/hr, Hb 115g/l, Er 3,5x1012/l, Leuc 27x109/l, Plt70x109/l. What is your
best explanation for this condition? (1 of 4 correct answers;
. Single choice.
(2 Points)

Pancytopenia
Septicshock
Acute anemia
Shivering during raising body temperature in the course of a viral infection

54
54,

Q53 - What should be your treatment at that point? (1 of 4 correct answers)


. Single choice.
(2 Points)
Infusion of saline 2ml/kg for 2 hrs and start broad spectrum antibiotic i.v.
Transfusion of packed erythrocytes
Infusionofsaline20ml/kgfor20minandstartbroadspectrumantibiotici.v. Apply
adrenalin s.c. and antihistamines

55
55,

Q53 - After 1/2 hour a rash appeared on the buttocks and progressed rapidly.

Downloaded by Kato Caleb (katocalebnatamba@gmail.com)


lOMoAR cPSD| 41174528

What diagnosis do you suggest? (1 of 4 correct answers)

. Single choice.
(2 Points)
Schoenlein-Henoch purpura
Acute urticaria
Meningococcemia
Acute thrombocytopenia

56
56,

A 2-month-old infant presented with a one week history of dry spastic cough,
low-grade fever and tachypnea exceeding 80 breaths/minute in the last 2
days. On the admission there are no pulmonary findings from the physical
exam. The chest X-ray shows bilateral ground-glass opacities. Negative
inflammatory markers from the lab tests. The levels of IgG, IgM and IgA are
extremely low. Acid-base analysis shows severe hypoxia and low pCO2. What
is your probable diagnosis? (1 correct answer)
. Single choice.

(2 Points)

Downloaded by Kato Caleb (katocalebnatamba@gmail.com)


lOMoAR cPSD| 41174528

Bronchiolitis
Staphylococcal pneumonia
Pneumocystisjirovesii(carinii)pneumonia -dry cough, frosted/ground glass opacity
Viral pneumonia

57
57,

Q56 - Which is the most probable underlying primary immune deficiency? (1


correct answer)
. Single choice.
(2 Points)
X-linked agammaglobulinemia
Severe congenital neutropenia
Chronic granulomatous disease
Severecombinedimmunedeficiency - immunocompromised people -- also hyper IgM
syndrome, immunosuppressive Tx e.g CS, malignancy, malnutrition, HIV
Primary immune def = severe combined, hyper IgM syndrome

58 di hd
58, pneumonia caused by the organism pneumocystisjiroveci
Karimi
Q56 - Which is the appropriate treatment (1 correct answer)
. Single choice.
(2 Points)
Life-long substitution with immunoglobulins
Bone marrow transplantation asap
i.v. Pentamidine and Trimetoprim/Sulfamethoxazole
sure?
Broad spectrum antibiotics

59
59,

A 15-year-old girl attends the Emergency Department with her father as she
has ‘turned yellow’. She reported that she has taken some "pills" two days
ago. She is being bullied at school, which has been going on for over a year
now. On examination, she is jaundiced and her liver function tests are
deranged. What is the most likely poison: (1 correct answer)
. Single choice.
(2 Points)
Paracetamol - hepatotoxicity
Tricyclic antidepressants - SNS Sx e.g dry mouth, constipation, blurred vision + arrhythmias etc
Phenobarbital - children - paradoxical irritability, hyperactivity etc

Downloaded by Kato Caleb (katocalebnatamba@gmail.com)


lOMoAR cPSD| 41174528

Beta blockers

60
60,

Q59 - What is the appropriate management in this case: (1 correct answer)


. Single choice.
(2 Points)
IntravenousN-acetylcysteine
Gastric lavage
Bolus doses of Atropine
Forced diuresis with i.v. sodium bicarbonate
61
61,

Q59 - What are the most likely complications to expect: (1 correct answer)
. Single choice.
(2 Points)
Coagulopathy- ͭsynthesis by liver
Rhythm-conduction disorders
Gastric ulcer
Severe dehydration and metabolic acidosis

Q.60 A 1year 7 months old boy from normal pregnancy and delivery is taken to
pediatrician because he does not respond when called and says no distinct
words. He is able to walk, scribble with pen and feed himself, startles to loud
noises, but does not point to the objects he wants and does not look at his
mother’s face. What is the most appropriate investigation? (1 correct answer)
. Single choice.
(2 Points)
Reassurance of the parents
Perform a MRI
Assessmentforautismspectrumdisorder
Hearing test

Downloaded by Kato Caleb (katocalebnatamba@gmail.com)


lOMoAR cPSD| 41174528

Downloaded by Kato Caleb (katocalebnatamba@gmail.com)


lOMoAR cPSD| 41174528
Downloaded by Kato Caleb (katocalebnatamba@gmail.com)

You might also like